Yr12 Maths in Focus 2U HSC

August 12, 2017 | Author: Richard Yang | Category: Rectangle, Area, Triangle, Tangent, Calculus
Share Embed Donate


Short Description

Download Yr12 Maths in Focus 2U HSC...

Description

1

Geometry 2 TERMINOLOGY Congruent: Two figures are congruent if they have the same size and shape. They are identical in every way

are equal and corresponding sides are in the same ratio

Polygon: A polygon is a closed plane figure with straight sides

Vertex: A vertex is a corner of a figure (vertices is plural, meaning more than one vertex)

Similar: Two figures are similar if they have the same shape but a different size. Corresponding angles

Chapter 1 Geometry 2

INTRODUCTION YOU STUDIED GEOMETRY IN the Preliminary Course. In this

chapter, you will revise this work and extend it to include some more general applications of geometrical properties involving polygons. You will also use the Preliminary topic on straight-line graphs to explore coordinate methods in geometry.

Plane Figure Geometry Here is a summary of the geometry you studied in the Preliminary Course.

Vertically opposite angles

+AEC and +DEB are called vertically opposite angles. +AED and +CEB are also vertically opposite angles.

Vertically opposite angles are equal.

Parallel lines If the lines are parallel, then alternate angles are equal.

If the lines are parallel, then corresponding angles are equal.

3

4

Maths In Focus Mathematics HSC Course

If the lines are parallel, cointerior angles are supplementary (i.e. their sum is 180°).

TESTS FOR PARALLEL LINES If alternate angles are equal, then the lines are parallel. If +AEF = +EFD, then AB || CD.

If corresponding angles are equal, then the lines are parallel. If +BEF = +DFG, then AB || CD.

If cointerior angles are supplementary, then the lines are parallel.

If +BEF + +DFE = 180c, then AB || CD.

Angle sum of a triangle

The sum of the interior angles in any triangle is 180°, that is, a + b + c = 180

Chapter 1 Geometry 2

Exterior angle of a triangle

The exterior angle in any triangle is equal to the sum of the two opposite interior angles. That is, x+y=z

Congruent triangles Two triangles are congruent if they are the same shape and size. All pairs of corresponding sides and angles are equal. For example:

We write Δ ABC / Δ XYZ. TESTS To prove that two triangles are congruent, we only need to prove that certain combinations of sides or angles are equal. Two triangles are congruent if • SSS: all three pairs of corresponding sides are equal • SAS: two pairs of corresponding sides and their included angles are equal • AAS: two pairs of angles and one pair of corresponding sides are equal • RHS: both have a right angle, their hypotenuses are equal and one other pair of corresponding sides are equal

Similar triangles Triangles, for example ABC and XYZ, are similar if they are the same shape but different sizes. As in the example, all three pairs of corresponding angles are equal. All three pairs of corresponding sides are in proportion (in the same ratio).

The included angle is the angle between the 2 sides.

5

6

Maths In Focus Mathematics HSC Course

We write: Δ ABC 0, the curve is increasing If f l] x g < 0, the curve is decreasing If f l] x g = 0, the curve is stationary

A curve is monotonic increasing or decreasing if it is always increasing or decreasing; that is, if f l] x g > 0 for all x (monotonic increasing) or f l] x g < 0 for all x (monotonic decreasing)

EXAMPLES 1. Find all x values for which the curve f ] x g = x2 − 4x + 1 is increasing.

Solution f l] x g = 2x − 4 f l] x g > 0 for increasing curve i.e. 2x − 4 > 0 2x > 4 x>2 So the curve is increasing for x > 2.

This function is a parabola.

CONTINUED

34

Maths In Focus Mathematics HSC Course

2. Find the stationary point on the parabola y = x2 − 6x + 3.

Solution dy dx

= 2x − 6 dy

=0 dx i.e. 2x − 6 = 0 2x = 6 x=3 2 ] When x = 3, y = 3 − 6 3 g + 3 = −6 For stationary points,

So the stationary point is ^ 3, − 6 h.

3. Find any stationary points on the curve y = x3 − 48x − 7.

Solution yl = 3x2 − 48 For stationary points, yl = 0 i.e. 3x2 − 48 = 0 3x2 = 48 x2 = 16 ` x = ±4 When x = 4, y = 43 − 48 (4) − 7 = − 135 You will use stationary points to sketch curves later in this chapter.

When x = − 4, y = ] − 4 g3 − 48 (− 4) − 7 = 121 So the stationary points are ^ 4, −135 h and ^ − 4, 121 h.

Chapter 2 Geometrical Applications of Calculus

35

PROBLEM What is wrong with this working out? Find the stationary point on the curve y = 2x2 + x − 1.

Solution yl = 4x + 1 For stationary points, yl = 0 i.e. 4x + 1 = 0 4x = − 1 x = − 0.25 When x = − 0.25, y = 4 (− 0.25) + 1 = −1 + 1 =0 So the stationary point is ^ − 0.25, 0 h.

Can you find the correct answer?

2.1 Exercises 1.

Find the parts of each curve where the gradient of the tangent is positive, negative or zero. Label each curve with +, − or 0.

(d)

(a)

2.

Find all values of x for which the curve y = 2x 2 − x is decreasing.

3.

Find the domain over which the function f ] x g = 4 − x 2 is increasing.

4.

Find values of x for which the curve y = x 2 − 3x − 4 is (a) decreasing (b) increasing (c) stationary.

5.

Show that the function f ] x g = − 2x − 7 is always (monotonic) decreasing.

(b)

(c)

36

Maths In Focus Mathematics HSC Course

6.

Prove that y = x 3 is monotonic increasing for all x ≠ 0.

7.

Find the stationary point on the curve f (x) = x 3.

8.

Find all x values for which the curve y = 2x 3 + 3x 2 − 36x + 9 is stationary.

9.

Find all stationary points on the curve (a) y = x 2 − 2x − 3 (b) f ] x g = 9 − x 2 (c) y = 2x 3 − 9x 2 + 12x − 4 (d) y = x 4 − 2x 2 + 1.

17. Sketch a function with f l] x g > 0 for x < 2, f l] 2 g = 0 and f l] x g < 0 when x > 2. 18. Draw a sketch showing a curve dy dy with < 0 for x < 4, = 0 when dx dx dy x = 4 and > 0 for x > 4. dx 19. Sketch a curve with x ≠ 1 and

dy dx

dy dx

> 0 for all

= 0 when x = 1.

10. Find any stationary points on the curve y = ] x − 2 g4.

20. Draw a sketch of a function that has f l] x g > 0 for x < −2, x > 5, f l] x g = 0 for x = − 2, 5 and f l] x g < 0 for − 2 < x < 5.

11. Find all values of x for which the curve f (x) = x 3 − 3x + 4 is decreasing.

21. A function has f (3) = 2 and f l] 3 g < 0. Show this information on a sketch.

12. Find the domain over which the curve y = x 3 + 12x 2 + 45x − 30 is increasing.

22. The derivative is positive at the point (−2, −1). Show this information on a graph.

13. Find any values of x for which the curve y = 2x 3 − 21x 2 + 60x − 3 is (a) stationary (b) decreasing (c) increasing.

23. Find the stationary points on the curve y = ] 3x − 1 g ] x − 2 g4.

14. The function f ] x g = 2x 2 + px + 7 has a stationary point at x = 3. Evaluate p. 15. Evaluate a and b if y = x 3 − ax 2 + bx − 3 has stationary points at x = − 1 and x = 2. 16. (a) Find the derivative of y = x 3 − 3x 2 + 27x − 3. (b) Show that the curve is monotonic increasing for all values of x.

24. Differentiate y = x x + 1 . Hence find the stationary point on the curve, giving the exact value. 25. The curve f (x) = ax 4 − 2x 3 + 7x 2 − x + 5 has a stationary point at x = 1. Find the value of a. 26. Show that f (x) = x has no stationary points. 1 has no x3 stationary points.

27. Show that f ] x g =

Chapter 2 Geometrical Applications of Calculus

37

Types of Stationary Points There are three types of stationary points.

Local minimum point

The curve is decreasing on the left and increasing on the right of the minimum turning point. x f l] x g

LHS

Minimum

RHS

0

Local maximum point

The curve is increasing on the left and decreasing on the right of the maximum turning point. x f l] x g

LHS

Maximum

RHS

>0

0

0

0

>0

or

0 then f l(x) is increasing if f m(x) < 0 then f l(x) is decreasing if f m(x) = 0 then f l(x) is stationary

Chapter 2 Geometrical Applications of Calculus

43

Concavity If f m] x g > 0 then f l] x g is increasing. This means that the gradient of the tangent is increasing, that is, the curve is becoming steeper.

f´ (x) is increasing since a 1 gradient of − is greater 2 than a gradient of −6.

Notice the upward shape of these curves. The curve lies above the tangents. We say that the curve is concave upwards. If f m] x g < 0 then f l] x g is decreasing. This means that the gradient of the tangent is decreasing. That is, the curve is becoming less steep.

f´ (x) is decreasing since 1 −4 is less then − . 2

Notice the downward shape of these curves. The curve lies below the tangents. We say that the curve is concave downwards. If f m] x g = 0 then f l] x g is stationary. That is, it is neither increasing nor decreasing. This happens when the curve goes from being concave upwards to concave downwards, or when the curve changes from concave downwards to concave upwards. We say that the curve is changing concavity.

The point where concavity changes is called an inflexion.

These points of inflexion are called horizontal inflexions, as the tangent is horizontal.

The curve has a point of inflexion as long as concavity changes.

44

Maths In Focus Mathematics HSC Course

Class Investigation How would you check that concavity changes?

• If f m] x g > 0, the curve is concave upwards. • If f m] x g < 0, the curve is concave downwards. • If f m] x g = 0 and concavity changes, there is a point of inflexion.

EXAMPLES 1. Does the curve y = x 4 have a point of inflexion?

Solution dy dx d2 y dx2

= 4x 3 = 12x2

i.e.

The sign must change for concavity to change.

d2 y

=0 dx2 12x2 = 0 x=0

For inflexions,

When x = 0, y = 04 =0 So ^ 0, 0 h is a possible point of inflexion. Check that concavity changes: x d y

−1

0

1

12

0

12

2

dx2 What type of point is it?

Since concavity doesn’t change, (0, 0) is not a point of inflexion.

Chapter 2 Geometrical Applications of Calculus

2. Find all values of x for which the curve f ] x g = 2x 3 − 7x 2 − 5x + 4 is concave downwards.

Solution f l(x) = 6x2 − 14x − 5 f m(x) = 12x − 14 For concave downwards, f m(x) < 0 12x − 14 < 0 12x < 14 1 ` x 1 and f m] x g > 0 for x < 1. 13. Find any points of inflexion on the curve y = x4 − 8x3 + 24x2 – 4x − 9. 14. Show that f ] x g =

2 is concave x2

upwards for all x ≠ 0.

15. For the function f ] x g = 3x5 − 10x3 + 7 (a) Find any points of inflexion.

(b) Find which of these points are horizontal points of inflexion (stationary points). 16. (a) Show that the curve y = x4 + 12x2 − 20x + 3 has no points of inflexion. (b) Describe the concavity of the curve. 17. If y = ax3 − 12x2 + 3x − 5 has a point of inflexion at x = 2, evaluate a. 18. Evaluate p if f ] x g = x4 − 6px2 − 20x + 11 has a point of inflexion at x = − 2. 19. The curve y = 2ax4 + 4bx3 − 72x2 + 4x − 3 has points of inflexion at x = 2 and x = − 1. Find the values of a and b. 20. The curve y = x6 − 3x5 + 21x − 8 has two points of inflexion. (a) Find these points. (b) Show that these points of inflexion are not stationary points.

If we combine the information from the first and second derivatives, this will tell us about the shape of the curve.

EXAMPLES 1. For a particular curve, f ] 2 g = −1, f l] 2 g > 0 and f m] 2 g < 0. Sketch the curve at this point, showing its shape.

Solution f ] 2 g = −1 means that the point ^ 2, −1 h lies on the curve. If f l] 2 g > 0, the curve is increasing at this point. If f m ] 2 g < 0, the curve is concave downwards at this point.

Chapter 2 Geometrical Applications of Calculus

47

2. The curve below shows the number of unemployed people P over time t months.

d2 P dP and 2 . dt dt (b) How is the number of unemployed people changing over time? (c) Is the unemployment rate increasing or decreasing? (a) Describe the sign of

Solution (a) The curve is decreasing, so

dP < 0 and the curve is concave upwards, dt

d2 P > 0. dt2 (b) As the curve is decreasing, the number of unemployed people is decreasing. (c) Since the curve is concave upwards, the gradient is increasing. This means that the unemployment rate is increasing. so

Remember that the gradient, or first derivative, measures the rate of change.

48

Maths In Focus Mathematics HSC Course

2.5 Exercises 1.

For each curve, describe the sign dy d2 y of and 2 . dx dx

2.

The curve below shows the population of a colony of seals.

(a)

(a) Describe the sign of the first and second derivatives. (b) How is the population rate changing?

(b)

(c)

3.

Inflation is increasing, but the rate of increase is slowing. Draw a graph to show this trend.

4.

Draw a sketch to show the shape of each curve: (a) f l] x g < 0 and f m ] x g < 0 (b) f l] x g > 0 and f m] x g < 0 (c) f l] x g < 0 and f m] x g > 0 (d) f l] x g > 0 and f m] x g > 0

5.

The size of classes at a local TAFE college is decreasing and the rate at which this is happening is decreasing. Draw a graph to show this.

6.

As an iceblock melts, the rate at which it melts increases. Draw a graph to show this information.

7.

The graph shows the decay of a radioactive substance.

(d)

(e)

Describe the sign of

dM d2 M and . dt dt2

Chapter 2 Geometrical Applications of Calculus

8.

9.

The population P of fish in a certain lake was studied over time, and at the start the number of fish was 2500. dP (a) During the study, < 0. dt What does this say about the number of fish during the study? d2 P (b) If at the same time, 2 > 0, d what can you say about the population rate? (c) Sketch the graph of the population P against t. The graph shows the level of education of youths in a certain rural area over the past 100 years.

Describe how the level of education has changed over this period of time. Include mention of the rate of change. 10. The graph shows the number of students in a high school over several years.

Describe how the school population is changing over time, including the rate of change.

Here is a summary of the shape of a curve given the first and second derivatives.

dy dx

d2 y dx2

d2 y dx2

d2 y dx2

>0

0

dy dx

0, there is a minimum turning point.

The curve is concave upwards at this point.

• If f l] x g = 0 and f m] x g < 0, there is a maximum turning point.

The curve is concave downwards at this point.

• If f l] x g = 0, f m] x g = 0 and concavity changes, then there is a horizontal point of inflexion.

• Minimum turning point: f l] x g = 0, f m] x g > 0. • Maximum turning point: f l] x g = 0, f m] x g < 0. • Horizontal inflexion: f l] x g = 0, f m] x g = 0 and concavity changes.

Chapter 2 Geometrical Applications of Calculus

EXAMPLES 1. Find the stationary points on the curve f ] x g = 2x3 − 3x2 − 12x + 7 and distinguish between them.

Solution f l] x g = 6x2 − 6x − 12 For stationary points, f l] x g = 0 i.e.

`

6x2 − 6x − 12 = 0 x2 − x − 2 = 0 (x − 2) (x + 1) = 0 x = 2 or −1 f (2) = 2 ] 2 g 3 − 3 ] 2 g 2 − 12 (2) + 7 = −13

So (2, −13) is a stationary point. f (−1) = 2 ] −1 g 3 − 3 ] −1 g 2 − 12 (−1) + 7 = 14 So ^ –1, 14 h is a stationary point. Now f m(x) = 12x − 6 f m(2) = 12 (2) − 6 = 18 >0

(concave upwards)

So ^ 2, −13 h is a minimum turning point. f m(−1) = 12 (−1) − 6 = − 18 0

(concave upwards)

` ^ 3, −26 h is a minimum turning point f m(−1) = 6 (−1) − 6 = − 12 0

(concave upwards)

` ^ 2, −1 h is a minimum turning point

Chapter 2 Geometrical Applications of Calculus

Endpoints: f ] − 4 g = ] − 4 g 2 − 4 (− 4) + 3 = 35 f (3) = 3 2 − 4 (3) + 3 =0

The maximum value of y is 35 and the minimum value is −l in the domain − 4 ≤ x ≤ 3. 2. Find the maximum and minimum values of the curve y = x 4 − 2x 2 + 1 for − 2 ≤ x ≤ 3.

Solution yl = 4x 3 − 4x For stationary points, yl = 0 i.e. 4x 3 − 4x = 0 4x (x 2 − 1) = 0 4x (x + 1) (x − 1) = 0 ` x = 0 or ±1 When x = 1, y = 14 − 2 (1) 2 + 1 =0 So (1, 0) is a stationary point. When x = − 1, y = ] − 1 g 4 − 2 ] − 1 g 2 + 1 =0 So ^ − 1, 0 h is a stationary point. When x = 0, y = ] 0 g 4 − 2 ] 0 g 2 + 1 =1 So (0, 1) is a stationary point. Now y m = 12x2 − 4 When x = 1, y m = 12 ] 1 g 2 − 4 =8 >0

(concave upwards) CONTINUED

59

60

Maths In Focus Mathematics HSC Course

So (1, 0) is a minimum turning point. When x = − 1, y m = 12 ] − 1 g 2 − 4 =8 >0

(concave upwards)

So (−1, 0) is a minimum turning point. When x = 0, y m = 12 ] 0 g 2 − 4 = −4 0 dx 2

(concave upwards)

` x = 3 gives a minimum value So 300 items manufactured each year will give the minimum expenses. (c) When x = 3, E = 32 − 6 ] 3 g + 12 =3 So the minimum expense per year is $30 000. 2. The formula for the volume of a pond is given by V = 2h 3 − 12h 2 + 18h + 50, where h is the depth of the pond in metres. Find the maximum volume of the pond.

Solution Vl = 6h 2 − 24h + 18 For stationary points, Vl = 0 i.e. 6h 2 − 24h + 18 = 0 h 2 − 4h + 3 = 0 (h − 1) (h − 3) = 0 So h = 1 or 3 V m = 12h − 24 When h = 1, V m = 12 (1) − 24 = − 12 0

(concave upwards)

Chapter 2 Geometrical Applications of Calculus

So h = 3 gives a minimum V. When h = 1, V = 2 ] 1 g 3 − 12 ] 1 g 2 + 18 ] 1 g + 50 = 58 So the maximum volume is 58 m3.

In the above examples, the equation is given as part of the question. However, we often need to work out an equation before we can start answering the question. Working out the equation is often the hardest part!

EXAMPLES 1. A rectangular prism has a base with length twice its breadth. The volume 900 is to be 300 cm3. Show that the surface area is given by S = 4x2 + x .

Solution

Volume: V = lbh = 2x ´ x ´ h `

300 = 2x 2 h 300 =h 2x 2

(1)

Surface area: S = 2 (lb + bh + lh) = 2 (2x 2 + xh + 2xh) = 2 (2x 2 + 3xh) = 4x 2 + 6xh Now we substitute (1) into this equation. S = 4x 2 + 6x $

300 2x 2

900 = 4x 2 + x 2. ABCD is a rectangle with AB = 10 cm and BC = 8 cm. Length AE = x cm and CF = y cm. CONTINUED

63

64

Maths In Focus Mathematics HSC Course

(a) Show that triangles AEB and CBF are similar. (b) Show that xy = 80. 320 (c) Show that triangle EDF has area given by A = 80 + 5x + x .

Solution (a) +EAB = +BCF = 90° (given) (corresponding +s, AB < DC ) +BFC = +EBA +BEA = +FBC (similarly, AD < BC ) ` Δ AEB and ΔCBF are similar (AAA) 10 x (b) ` y =8 xy = 80

(similar triangles have sides in proportion)

(c) Side FD = y + 10 and side ED = x + 8 A=

1 bh 2

=

1 (y + 10) (x + 8) 2

=

1 (xy + 8y + 10x + 80) 2

We need to eliminate y from this equation. xy = 80, 80 then y = x

If

80 Substituting xy = 80 and y = x into the area equation gives: A= =

1 (xy + 8y + 10x + 80) 2 80 1 (80 + 8 $ x + 10x + 80) 2

640 1 (160 + x + 10x) 2 320 = 80 + x + 5x =

Chapter 2 Geometrical Applications of Calculus

2.9 Exercises 1.

The area of a rectangle is to be 50 m2.

6.

A timber post with a rectangular cross-sectional area is to be cut out of a log with a diameter of 280 mm as shown.

Show that the equation of its 100 perimeter is given by P = 2x + x . 2.

A rectangular paddock on a farm is to have a fence with a 120 m perimeter. Show that the area of the paddock is given by A = 60x − x 2 .

3.

The product of two numbers is 20. Show that the sum of the 20 numbers is S = x + x .

4.

A closed cylinder is to have a volume of 400 cm3.

(a) Show that y = 78400 − x 2 . (b) Show that the cross-sectional area is given by A = x 78400 − x 2 . 7.

A 10 cm ´ 7 cm rectangular piece of cardboard has equal square corners with side x cut out and folded up to make an open box.

Show that its surface area is 800 S = 2π r 2 + r . 5.

A 30 cm length of wire is cut into 2 pieces and each piece bent to form a square.

Show that the volume of the box is V = 70x − 34x2 + 4x3. 8.

(a) Show that y = 30 − x. (b) Show that the total area of the 2 squares is given by A=

x2 − 30x + 450 . 8

A travel agency calculates the expense E of organising a holiday per person in a group of x people as E = 200 + 400x. The cost C for each person taking a holiday is C = 900 − 100x. Show that the profit to the travel agency on a holiday with a group of x people is given by P = 700x − 500x 2 .

65

66

Maths In Focus Mathematics HSC Course

9.

Joel is 700 km north of a town, travelling towards it at an average speed of 75 km/h. Nick is 680 km east of the town, travelling towards it at 80 km/h.

Show that after t hours, the distance between Joel and Nick is given by d = 952400 − 213800t + 12025t 2 .

10. Sean wants to swim from point A to point B across a 500 m wide river, then walk along the river bank to point C. The distance along the river bank is 7 km.

If he swims at 5 km/h and walks at 4 km/h, show that the time taken to reach point C is given by t=

x2 + 0.25 7 − x . + 5 4

When you have found the equation, you can use calculus to find the maximum or minimum value. The process is the same as for finding stationary points on curves. Always check that an answer gives a maximum or minimum value. Use the second derivative to find its concavity, or if the second derivative is too hard to find, check the first derivative either side of this value.

EXAMPLES 1. The council wanted to make a rectangular swimming area at the beach using a straight cliff on one side and a length of 300 m of sharkproof netting for the other three sides. What are the dimensions of the rectangle that encloses the greatest area?

Solution Let the length of the rectangle be y and the width be x.

There are many differently shaped rectangles that could have a perimeter of 300 m.

Chapter 2 Geometrical Applications of Calculus

Perimeter: 2x + y = 300 m ` y = 300 − 2x Area A = xy = x (300 − 2x)

(1) 7 substituting ] 1 g A

= 300x − 2x 2 dA = 300 − 4x dx dA =0 dx 300 − 4x = 0 300 = 4x 75 = x

For stationary points, i.e.

d2 A = −4 < 0 dx2

(concave downwards)

So x = 75 gives maximum A. When x = 75, y = 300 − 2 (75) = 150 So the dimensions that give the maximum area are 150 m × 75 m. 2. Trinh and Soi set out from two towns. They travel on roads that meet at right angles, and they walk towards the intersection. Trinh is initially 15 km from the intersection and walks at 3 km/h. Soi is initially 10 km from the intersection and walks at 4 km/h. (a) Show that their distance apart after t hours is given by D2 = 25t2 − 170t + 325. (b) Hence find how long it takes them to reach their minimum distance apart. (c) Find their minimum distance apart.

Solution (a) After t hours, Trinh has walked 3t km. She is now 15 − 3t km from the intersection. After t hours, Soi has walked 4t km. He is now 10 − 4t km from the intersection.

CONTINUED

67

68

Maths In Focus Mathematics HSC Course

By Pythagoras’ theorem: D2 = ] 15 − 3t g 2 + ] 10 − 4t g 2 = 225 − 90t + 9t2 + 100 − 80t + 16t2 = 25t2 − 170t + 325 (b)

dD 2 = 50t − 170 Dt dD 2 dt 50t − 170 50t t

For stationary points,

=0

i.e.

=0 = 170 = 3 .4

d2 D2 = 50 > 0 dt 2 So t = 3.4 gives minimum D 2 .

(concave upwards)

Now 0.4 ´ 60 minutes = 24 minutes ` 3.4 hours = 3 hours and 24 minutes So Trinh and Soi are a minimum distance apart after 3 hours and 24 minutes. (c) When t = 3.4, D 2 = 25 ] 3.4 g 2 − 170 (3.4) + 325 = 36 D = 36 =6 So the minimum distance apart is 6 km.

2.10 Exercises 1.

The height, in metres, of a ball is given by the equation h = 16t − 4t 2, where t is time in seconds. Find when the ball will reach its maximum height, and what the maximum height will be.

3.

The perimeter of a rectangle is 60 m and its length is x m. Show that the area of the rectangle is given by the equation A = 30x − x 2 . Hence find the maximum area of the rectangle.

2.

The cost per hour of a bike ride is given by the formula C = x 2 − 15x + 70, where x is the distance travelled in km. Find the distance that gives the minimum cost.

4.

A farmer wants to make a rectangular paddock with an area of 4000 m2. However, fencing costs are high and she wants the paddock to have a minimum perimeter.

Chapter 2 Geometrical Applications of Calculus

(a) Show that the perimeter is given by the equation P = 2x +

A box is made from an 80 cm by 30 cm rectangle of cardboard by cutting out 4 equal squares of side x cm from each corner. The edges are turned up to make an open box.

Bill wants to put a small rectangular vegetable garden in his backyard using two existing walls as part of its border. He has 8 m of garden edging for the border on the other two sides. Find the dimensions of the garden bed that will give the greatest area.

(a) Show that the volume of the box is given by the equation V = 4x 3 − 220x 2 + 2400x. (b) Find the value of x that gives the box its greatest volume. (c) Find the maximum volume of the box.

8000 x . (b) Find the dimensions of the rectangle that will give the minimum perimeter, correct to 1 decimal place. (c) Calculate the cost of fencing the paddock, at $48.75 per metre. 5.

6.

Find two numbers whose sum is 28 and whose product is a maximum.

7.

The difference of two numbers is 5. Find these numbers if their product is to be minimum.

8.

A piece of wire 10 m long is broken into two parts, which are bent into the shape of a rectangle and a square as shown. Find the dimensions x and y that make the total area a maximum.

9.

10. The formula for the surface area of a cylinder is given by S = 2π r ] r + h g, where r is the radius of its base and h is its height. Show that if the cylinder holds a volume of 54π m3, the surface area is given by the 108π equation S = 2π r 2 + r . Hence find the radius that gives the minimum surface area. 11. A silo in the shape of a cylinder is required to hold 8600 m3 of wheat.

69

70

Maths In Focus Mathematics HSC Course

(a) Find an equation for the surface area of the silo in terms of the base radius. (b) Find the minimum surface area required to hold this amount of wheat, to the nearest square metre. 12. A rectangle is cut from a circular disc of radius 6 cm. Find the area of the largest rectangle that can be produced. 13. A poster consists of a photograph bordered by a 5 cm margin. The area of the poster is to be 400 cm2.

16. A half-pipe is to be made from a rectangular piece of metal of length x m. The perimeter of the rectangle is 30 m.

(a) Find the dimensions of the rectangle that will give the maximum surface area. (b) Find the height from the ground up to the top of the halfpipe with this maximum area, correct to 1 decimal place. 17. Find the least surface area, to the nearest cm2, of a closed cylinder that will hold a volume of 400 cm3.

(a) Show that the area of the photograph is given by the 4000 equation A = 500 − 10x − x (b) Find the maximum area possible for the photograph.

18. The picture frame shown below has a border of 2 cm at the top and bottom and 3 cm at the sides. If the total area of the border is to be 100 cm2, find the maximum area of the frame.

14. The sum of the dimensions of a box with a square base is 60 cm. Find the dimensions that will give the box a maximum volume. 15. A surfboard is in the shape of a rectangle and semicircle, as shown. The perimeter is to be 4 m. Find the maximum area of the surfboard, correct to 2 decimal places.

19. A 3 m piece of wire is cut into two pieces and bent around to form a square and a circle. Find the size of the two lengths, correct to 2 decimal places, that will make the total area of the square and circle a minimum.

Chapter 2 Geometrical Applications of Calculus

(a) Show that 2400 − 2y − π y x= . 4 (b) Show that the area of the enclosure is given by

20. Two cars are travelling along roads that intersect at right angles to one another. One starts 200 km away and travels towards the intersection at 80 kmh-1, while the other starts at 120 km away and travels towards the intersection at 60 kmh-1. Show that their distance apart after t hours is given by d 2 = 10 000t 2 − 46 400t + 54 400 and hence find their minimum distance apart.

A=

4800y − 4y2 − π y2 . 8

(c) Find the dimensions x and y (to the nearest metre) that maximises the area. 23. Points A ^ − a, 0 h, B ^ 0, b h and O ^ 0, 0 h form a triangle as shown and AB always passes through the point ^ − 1, 2 h .

21. X is a point on the curve y = x 2 − 2x + 5. Point Y lies directly below X and is on the curve y = 4x − x 2.

y B (0, b)

y X

x

A (-a, 0)

x

Y

2a . a−1 (b) Find values of a and b that give the minimum area of triangle OAB.

(a) Show that b =

(a) Show that the distance d between X and Y is given by d = 2x 2 − 6x + 5. (b) Find the minimum distance between X and Y.

24. Grant is at point A on one side of a 20 m wide river and needs to get to point B on the other side 80 m along the bank as shown.

22. A park is to have a dog-walking enclosure in the shape of a rectangle with a semi-circle at the end as shown, and a perimeter of 1200 m.

A

20 m

x

x

80 m

y

B

Grant swims to any point on the other bank and then runs along the side of the river to point B. If he can swim at 7 km/h and run at 11 km/h, find the distance he

71

72

Maths In Focus Mathematics HSC Course

swims (x) to minimise the time taken to reach point B. Answer to the nearest metre.

25. A truck travels 1500 km at an hourly cost given by s 2 + 9000 cents where s is the average speed of the truck. (a) Show that the cost for the trip is given by C = 1500 b s +

9000 l . s

(b) Find the speed that minimises the cost of the trip. (c) Find the cost of the trip to the nearest dollar.

Class Challenge Can you solve either of these problems? 1. Heron’s problem One boundary of a farm is a straight river bank, and on the farm stands a house and some distance away, a shed; each is sited away from the river bank. Each morning the farmer takes a bucket from his house to the river, fills it with water, and carries the water to the shed. Find the position on the river bank that will allow him to walk the shortest distance from house to river to shed. Further, describe how the farmer could solve the problem on the ground with the aid of a few stakes for sighting. 2. Lewis Carroll’s problem After a battle at least 95% of the combatants had lost a tooth, at least 90% had lost an eye, at least 80% had lost an arm, and at least 75% had lost a leg. At least how many had lost all four?

Primitive Functions This chapter uses differentiation to find the gradient of tangents and stationary points of functions.

Chapter 2 Geometrical Applications of Calculus

Sometimes you may know f l] x g and need to find the original function, ] g f x . This process is called anti-differentiation, and the original function is called the primitive function.

EXAMPLE Sketch the primitive function (the original function) given the derivative function below.

Solution Reversing what you would do to sketch the derivative function, the parts at the x-axis have a zero gradient, so show stationary points on the original function. There are stationary points at x1 and x2. The parts of the graph above the x-axis show a positive gradient, so the original function is increasing. This happens to the left of x1 and to the right of x2. The parts of the graph below the x-axis show a negative gradient, so the original function is decreasing. This happens between x1 and x2. Sketching this information gives:

CONTINUED

73

74

Maths In Focus Mathematics HSC Course

The only problem is, we do not have enough information to know where the curve turns around. There are many choices of where we could put the graph:

The primitive function gives a family of curves.

The examples below use what we know about differentiation to find out how to reverse this to find the primitive function.

EXAMPLES 1. Differentiate x2. Hence find a primitive function of 2x.

Solution The derivative of x2 is 2x ` a primitive function of 2x is x2 2. Differentiate x2 + 5. Hence find a primitive function of 2x.

Solution The derivative of x2 + 5 is 2x ` a primitive function of 2x is x2 + 5 3. Differentiate x 2 − 3. Hence find a primitive function of 2x.

Solution The derivative of x2 − 3 is 2x ` a primitive function of 2x is x2 − 3

Chapter 2 Geometrical Applications of Calculus

Thus 2x has many different primitive functions. In general, the primitive of 2x is x 2 + C, where C is a constant (real number).

EXAMPLES 1. Find the primitive of x.

Solution The derivative of x2 is 2x. The derivative of

x2 x2 is x. So the primitive of x is + C. 2 2

2. Find the primitive of x2.

Solution The derivative of x3 is 3x2. x3 The derivative of is x2. 3 x3 So the primitive of x2 is + C. 3 Continuing this pattern gives the general primitive function of x n.

dy

xn + 1 +C n+1 dx where C is a constant If

= xn then y =

Proof ] n + 1 g xn + 1 − 1 d xn + 1 c + Cm = n+1 dx n + 1 = xn

EXAMPLES dy

= 6x2 + 8x. If the curve passes dx through the point ^ 1, −3 h, find the equation of the curve. 1. The gradient of a curve is given by

Solution dy dx

= 6x 2 + 8x

x3 x2 m + 8c m + C 3 2 ` y = 2x 3 + 4x 2 + C y = 6c

CONTINUED

75

76

Maths In Focus Mathematics HSC Course

The curve passes through ^ 1, −3 h ` −3 = 2 ]1 g3 + 4 ]1 g2 + C =2+4+C −9 = C Equation is y = 2x 3 + 4x 2 − 9. 2. If f m] x g = 6x + 2 and f l] 1 g = f ] − 2 g = 0, find f ] 3 g .

Solution f m(x) = 6x + 2 x2 m + 2x + C 2 = 3x 2 + 2x + C

f l(x) = 6 c

Now f l(1) = 0 So `

0 = 3 ] 1 g 2 + 2 (1) + C −5 = C f l(x) = 3x2 + 2x − 5 x3 x2 m + 2 c m − 5x + C 3 2 = x 3 + x 2 − 5x + C

f (x) = 3 c Now f (− 2) = 0 So

`

0 = ] − 2 g 3 + ] − 2 g 2 − 5 (−2) + C = − 8 + 4 + 10 + C −6 = C f (x) = x 3 + x 2 − 5x − 6 f (3) = 3 3 + 3 2 − 5 (3) − 6 = 27 + 9 − 15 − 6 = 15

2.11 Exercises 1.

Find the primitive function of (a) 2x − 3 (b) x2 + 8x + 1 (c) x5 − 4x3 (d) ] x − 1 g 2 (e) 6

2.

Find f (x) if (a) f l(x) = 6x 2 − x (b) f l(x) = x4 − 3x2 + 7 (c) f l(x) = x − 2 (d) f l(x) = (x + 1) (x − 3) 1

(e) f l(x) = x 2

Chapter 2 Geometrical Applications of Calculus

3.

Express y in terms of x if dy = 5x 4 − 9 (a) dx (b)

dy dx

= x − 4 − 2x − 2

x3 = − x2 (c) dx 5

(e) 4.

dy dx dy dx

2 x2

(e) x If

dy dx

1 − 2 −7

= x3 −

+ 2x

2x +1 3

2 − 3

− 2x − 2

= x3 − 3x2 + 5 and y = 4 when

x = 1, find an equation for y in terms of x. 6.

If f l(x) = 4x − 7 and f (2) = 5, find f (x) .

7.

Given f l] x g = 3x 2 + 4x − 2 and f (− 3) = 4, find f ] 1 g.

8.

Given that the gradient of the tangent to a curve is given dy by = 2 − 6x and the curve dx passes through ^ − 2, 3 h, find the equation of the curve.

9.

If

dx ] = t − 3 g 2 and x = 7 when dt

t = 0, find x when t = 4. d2 y 2

= 8 and

dy

= 0 and dx dx y = 3 when x = 1, find the equation of y in terms of x.

10. Given

= 12x + 6 and

dy

13. Given f m(x) = 5x 4, f l(0) = 3 and f ] − 1 g = 1, find f ] 2 g .

Find the primitive function of (a) x (b) x − 3 1 (c) 8 x (d) x

5.

=

2

12. If f m(x) = 6x − 2 and f l] 2 g = f ] 2 g = 7, find the function f ] x g .

dy

(d)

d2 y

= 1 at the dx dx point ^ − 1, −2 h, find the equation of the curve.

11. If

d2 y

= 2x + 1 and there is a dx2 stationary point at (3, 2), find the equation of the curve.

14. If

d2 y

= 8x and the dx2 tangent at (−2, 5) makes an angle of 45° with the x-axis. Find the equation of the curve.

15. A curve has

16. The tangent to a curve with d2 y = 2x − 4 makes an angle dx2 of 135° with the x-axis in the positive direction at the point ^ 2, −4 h . Find its equation. 17. A function has a tangent parallel to the line 4x − y − 2 = 0 at the point ^ 0, −2 h and f m(x) = 12x 2 − 6x + 4. Find the equation of the function. d2 y

= 6 and the dx2 tangent at ^ −1, 3 h is perpendicular to the line 2x + 4y − 3 = 0. Find the equation of the curve.

18. A curve has

19. A function has f l(1) = 3 and f (1) = 5. Evaluate f (−2) given f m] x g = 6x + 18. d2 y

= 12x − 24 and dx2 a stationary point at ^ 1, 4 h . Evaluate y when x = 2.

20. A curve has

77

78

Maths In Focus Mathematics HSC Course

Test Yourself 2 1.

Find the stationary points on the curve y = x 3 + 6x 2 + 9x − 11 and determine their nature.

2.

Find all x-values for which the curve y = 2x 3 − 7x 2 − 3x + 1 is concave upwards.

3.

dy = 6x2 + 12x − 5. If the dx curve passes through the point (2, −3), find the equation of the curve. A curve has

4.

If f (x) = 3x 5 − 2x 4 + x 3 − 2, find (a) f (−1) (b) f l(−1) (c) f m] − 1 g

5.

The height in metres of an object thrown up into the air is given by h = 20t − 2t 2 where t is time in seconds. Find the maximum height that the object reaches.

6.

Find the stationary point on the curve y = 2x 2 and determine its nature.

7.

Find the domain over which the curve y = 5 − 6x − 3x 2 is decreasing.

8.

For the curve y = 3x 4 + 8x 3 − 48x 2 + 1 (a) find any stationary points and determine their nature (b) sketch the curve.

9.

Find the point of inflexion on the curve y = 2x 3 − 3x 2 + 3x − 2.

10. If f m(x) = 15x + 12, and f (2) = f l(2) = 5, find f (x) . 11. A soft drink manufacturer wants to minimise the amount of aluminium in its cans while still holding 375 mL of soft drink. Given that 375 mL has a volume of 375 cm3,

(a) show that the surface area of a can is 750 given by S = 2π r2 + r (b) find the radius of the can that gives the minimum surface area. 12. For the curve y = 3x 4 + 8x 3 + 6x 2, (a) find any stationary points (b) determine their nature (c) sketch the curve for − 3 ≤ x ≤ 3. 13. A rectangular prism with a square base is to have a surface area of 250 cm2. (a) Show that the volume is given by 125x − x3 V= . 2 (b) Find the dimensions that will give the maximum volume. 14. Find all x-values for which the curve y = 3x 2 − 18x + 4 is decreasing. d2 y

= 6x + 6, and there is a stationary dx2 point at (0, 3), find the equation of the curve.

15. If

16. The cost to a business of manufacturing x products a week is given by C = x 2 − 300x + 9000. Find the number of products that will give the minimum cost each week. 17. Show that y = − x 3 is monotonic decreasing for all x ≠ 0.

Chapter 2 Geometrical Applications of Calculus

18. Sketch a primitive function for each graph. (a)

(a) Show that the area of the garden is 1 A = x 25 − x2 . 2 (b) Find the greatest possible area of the garden bed. 20. For the curve y = x 3 + 3x 2 − 72x + 5, (a) find any stationary points on the curve and determine their nature (b) find any points of inflexion (c) sketch the curve.

(b)

21. Find the domain over which the curve y = x 3 − 3x 2 + 7x − 5 is concave downwards. 22. A function has f l(3) = 5 and f (3) = 2. If f m(x) = 12x − 6, find the equation of the function.

(c)

23. Find any points of inflexion on the curve y = x 4 − 6x 3 + 2x + 1 . 24. Find the maximum value of the curve y = x 3 + 3x 2 − 24x − 1 in the domain − 5 ≤ x ≤ 6. 25. A function has f l(2) < 0 and f m(2) < 0. Sketch the shape of the function near x = 2.

19. A 5 m length of timber is used to border a triangular garden bed, with the other sides of the garden against the house walls.

79

80

Maths In Focus Mathematics HSC Course

Challenge Exercise 2 1.

Find the first and second derivatives of 5−x . ] 4x2 + 1 g3

2.

Sketch the curve y = x ] x − 2 g3, showing any stationary points and inflexions.

3.

Find all values of x for which the curve y = 4x 3 − 21x 2 − 24x + 5 is increasing.

4.

Find the maximum possible area if a straight 8 m length of fencing is placed across a corner to enclose a triangular space.

10. Find the radius and height, correct to 2 decimal places, of a cylinder that holds 200 cm3, if its surface area is to be a minimum. 11. A curve passes through the point (0, −1) and the gradient at any point is given by ] x + 3 g ] x − 5 g. Find the equation of the curve. 12. The cost of running a car at an average v2 speed of ν km/h is given by c = 150 + 80 cents per hour. Find the average speed, to the nearest km/h, at which the cost of a 500 km trip is a minimum. 13. Find the equation of a curve that is always concave upwards with a stationary point at (−1, 2) and y-intercept 3.

5.

Find the greatest and least values of the function f (x) = 4x 3 − 3x 2 − 18x in the domain − 2 ≤ x ≤ 3.

6.

Show that the function f (x) = 2 (5x − 3)3 has a horizontal point of inflexion at ^ 0 .6 , 0 h .

7.

Two circles have radii r and s such that r + s = 25. Show that the sum of areas of the circles is least when r = s.

8.

The rate of change of V with respect to t is given by t=

9.

dV ] = 2t − 1 g 2 . If V = 5 when dt

1 , find V when t = 3. 2

(a) Show that the curve y = x − 1 has no stationary points. (b) Find the domain and range of the curve. (c) Hence sketch the curve.

14. Given f l(x) = x (x − 3) (x + 1) and f (0) = 0 (a) find the equation for f (x) (b) find any stationary points on the function (c) sketch the function. 15. The volume of air in a cubic room on a submarine is given by the formula V = − 4x 3 + 27x 2 − 24x + 2, where x is the side of the room in metres. Find the dimensions of the room that will give the maximum volume of air. 16. If f m(x) = x − 9 and f l(−1) = f (−2) = 7, find f (3) . 17. Given

dy dx

=

1 and y = 1 when x = 4, x − 5 h2 ^

find d2 y when x = 6 (a) dx2 (b) y when x = 6.

Chapter 2 Geometrical Applications of Calculus

18. Show that y = x n has a stationary point at (0, 0) where n is a positive integer. (a) If n is even, show that (0, 0) is a minimum turning point. (b) If n is odd, show that (0, 0) is a point of inflexion. 19. Find the maximum possible volume if a rectangular prism with a length twice its breadth has a surface area of 48 cm2. 20. (a) Find the stationary point on the curve y = x k + 1 where k is a positive integer.

(b) What values of k give a minimum turning point? 21. Find the minimum and maximum values x+3 of y = 2 in the domain − 2 ≤ x ≤ 2. x −9 22. The cost of running a car at an average V2 speed of V km/h is given by c = 100 + 75 cents per hour. Find the average speed (to the nearest km/h) at which the cost of a 1000 km trip is a minimum.

81

3

Integration TERMINOLOGY Definite integral: The integral or primitive function restricted to a lower and upper boundary. It has the notation

#a

b

f (x) dx and geometrically represents the area

between the curve y = f (x), the x-axis and the ordinates x = a and x = b Even function: A function where f (− x) = f (x). It is symmetrical about the y-axis

Indefinite integral: General primitive function represented by

# f (x) dx

Integration: The process of finding a primitive function Odd function: A function where f (− x) = − f (x). An odd function has rotational symmetry about the origin

Chapter 3 Integration

INTRODUCTION INTEGRATION IS THE PROCESS of finding an area under a curve. It is an important process in many areas of knowledge, such as surveying, physics and the social sciences. In this chapter, you will look at approximation methods of integrating, as well as shorter methods that lead to finding areas and volumes. You will learn how integration and differentiation are related.

DID YOU KNOW? Integration has been of interest to mathematicians since very early times. Archimedes (287–212 BC) found the area of enclosed curves by cutting them into very thin layers and finding their sum. He found the formula for the volume of a sphere this way. He also found an estimation of π, correct to 2 decimal places.

DID YOU KNOW? Box text... Area within curve

Archimedes

Approximation Methods Mathematicians used rectangles in order to find the approximate area between a curve and the x-axis.

83

84

Maths In Focus Mathematics HSC Course

NOTATION The area of each rectangle is f (x) δ x where f (x) is the height and δ x is the width of each rectangle. As δ x " 0, sum of rectangles " exact area. Area = lim Σ f (x) δ x

The symbol # comes from S, the sum of rectangles.

δx"0

= # f (x) dx Now there are other, more accurate ways to find the area under a curve. However, the notation is still used.

Trapezoidal rule The trapezoidal rule uses a trapezium for the approximate area under a curve. A trapezium generally gives a better approximation to the area than a rectangle.

#a

b

f (x) dx Z

1 (b − a) [ f (a) + f (b)] 2

Proof 1 h (a + b) 2 1 = (b − a) [ f (a) + f (b)] 2

A=

EXAMPLES 1. Find an approximation for

#1

Solution

#1 #a

b

4

4 1 dx x = #1 x dx

1 (b − a) [ f (a) + f (b)] 2 4 1 #1 dx x Z 2 (4 − 1) [ f (1) + f (4)]

f (x) dx Z

4

dx x using the trapezoidal rule.

Chapter 3 Integration

1 1 1 (3) c + m 2 1 4 3 5 = ´ 2 4 = 1.875 =

85

1 The function f (x) is x .

2. Find an approximation for 2 subintervals.

#0

1

x 3 dx using the trapezoidal rule with

Solution

Two subintervals mean 2 trapezia.

#a

b

f (x) dx Z

#0

1

x3 dx Z

1 (b − a) [ f (a) + f (b)] 2

#0

0.5

x3 dx + # x3 dx 1

0.5

1 1 (0.5 − 0) [ f (0) + f (0.5)] + (1 − 0.5) [ f (0.5) + f (1)] 2 2 1 1 = (0.5) (0 3 + 0.5 3) + (0.5) (0.5 3 + 1 3) 2 2 = 0.25 (0.125) + 0.25 (1.125) = 0.3125 =

There is a more general formula for n subintervals. Several trapezia give a more accurate area than one. However, you could use the first formula several times if you prefer using it.

The function is x3.

86

Maths In Focus Mathematics HSC Course

#a

b

f (x) dx Z where h =

h [(y + yn ) + 2 (y1 + y2 + y3 + . . . + yn – 1 )] 2 0 b−a n ^ width of each trapezium h

Proof

#a

b

f (x) dx Z

h h h h ( y + y 1 ) + ( y 1 + y 2 ) + ( y 2 + y 3 ) + . . . + (y n − 1 + y n ) 2 0 2 2 2

h (y + 2y 1 + 2y 2 + 2y 3 + . . . + 2y n − 1 + y n ) 2 0 h = [(y 0 + y n ) + 2 (y 1 + y 2 + y 3 + . . . + y n − 1 )] 2 =

EXAMPLES 1. Find an approximation for with 7 subintervals.

Solution

Seven subintervals mean 7 trapezia.

#0

14

(t 2 + 3) dt, using the trapezoidal rule

Chapter 3 Integration

87

h [(y + yn ) + 2 (y1 + y2 + y3 + . . . + yn − 1)] 2 0 14 #0 (t2 + 3) dt Z h2 [(y0 + y7) + 2 (y1 + y2 + y3 + y4 + y5 + y6 )]

#a

b

f (x) dx Z

14 − 0 =2 7 14 #0 (t2 + 3) dt Z 22 " [(02 + 3) + (142 + 3)] + 2 [(22 + 3) + (42 + 3) + (62 + 3) + (82 + 3) + (102 + 3) + (122 + 3)]} = 966

where

h=

2 dx, using the trapezoidal rule x −1 with 4 subintervals, correct to 3 decimal places. 2. Find an approximation for

#2

3

Solution

There are 4 trapezia.

#a

b

where

#2

3

h [(y + y 4 ) + 2 (y 1 + y 2 + y 3 )] 2 0 b−a h= n 3−2 = 4 = 0.25

f (x) dx Z

0.25 2 2 2 2 2 2 a

97

98

Maths In Focus Mathematics HSC Course

Let area ABCD be A(x) Let area ABGE be A (x + h) Then area DCGE is A (x + h) − A (x)

This is the formula for differentiation from first principles. You first saw this in Chapter 8 of The Preliminary Course book.

Area DCFE < area DCGE < area DHGE i.e. f (x) . h < A(x + h) − A(x) < f (x + h) . h A(x + h) − A(x) f (x) < < f (x + h) h A (x + h) − A (x) lim f (x) < lim < lim f (x + h) h "0 h "0 h "0 h f (x) < Al(x) < f (x) ` Al(x) = f (x) i.e. A(x) is a primitive function of f(x) A (x) = F(x) + C where F(x) is the primitive function of f(x) Now A(x) is the area between a and x ` A(a) = 0 Substitute in (1):

`

A(a) = F(a) + C 0 = F(a) + C − F(a) = C A(x) = F(x) − F(a) If x = b where b > a, A(b) = F(b) − F(a)

Definite Integrals By the fundamental theorem of calculus, b # f (x) dx = F(b) − F(a) where F(x) is the primitive function of f(x). a

The primitive function of x n is

xn + 1 + C. n+1

Putting these pieces of information together, we can find areas under simple curves.

(1)

Chapter 3 Integration

#a

b

99

xn + 1 b F n+1 a an + 1 bn + 1 = − n+1 n+1

xn = <

EXAMPLES Evaluate 1.

#3

4

(2x + 1) dx

Solution

#3

4

4

(2x + 1) dx = 8 x2 + x B 3 Constant C will cancel out. That is, (4 2 + 4 + C ) − (3 2 + 3 + C )

= ( 4 2 + 4 ) − ( 3 2 + 3) = 20 − 12 =8

= 20 − 12 = 8.

The graph shows the area that the definite integral calculates.

2.

#0

5

3x2 dx

Solution

#0

5

5

3x 2 dx = 8 x 3 B 0 = (5 3) − (0 3) = 125 CONTINUED

100

Maths In Focus Mathematics HSC Course

3.

#0

2

− 3x2 dx

Solution

#0

2

− 3x2 dx = : − x3 D

2 0

= (−23) − (−03) = −8

Can you see why there is a negative solution?

4.

#− 1 x3 dx 1

Solution 4 1

#− 1 x 3 dx = ; x4 E − 1 1

4 1 4 ( − 1) − 4 4 1 1 = − 4 4 =0

=

The definite integral gives an area of zero. Can you see why?

Chapter 3 Integration

101

3.3 Exercises Evaluate 4x dx

9.

#− 1 (3x2 − 2x) dx

18.

#3

(2x + 1) dx

10.

#1

19.

#− 2 4t3 dt

3x2 dx

11.

#− 1 x2 dx

20.

#2

4

x2 dx 3

(4t − 7) dt

12.

#− 2 (x3 + 1) dx

21.

#0

3

5x 4 x dx

5.

#− 1 (6y + 5) dy

13.

#− 2 x5 dx

22.

#− 2 x

6.

#0

3

6x2 dx

14.

#1

4

23.

#0

2

4x 3 + x 2 + 5 x dx x

7.

#1

2

(x2 + 1) dx

15.

#0

1

(x3 − 3x2 + 4x) dx

24.

#0

1

x 3 − 2x 2 + 3x dx x

8.

#0

2

4x3 dx

16.

#1

2

(2x − 1) 2 dx

25.

#3

4

x2 + x + 3 dx 3x5

17.

#− 1 (y3 + y) dy

1.

#0

2

2.

#1

3

3.

#0

5

4.

#1

2

1

4

3

(4x2 + 6x − 3) dx

1

3

2

x dx

4

(2 − x) 2 dx

2

1

4

− 3x dx x

1

Class Investigation Look at the results of definite integrals in the examples and exercises. Sketch the graphs where possible and shade in the areas found. • Can you see why the definite integral sometimes gives a negative answer? • Can you see why it will sometimes be zero?

Indefinite Integrals Sometimes it is necessary to find a general or indefinite integral (primitive function).

# f (x) dx = F (x) + C where F(x) is a primitive function of f (x) In Chapter 2 you learned the result for the primitive function of x n. This result is the same as the integral of x n.

n+1

# x n dx = nx + 1 + C

Simplify first, then find the definite integral.

102

Maths In Focus Mathematics HSC Course

EXAMPLES Find each indefinite integral (primitive function) 1.

# (x4 − 3x2 + 4x − 7) dx

Solution

# (x4 − 3x2 + 4x − 7) dx = x5

5

=

2.

−3:

x3 x2 +4: − 7x + C 3 2

x5 − x 3 + 2x 2 − 7 x + C 5

# 5x9 dx

Solution

# 5x9 dx = 5 c x10 m + C 10

# 5x9 dx

is the same as 5 # x9 dx.

=

3.

# b 2x

x10 +C 2

5

+ 7x2 − 3x l dx x

Solution

# b 2xx

4.

5

7x 2 3 x + x − x l dx = # (2x4 + 7x − 3) dx 2 x 5 7x 2 = + − 3x + C 5 2

# c x13 +

x m dx

Solution

# c x13 +

1

x m dx = # (x− 3 + x 2 ) dx 3

x− 2 x 2 = + +C −2 3 2 2 x3 1 =− 2 + +C 3 2x

Chapter 3 Integration

DID YOU KNOW? John Wallis (1616–1703) found that the area under the curve y = 1 + x + x2 + x3 + . . . is given by x+

x2 x3 x4 + + + .... 4 2 3

He found this result independently of the fundamental theorem of calculus.

3.4 Exercises Find each indefinite integral (primitive function) 1.

# x2 dx

19.

# (6x3 + 5x2 − 4) dx

2.

# 3x5 dx

20.

# (3x− 4 + x− 3 + 2x− 2) dx

3.

# 2x4 dx

21.

4.

# (m + 1) dm

# dx x8

22.

# x 3 dx

5.

# (t2 − 7) dt

23.

#x

6.

# (h2 + 5) dh

24.

# (1 − 2x)2 dx

7.

# (y − 3) dy

25.

# (x − 2) (x + 5) dx

8.

# (2x + 4) dx

26.

9.

# (b2 + b) db

# x32 dx

10.

# (a3 − a − 1) da

27.

# dx x3 # 4x

1

6

− 3 x 5 + 2x 4 dx x3

3

− x 5 − 3x 2 + 7 dx x5

11.

# (x2 + 2x + 5) dx

28.

12.

# (4x3 − 3x2 + 8x − 1) dx

29.

# (y2 − y −7 + 5) dy

13.

# (6x5 + x4 + 2x3) dx

30.

# (t2 − 4) (t − 1) dt

14.

# (x7 − 3x6 − 9) dx

31.

#

15.

# (2x3 + x2 − x − 2) dx

32.

# t25 dt

16.

# (x5 + x3 + 4) dx

33.

#3

17.

# (4x2 − 5x − 8) dx

34.

#x

18.

# (3x4 − 2x3 + x) dx

35.

#

x dx

x dx x dx x e1 +

1 o dx x

103

104

Maths In Focus Mathematics HSC Course

Function of a function rule EXAMPLE Differentiate (2x + 3)5 . Hence find Find

# 10 (2x + 3) 4 dx.

# (2x + 3) 4 dx.

Solution d ] 2x + 3 g5 = 5 ] 2x + 3 g4 $ 2 dx = 10 ] 2x + 3 g4

Remember: Integration is the reverse of differentiation.

# 10 ] 2x + 3 g4 dx = ] 2x + 3 g5 + C 1 # ] 2x + 3 g4 dx = 10 # 10 ] 2x + 3 g4 dx

`

=

1 ] 2x + 3 g5 + C 10

# (ax + b) dx = n

(ax + b) n + 1 +C a ( n + 1)

Proof d ] ax + b gn + 1 = ] n + 1 g ] ax + b gn $ a dx = a ] n + 1 g ] ax + b gn `

# a ] n + 1 g ] ax + b gn dx = ] ax + b gn + 1 + C # ] ax + b gn dx = ] 1 g # a ] n + 1 g ] ax + b gn dx a n+1 1 ] ax + b gn + 1 + C = ] a n + 1g ] ax + b gn + 1 +C = a ]n + 1g

Chapter 3 Integration

EXAMPLES Find 1. # (5x − 9)3 dx

Solution

# (5x − 9)3

(5x − 9)4 +C 5´4 4 (5x − 9) = +C 20

dx =

# (3 − x)8 dx

2.

Solution (3 − x)9 +C −1 ´ 9 (3 − x)9 = − +C 9

# (3 − x)8 dx =

3.

#

4x + 3 dx

Solution 3

(4x + 3) 2 +C # (4x + 3) dx = 3 4´ 2 (4x + 3) 3 = +C 6 1 2

3.5 Exercises Find each indefinite integral (primitive function) 1.

(a)

# (3x − 4)2 dx by (i) expanding (ii) the function of a function rule

(b) (c) (d) (e)

# (x + 1)4 dx # (5x − 1)9 dx # (3y − 2)7 dy # (4 + 3x)4 dx

(f) (g) (h) (i) (j)

# (7x + 8)12 dx # (1 − x)6 dx # 2x − 5 dx # 2 (3x + 1)− 4 dx # 3 (x + 7)− 2 dx

(k)

# 2 (4x1− 5) 3 dx

(l)

#3

4x + 3 dx

105

106

Maths In Focus Mathematics HSC Course

1

(m) # (2 − x) − 2 dx

2.

(n)

#

(t + 3)3 dt

(o)

#

(5x + 2)5 dx

Evaluate (a)

#1

(b)

#0

(c)

#1

2

(d)

#0

2

2

1

(2x + 1)4 dx (3y − 2)3 dy

(e)

#0

(3x − 1) 2 dx 6

(f)

#4

(5 − x)6 dx

(g)

#3

(h)

#0

(i)

#0

2

(j)

#1

4

(1 − x)7 dx (3 − 2x)5 dx

1

5

6

1

x − 2 dx dx (3x − 2) 4 5 dn (2n + 1) 3 2 dx (5x − 4)3

Class Exercise Differentiate (x + 1) 2x − 3 . Hence find

#

3x − 2 dx. 2x − 3

Areas Enclosed by the x-axis The definite integral gives the signed area under a curve. Areas above the x-axis give a positive definite integral.

Chapter 3 Integration

Areas below the x-axis give a negative definite integral.

We normally think of areas as positive. So to find areas below the x-axis, take the absolute value of the definite integral. That is, Area =

#a

b

f (x) dx

Always sketch the area to be found.

EXAMPLES 1. Find the area enclosed by the curve y = 2 + x − x2 and the x-axis.

Solution Sketch y = 2 + x − x 2 . The integral will be positive as the area is above the x-axis.

Area = # (2 + x − x2) dx 2

−1

x2 x3 2 − F 2 3 −1 2 (−1) 2 (−1) 3 2 23 n = c 2 ( 2) + − m − d 2 (−1) + − 2 3 2 3 8 1 1 = c 4 + 2 − m − c −2 + + m 3 2 3 1 =4 2 So the area is 4 1 units2. = < 2x +

2

CONTINUED

107

108

Maths In Focus Mathematics HSC Course

2. Find the area bounded by the curve y = x 2 − 4 and the x-axis.

Solution The integral will be negative as the area is below the x-axis.

Sketch y = x 2 − 4.

#− 2 (x2 − 4)dx = < x3

3

2

− 4x F

2 −2

(−2)3 2 − 4 ( 2) m − d − 4 (−2) n 3 3 8 −8 = c − 8m − c + 8m 3 3 2 = − 10 3 2 So the area is 10 units2. =c

3

3

3. Find the area enclosed between the curve y = x 3, the x-axis and the lines x = − 1 and x = 3.

Solution Sketch y = x 3.

Some of the area is below the x-axis, so its integral will be negative.

Chapter 3 Integration

We need to find the sum of the areas between x = − 1 and 0, and x = 0 and 3. 4 0

#− 1 x 3 dx = ; x4 E − 1 0

4 0 4 (−1) − 4 4 1 =− 4 1 So A 1 = units2. 4

=

#0

3

x4 3 E 4 0 34 04 = − 4 4 81 = 4

x3 dx = ;

81 units 2 . 4 1 81 A1 + A2 = + 4 4 1 = 20 units 2 2

So

A2 =

Even and odd functions Some functions have special properties.

For odd functions,

#− a f (x) dx = 0 a

For even functions,

#− a f (x) dx = 2 #0 a

a

f (x) dx

109

110

Maths In Focus Mathematics HSC Course

EXAMPLES 1. Find the area between the curve y = x 3, the x-axis and the lines x = − 2 and x = 2.

Solution

#− 2 x 3 dx = 0 2

y = x 3 is an odd function since f (− x) = − f (x) .

Area = 2 # x 3 dx 2

0

x4 2 E 4 0 24 04 = 2c − m 4 4 =8 So the area is 8 units2. = 2;

2. Find the area between the curve y = x 2, the x-axis and the lines x = − 4 and x = 4.

Solution y = x 2 is an even function since f (− x) = f (x) .

#− 4 x 2 dx = 2 #0 4

4

x 2 dx

x3 4 F 3 0 43 03 m = 2c − 3 3 2 = 42 3 = 2<

So the area is 42 2 units2. 3

3.6 Exercises 1.

Find the area enclosed between the curve y = 1 − x 2 and the x-axis.

2.

3.

4.

5.

Find the area bounded by the curve y = x 2 − 9 and the x-axis.

Find the area bounded by the curve y = − x 2 + 9x − 20 and the x-axis.

6.

Find the area enclosed between the curve y = x 2 + 5x + 4 and the x-axis.

Find the area enclosed between the curve y = − 2x 2 − 5x + 3 and the x-axis.

7.

Find the area enclosed between the curve y = x 3, the x-axis and the lines x = 0 and x = 2.

Find the area enclosed between the curve y = x 2 − 2x − 3 and the x-axis.

Chapter 3 Integration

8.

Find the area enclosed between the curve y = x 4, the x-axis and the lines x = − 1 and x = 1.

9.

Find the area enclosed between the curve y = x 3, the x-axis and the lines x = − 2 and x = 2.

10. Find the area enclosed between the curve y = x 3, the x-axis and the lines x = − 3 and x = 2. 11. Find the area bounded between the curve y = 3x 2, the x-axis and the lines x = − 1 and x = 1. 12. Find the area enclosed between the curve y = x 2 + 1, the x-axis and the lines x = − 2 and x = 2. 13. Find the area enclosed between the curve y = x 2, the x-axis and the lines x = − 3 and x = 2. 14. Find the area enclosed between the curve y = x 2 + x, and the x-axis.

15. Find the area enclosed between 1 the curve y = 2 , the x-axis and x the lines x = 1 and x = 3. 16. Find the area enclosed between 2 the curve y = , the x-axis (x − 3) 2 and the lines x = 0 and x = 1. 17. Find the area bounded by the curve y = x , the x-axis and the line x = 4. 18. Find the area bounded by the curve y = x + 2 , the x-axis and the line x = 7. 19. Find the area bounded by the curve y = 4 − x 2 , the x-axis and the y-axis in the first quadrant. 20. Find the area bounded by the x-axis, the curve y = x 3 and the lines x = − a and x = a.

Areas Enclosed by the y-axis

To find the area between a curve and the y-axis, we change the subject of the equation of the curve to x. That is, x = f (y) . The definite integral is given by

#a

b

f (y) dy or # x dy b

a

Since x is positive on the right-hand side of the y-axis, the definite integral is positive. Since x is negative on the left-hand side of the y-axis, the definite integral is negative.

111

112

Maths In Focus Mathematics HSC Course

EXAMPLES 1. Find the area enclosed by the curve x = y2, the y-axis and the lines y = 1 and y = 3.

Solution Area = # y 2 dy 3

1

y3 3 G 3 1 33 13 = − 3 3 2 = 8 units 2 3 ==

The integral is positive since the area is to the right of the y-axis.

2. Find the area enclosed by the curve y = x2, the y-axis and the lines y = 0 and y = 4 in the first quadrant.

Solution

Change the subject of the equation to x. y = x2 ` ± y =x In the first quadrant, y =x 1

i.e. y 2 = x 1

Area = # y 2 dy 0 R 3 V4 S y2 W =S W SS 3 WW T 2 X0 4

4

2 y3 H 3 0 2 43 2 03 = − 3 3 1 = 5 units 2 3 =>

Chapter 3 Integration

3. Find the area enclosed between the curve y = the lines y = 0 and y = 3.

113

x + 1 , the y-axis and

Solution

Some of the area is on the left of the y-axis.

y= x+1 y2 = x + 1 y2 − 1 = x Area =

#0

1

^ y 2 − 1 h dy + # ^ y 2 − 1 h dy 3

1

1 3 y3 y3 = = − yG + = − yG 3 3 0 1 33 03 13 13 = c − 1m − c − 0m + c − 3m − c − 1m 3 3 3 3 2 2 = − +6 + 3 3 2 2 = +6 3 3 1 = 7 units 2 3

3.7 Exercises 1.

Find the area bounded by the y-axis, the curve x = y2 and the lines y = 0 and y = 4.

4.

Find the area between the lines y = x − 1, y = 0 and y = 1 and the y-axis.

2.

Find the area enclosed between the curve x = y3, the y-axis and the lines y = 1 and y = 3.

5.

Find the area bounded by the line y = 2x + 1, the y-axis and the lines y = 3 and y = 4.

3.

Find the area enclosed between the curve y = x2, the y-axis and the lines y = 1 and y = 4 in the first quadrant.

6.

Find the area bounded by the curve y = x , the y-axis and the lines y = 1 and y = 2.

114

Maths In Focus Mathematics HSC Course

7.

Find the area bounded by the curve x = y 2 - 2y - 3 and the y-axis.

8.

Find the area bounded by the curve x = -y 2 - 5y - 6 and the y-axis.

9.

Find the area enclosed by the curve y = 3x − 5 , the y-axis and the lines y = 2 and y = 3.

10. Find the area enclosed between 1 the curve y = 2 , the y-axis and x the lines y = 1 and y = 4 in the first quadrant.

12. Find the area enclosed between the curve y = x3 - 2 and the y-axis between y = -1 and y = 25. 13. Find the area enclosed between the lines y = 4 and y = 1 - x in the second quadrant. 14. Find the area enclosed between the y-axis and the curve x = y(y - 2). 15. Find the area bounded by the curve y = x4 + 1, the y-axis and the lines y = 1 and y = 3 in the first quadrant, correct to 2 significant figures.

11. Find the area enclosed between the curve y = x3, the y-axis and the lines y = 1 and y = 8.

Sums and Differences of Areas EXAMPLES 1. Find the area enclosed between the curve y = x2, the y-axis and the lines y = 0 and y = 4 in the first quadrant.

Solution A = Area of rectangle − # x 2 dx 2

0

x3 2 F 3 0 8 0 = 8 −c − m 3 3 1 = 5 units 2 3 = (4 ´ 2) − <

This example was done before by finding the area between the curve and the y-axis.

Chapter 3 Integration

2. Find the area enclosed between the curves y = x2, y = (x − 4)2 and the x-axis.

Solution Solve simultaneous equations to find the intersection of the curves. y = x2 y = ]x − 4 g2 x2 = ] x − 4 g 2 = x 2 − 8x + 16 0 = − 8x + 16 8x = 16 x=2

`

(1) (2 )

Area = # x2 dx + # ] x − 4 g2 dx 2

0

4

2

4 x3 2 < ] x − 4 g 3 F F + 3 0 3 2 8 0 0 −8 m = c − m+c − 3 3 3 3 1 = 5 units2 3

=<

3. Find the area enclosed between the curve y = x2 and the line y = x + 2.

Solution Solve simultaneous equations to find the intersection of the curve and line. y = x2 y=x+2 `

(1) (2)

x2 = x + 2 x2 − x − 2 = 0 (x − 2 ) (x + 1 ) = 0 x = 2 or − 1

Area = # ] x + 2 g dx − # x 2 dx 2

2

−1 2

−1

= # ] x + 2 − x 2 g dx −1

=<

x2 x3 2 + 2x − F 2 3 −1

(−1)2 (−1)3 22 23 n + 2 (2) − m − d + 2 (−1) − 2 3 2 3 8 1 1 = c2 + 4 − m − c − 2 + m 3 2 3 1 = 4 units2 2

=c

115

116

Maths In Focus Mathematics HSC Course

3.8 Exercises 9.

Find the area enclosed between the curve y = x3, the x-axis and the line y = -3x + 4.

1.

Find the area bounded by the line y = 1 and the curve y = x2.

2.

Find the area enclosed between the line y = 2 and the curve y = x2 + 1.

10. Find the area enclosed by the curves y = (x - 2)2 and y = (x - 4)2.

3.

Find the area enclosed by the curve y = x2 and the line y = x.

11. Find the area enclosed between the curves y = x2 and y = x3.

4.

Find the area bounded by the curve y = 9 - x2 and the line y = 5.

12. Find the area enclosed by the curves y = x2 and x = y2.

5.

Find the area enclosed between the curve y = x2 and the line y = x + 6.

13. Find the area bounded by the curve y = x2 + 2x - 8 and the line y = 2x + 1.

6.

Find the area bounded by the curve y = x3 and the line y = 4x.

14. Find the area bounded by the curves y = 1 - x2 and y = x2 - 1.

7.

Find the area enclosed between the curves y = (x - 1)2 and y = (x + 1)2.

15. Find the exact area enclosed between the curve y = 4 − x 2 and the line x - y + 2 = 0.

8.

Find the area enclosed between the curve y = x2 and the line y = -6x + 16.

Volumes The volume of a solid can be found by rotating an area under a curve about the x-axis or the y-axis.

Chapter 3 Integration

117

Volumes about the x-axis

V = π # y 2 dx b

a

Proof

Take a disc of the solid with width δ x.

The disc is approximately a cylinder with radius y and height δ x.

Volume = π r2 h = π y 2 δx Taking the sum of an infinite number of these discs, Volume = lim π y2 δx δx " 0 b

= # π y2 dx a

= π # y2 dx b

a

118

Maths In Focus Mathematics HSC Course

EXAMPLES 1. Find the volume of the solid of revolution formed when the curve x2 + y2 = 9 is rotated about the x-axis between x = 1 and x = 3.

Solution x2 + y2 = 9 ` y2 = 9 − x2 V = π # y2 dx b

a

= π # ^ 9 − x2 h dx 3

1

3

x3 = π < 9x − F 3 1 1 = π c (27 − 9) − (9 − ) m 3 28π 3 = units 3 2. Find the volume of the solid formed when the line y = 3x - 2 is rotated about the x-axis from x = 1 to x = 2.

Solution y = 3x − 2 y 2 = ] 3x − 2 g 2

Use the function of a function rule

V=π

#a

b



#1

2

y2 dx ] 3x − 2 g2 dx 2

] 3x − 2 g3 F 3´3 1 43 13 n =π d − 9 9 63π = 9 = 7π units3

=π <

Chapter 3 Integration

119

PROBLEM What is wrong with the working out of this volume? Find the volume of the solid of revolution formed when the curve y = x 2 + 1 is rotated about the x-axis between x = 0 and x = 2.

Solution y = x2 + 1 `

y2 = ^ x2 + 1 h

2

V = π # y2 dx b

a

2 = π # ^ x2 + 1 h dx 2

0

3 2

^ x2 + 1 h G =π= 3 0 ^ 22 + 1 h 3 ^ 02 + 1 h 3 o =πe − 3 3 53 13 n =πd − 3 3 124π = units3 3

What is the correct volume?

Volumes about the y-axis The formula for rotations about the y-axis is similar to the formula for the x-axis.

The proof of this result is similar to that for volumes about the x-axis.

V = π # x2 dy b

a

120

Maths In Focus Mathematics HSC Course

EXAMPLE Find the volume of the solid formed when the curve y = x2 - 1 is rotated about the y-axis from y = -1 to y = 3.

Solution `

y = x2 − 1 y + 1 = x2 V=π =π

#a

b

x2 dy

#− 1 ^ y + 1 h dy 3

3

y2 + yG 2 −1 9 1 = π < c + 3 m − c − 1 mF 2 2 = 8π units3 =π=

3.9 Exercises 1.

Find the volume of the solid of revolution formed when the curve y = x2 is rotated about the x-axis from x = 0 to x = 3.

3.

Find the volume of the solid of revolution that is formed when the curve y = x2 + 2 is rotated about the x-axis from x = 0 to x = 2.

2.

Find the volume of the solid formed when the line y = x + 1 is rotated about the x-axis between x = 2 and x = 7.

4.

Determine the volume of the solid formed when the curve y = x3 is rotated about the x-axis from x = 0 to x = 1.

Chapter 3 Integration

5.

Find the volume of the solid formed when the curve x = y2 - 5 is rotated about the x-axis between x = 0 and x = 3.

15. Find the volume of the solid formed when the curve y = x + 3 is rotated about the x-axis from x = 1 to x = 6.

6.

Find the volume of the solid of revolution formed by rotating the line y = 4x - 1 about the x-axis from x = 2 to x = 4.

16. Find the volume of the solid formed when the curve y = x is rotated about the y-axis between y = 1 and y = 4.

7.

Find the volume of the hemisphere formed when the curve x2 + y2 = 1 is rotated about the x-axis between x = 0 and x = 1.

17. Find the volume of the solid formed when the curve y = 4 − x 2 is rotated about the y-axis from y = 1 to y = 2.

8.

The parabola y = (x + 2) is rotated about the x-axis from x = 0 to x = 2. Find the volume of the solid formed.

9.

Find the volume of the spherical cap formed when the curve x2 + y2 = 4 is rotated about the y-axis from y = 1 to y = 2.

2

10. Find the volume of the paraboloid formed when y = x2 is rotated about the y-axis from y = 0 to y = 3.

18. Find the volume of the solid formed when the line x + 3y - 1 = 0 is rotated about the y-axis from y = 1 to y = 2. 19. Find the volume of the solid formed when the line x + 3y - 1 = 0 is rotated about the x-axis from x = 0 to x = 8. 20. The curve y = x3 is rotated about the y-axis from y = 0 to y = 1. Find the volume of the solid formed.

11. Find the volume of the solid formed when y = x2 - 2 is rotated about the y-axis from y = 1 to y = 4.

21. Find the volume of the solid of revolution formed if the area enclosed between the curves y = x2 and y = (x - 2)2 is rotated about the x-axis.

12. The line y = x + 2 is rotated about the y-axis from y = -2 to y = 2. Find the volume of the solid formed.

22. The area bounded by the curve y = x2 and the line y = x + 2 is rotated about the x-axis. Find the exact volume of the solid formed.

13. Determine the volume of the solid formed when the curve y = x3 is rotated about the x-axis from x = -1 to x = 4.

23. Show that the volume of a sphere

14. Find the volume of the solid formed when the curve y = x is rotated about the x-axis between x = 0 and x = 5.

is given by the formula 4 V = π r3 by rotating the 3 semicircle y = r 2 − x 2 about the x-axis.

121

122

Maths In Focus Mathematics HSC Course

Application The volume of water in a small lake is to be measured by finding the depth of water at regular intervals across the lake.

What could the volume of this lake be?

The table below shows the results of the measurements, where x stands for the regular intervals chosen, in metres, and y is the depth in metres. x (m)

0

50

100

150

200

250

300

y (m)

10.2

39.1

56.9

43.2

28.5

15.7

9.8

Using Simpson’s rule to find an approximation for the volume of the lake, correct to 2 significant figures:

# = π#

V=π

a

0

#

a

b

y2 dx

300

y 2 dx

h 8 _ y0 + y6 i + 4 _ y1 + y3 + y5 i + 2 _ y2 + y4 i B 3 b−a h= 6 300 − 0 = 6 = 50

f ] x g dx Z

where

b

Chapter 3 Integration

#

0

300

y 2 dx Z

50 6 ] 10.22 + 9.82 g + 4 ] 39.12 + 43.22 + 15.72 g + 2 ] 56.92 + 28.52 g @ 3

= 381 099.3 `

V=π

#

0

300

y 2 dx

= π ´ 381 099.3 Z 1197 258.866 = 1 200 000

correct to 2 significant figures

So the volume of the lake is approximately 1 200 000 m3 of water.

123

124

Maths In Focus Mathematics HSC Course

Test Yourself 3 1.

(a) Use the trapezoidal rule with 2 subintervals to find an approximation 2 dx to # . 1 x2 (b) Use integration to find the exact 2 dx value of # . 1 x2

2.

Find the indefinite integral (primitive function) of (a) 3x + 1 5x 2 − x (b) x (c)

(b)

x

(d) ] 2x + 5 g7 3.

In an experiment, the velocity of a particle over time was found and the results placed in the table below. t

0

v

1

2

3

4

0

5.

Find the area bounded by the curve y = x 2, the x-axis and the lines x = − 1 and x = 2.

7.

Use Simpson’s rule with 5 function values to find the approximate area 1 enclosed between the curve y = x , the x-axis and the lines x = 1 and x = 3.

8.

Find the area enclosed between the curves y = x 2 and y = 2 − x 2 .

9.

The line y = 2x − 3 is rotated about the x-axis from x = 0 to x = 3. Find the volume of the solid of revolution.

6

1.3 1.7 2.1 3.5 2.8 2.6 2.2

Use Simpson’s rule to find the 6 approximate value of # f ] t g dt 4.

5

6.

Evaluate (a)

#0

(b)

#− 1 x5 dx

(c)

#0

2

] x3 − 1 g dx

1

1

] 3x − 1 g4 dx

Draw a primitive function of the following curves. (a)

#1

3x 4 − 2x 3 + x 2 − 1 dx. x2 11. Find the area bounded by the curve y = x 3, the y-axis and the lines y = 0 and y = 1. 10. Evaluate

2

12. Find the indefinite integral (primitive function) of ] 7x + 3 g11 . 13. Find the area bounded by the curve y = x 2 − x − 2, the x-axis and the lines x = 1 and x = 3. 14. Find the volume of the solid formed if the curve y = x 2 + 1 is rotated about the (a) x-axis from x = 0 to x = 2 (b) y-axis from y = 1 to y = 2

Chapter 3 Integration

15. (a) Change the subject of y = ] x + 3 g2 to x. (b) Find the area bounded by the curve y = ] x + 3 g2, the y-axis and the lines y = 9 and y = 16 in the first quadrant. (c) Find the volume of the solid formed if this area is rotated about the y-axis. 16. Evaluate

#0

4

18. Find the volume of the solid formed if the curve y = x 3 is rotated about the y-axis from y = 0 to y = 1. 19. Find

] 3t − 6t + 5 g dt . 2

(a)

# 5 ] 2x − 1 g4 dx

(b)

# 34x

5

dx

17. Find the area bounded by the curve y = x2 + 2x − 15 and the x-axis.

Challenge Exercise 3 1.

(a) Find the area enclosed between the curves y = x 3 and y = x 2 . (b) Find the volume of the solid formed if this area is rotated about the x-axis.

2.

(a) Show that f ] x g = x 3 + x is an odd function. 2 (b) Hence find the value of # f ] x g dx. −2 (c) Find the total area between f ] x g, the x-axis and the lines x = − 2 and x = 2.

3.

The curve y = 2 x is rotated about the x-axis between x = 1 and x = 2. Use Simpson’s rule with 3 function values to find an approximation of the volume of the solid formed, correct to 3 significant figures.

4.

Find the area enclosed between the curves y = ] x − 1 g2 and y = 5 − x 2 .

5.

(a) Differentiate ] x 4 − 1 g9 . (b) Hence find

6.

# x 3 ] x 4 − 1 g8 dx. #0

1

x ^ 3x 2 − 4 h 2

Find the area enclosed between the 3 curve y = , the y-axis and the lines x−2 y = 1 and y = 3, using the trapezoidal rule with 4 subintervals.

8.

Find the exact volume of the solid 1 formed by rotating the curve y = x about the x-axis between x = 1 and x = 3.

9.

Show that the area enclosed between the 1 curve y = x , the x-axis and the lines x = 0 and x = 1 is infinite by using Simpson’s rule with 3 function values.

10. (a) Sketch the curve y = x ] x − 1 g ] x + 2 g . (b) Find the total area enclosed between the curve and the x-axis. 11. Find the exact area bounded by the parabola y = x 2 and the line y = 4 − x. 12. Find the volume of the solid formed when the curve y = ] x + 5 g2 is rotated about the y-axis from y = 1 to y = 4.

2x 2 + 1 (a) Differentiate . 3x 2 − 4 (b) Hence evaluate

7.

dx.

125

126

Maths In Focus Mathematics HSC Course

13. (a) Find the derivative of x x + 3 . x+2 (b) Hence find # dx. x+3 14. (a) Evaluate # ] x2 − x + 1 g dx. 1 (b) Use Simpson’s rule with 3 function 3

values to evaluate

#1

3

] x2 − x + 1 g dx and

show that this gives the exact value. 15. Find the area enclosed between the curves y = x and y = x 3 .

16. For the shaded region below, find (a) the area (b) the volume when this area is rotated about the y-axis.

4

Exponential and Logarithmic Functions TERMINOLOGY Exponential equation: Equation where the pronumeral is the index or exponent such as 3 x = 9 Exponential function: A function in the form y = a x where the variable x is a power or exponent

Logarithm: A logarithm is an index. The logarithm is the power or exponent of a number to a certain base i.e. 2 x = 8 is the same as log2 8 = x

Chapter 4 Exponential and Logarithmic Functions

INTRODUCTION THIS CHAPTER INTRODUCES A new irrational number, ‘e’, that has

special properties in calculus. You will learn how to differentiate and integrate the exponential function f (x) = e x. The definition and laws of logarithms are also introduced in this chapter, as well as differentiation and integration involving logarithms.

DID YOU KNOW? John Napier (1550–1617), a Scottish theologian and an amateur mathematician, was the first to invent logarithms. These ‘natural’, or ‘Naperian’, logarithms were based on ‘e’. Napier was also one of the first mathematicians to use decimals rather than fractions. He invented the notation of the decimal, using either a comma or a point. The point was used in England, but a few European countries still use a comma. Henry Briggs (1561–1630), an Englishman who was a professor at Oxford, decided that logarithms would be more useful if they were based on 10 (our decimal system). These are called common logarithms. Briggs painstakingly produced a table of logarithms correct to 14 decimal places. He also produced sine tables—to 15 decimal places—and tangent tables—to 10 decimal places. The work on logarithms was greatly appreciated by Kepler, Galileo and other astronomers at the time, since they allowed the computation of very large numbers.

Differentiation of Exponential Functions When differentiating exponential functions f (x) = a x from first principles, an interesting result can be seen. The derivative of any exponential function gives a constant which is multiplied by the original function.

EXAMPLE Sketch the derivative (gradient) function of y = 10 x.

Solution

CONTINUED

133

134

Maths In Focus Mathematics HSC Course

The graph of y = 10 x always has a positive gradient that is becoming steeper. So the derivative function will always be positive, becoming steeper.

The derivative function of an exponential function will always have a shape similar to the original function. We can use differentiation from first principles to find how close this derivative function is to the original function.

EXAMPLE Differentiate f (x) = 10x from first principles.

Solution f l(x) = lim

f (x + h ) − f ( x )

h 10 x + h − 10 x = lim h "0 h 10 x (10 h − 1) = lim h "0 h 10 h − 1 x = 10 lim h "0 h h "0

You can explore limits using a graphics package on a computer or a graphical calculator.

Using the 10 x key on the calculator, and finding values of is small, gives the result:

10 h − 1 when h h

f l(x) Z 2.3026 ´ 10x or

d (10x ) Z 2.3026 ´ 10x dx

Drawing the graphs of y = 2.3026 ´ 10 x and y = 10 x together shows how close the derivative function is to the original graph. y y = 10 x y = 2.3026 × 10 x

12 10 8 6 4 2 −2

−1

1

2

x

Chapter 4 Exponential and Logarithmic Functions

135

Similar results occur for other exponential functions. In general, d x (a ) = kax where k is a constant. dx

Application If y = a x then

dy dx

You will study exponential growth and decay in Chapter 6.

= ka x = ky

This means that the rate of change of y is proportional to y itself. That is, if y is small, its rate of change is small, but if y is large, then it is changing rapidly. This is called exponential growth (or decay, if k is negative) and has many applications in areas such as population growth, radioactive decay, the cooling of objects, the spread of infectious diseases and the growth of technology.

Different exponential functions have different values of k.

EXAMPLES 1.

d x (2 ) Z 0.6931 ´ 2 x. dx y y = 0.6931 × 2x 12

y = 2x

10 8 6 4 2 −3 −2 −1

2.

1

2

x

3

d x (3 ) Z 1.0986 ´ 3 x. dx y

12 10

y = 3x y = 1.0986 × 3x

8 6 4 2 −3 −2 −1

1 2 3

x

136

Maths In Focus Mathematics HSC Course

Notice that the derivative function of y = 3 x is very close to the original function. We can find a number close to 3 that gives exactly the same graph for the derivative function. This number is approximately 2.71828, and is called e. d x (e ) = e x dx

e is an irrational number like π.

DID YOU KNOW? A transcendental number is a number beyond ordinary numbers. Another transcendental number is π.

The number e was linked to logarithms before this useful result in calculus was known. It is a transcendental (irrational) number. This was proven by a French mathematician, Hermite, in 1873. Leonhard Euler (1707–83) gave e its symbol, and he gave an approximation of e to 23 decimal places. Currently, e is known to about 100 000 decimal places. Euler studied mathematics, theology, medicine, astronomy, physics and oriental languages. He did extensive research into mathematics and wrote more than 500 books and papers. Euler gave mathematics much of its important notation. He caused π to become standard notation and used i for the square root of –1. He first used small letters to show the sides of triangles and the corresponding capital letters for their opposite angles. Also, he introduced the symbol S for sums and f(x) notation.

ex

KEY

Use this key to find powers of e. For example, to find e2: Press SHIFT e x 2 = e 2 7.389056099 To find e: Press SHIFT e x 1 = e 1 2.718281828

EXAMPLES 1. Sketch the curve y = e x.

Solution Use ex on your calculator to draw up a table of values: x

−3

−2

−1

0

1

2

3

y

0.05

0.1

0.4

1

2.7

7.4

20.1

Chapter 4 Exponential and Logarithmic Functions

137

2. Differentiate 5e x.

Solution d x (e ) = e x dx d d ` (5e x) = 5 (e x) dx dx = 5e x 3. Find the equation of the tangent to the curve y = 3e x at the point (0, 3).

Solution dy dx

= 3e x

At (0, 3), `

dy dx

= 3e 0 dy

=3 m=3

dx

Equation y − y1 = m (x − x1) y − 3 = 3 (x − 0 ) = 3x y = 3x + 3

CONTINUED

gives the gradient of the tangent.

138

Maths In Focus Mathematics HSC Course

4. Differentiate

2x + 3 . ex

Solution

This is the quotient rule from Chapter 8 of the Preliminary Course book.

dy dx

= =

u lv − v lu v2 2 . e x − e x ( 2 x + 3)

( e x) 2 2e x − 2xe x − 3e x = e 2x − e x − 2xe x = e 2x − e x (1 + 2x) = e 2x − (1 + 2x) = ex

4.1 Exercises 1.

2.

3.

Find, correct to 2 decimal places, the value of (a) e 1.5 (b) e − 2 (c) 2e 0.3 1 (d) 3 e (e) − 3e − 3.1 Sketch the curve (a) y = 2e x (b) y = e − x (c) y = − e x Differentiate (a) 9e x (b) − e x (c) e x + x 2 (d) 2x 3 − 3x 2 + 5x − e x (e) (e x + 1) 3 (f) (e x + 5)7 (g) (2e x − 3) 2 (h) xe x ex (i) x

(j) x 2 e x (k) (2x + 1) e x (l) (m)

ex 7x − 3 5x ex

4.

If f (x) = x 3 + 3x − e x, find f l(1) and f m(1) in terms of e.

5.

Find the exact gradient of the tangent to the curve y = e x at the point (1, e).

6.

Find the exact gradient of the normal to the curve y = e x at the point where x = 5.

7.

Find the gradient of the tangent to the curve y = 4e x at the point where x = 1.6, correct to 2 decimal places.

8.

Find the equation of the tangent to the curve y = − e x at the point (1, –e).

Chapter 4 Exponential and Logarithmic Functions

9.

139

11. Find the first and second derivatives of y = 7e x. Hence show

Find the equation of the normal to the curve y = e x at the point where x = 3, in exact form.

that

10. Find the stationary point on the curve y = xe x and determine its nature. Hence sketch the curve.

d 2y dx 2

= y.

12. If y = 2e x + 1, show that

d2y dx 2

= y − 1.

Function of a function rule Remember that the function of a function rule uses the result dy dy du = ´ . dx du dx

You studied this in Chapter 8 of the Preliminary Course book.

EXAMPLE Differentiate e x

2

+ 5x − 3

.

Solution Let u = x 2 + 5x − 3 Then y = e u dy du = 2x + 5 and = eu dx du dy dx

dy

du ´ du dx = e u (2x + 5) =

= e x + 5x − 3 (2x + 5) = (2x + 5) e x + 5x − 3 2

Can you see a quick way to do this?

2

If y = e f (x) then

Proof Let Then

u = f (x) y = eu dy = e u and du dy dy du ´ = dx du dx = e u f l(x) = f l(x) e f (x)

du = f l(x) dx

dy dx

= f l(x) ef (x)

140

Maths In Focus Mathematics HSC Course

EXAMPLES 1. Differentiate e 5x − 2

Solution y l = f l(x) e f (x) = 5e 5x − 2 2. Differentiate x 2 e 3x.

Solution

This is the product rule from Chapter 8 of the Preliminary Course book.

dy dx

= ulv + vlu = 2x . e3x + 3e3x . x2 = xe3x (2 + 3x)

3. Given y = 2e 3x + 1, show that

d2 y dx 2

= 9 (y − 1) .

Solution y = 2e 3x + 1 dy = 6e 3x dx d2 y = 18e 3x dx 2 = 9 (2e 3x) = 9 (2e 3x + 1 − 1) = 9 (y − 1)

4.2 Exercises 1.

Differentiate (a) e 7x (b) e − x (c) e 6x − 2 (d) e x + 1 (e) ex + 5x + 7 (f) e 5x (g) e − 2x (h) e 10x (i) e 2x + x (j) x 2 + 2x + e 1 − x (k) (x + e 4x)5 (l) xe 2x

e 3x x2 3 5x (n) x e e 2x + 1 (o) 2x + 5

(m)

2

3

2.

Find the second derivative of (e 2x + 1)7.

3.

If f (x) = e 3x − 2, find the exact value of f l(1) and f m(0) .

4.

Find the gradient of the tangent to the curve y = e 5x at the point where x = 0.

Chapter 4 Exponential and Logarithmic Functions

5.

6.

7.

Find the equation of the tangent to the curve y = e 2x − 3x at the point (0, 1).

11. Prove

Find the exact gradient of the normal to the curve y = e 3x at the point where x = 1.

12. Show

Find the equation of the tangent to the curve y = e x at the point (1, e). 2

If f (x) = 4x 3 + 3x 2 − e − 2x, find f m(−1) in terms of e.

8.

9.

d2y dx 2

= 16y.

dx

2

−3

dy dx

+ 2y = 0, given

y = 3e 2x . d2 y dx 2

= b 2 y for y = ae bx .

13. Find the value of n if y = e 3x satisfies the equation d 2y dy +2 + ny = 0. 2 dx dx 14. Sketch the curve y = e x + x − 2, showing any stationary points and inflexions. 2

Find any stationary points on the curve y = x 2 e 2x and sketch the curve.

10. If y = e 4x + e − 4x, show that

d2y

141

x2 + 1 , ex showing any stationary points and inflexions.

15. Sketch the curve y =

Integration of Exponential Functions Since

d x (e ) = e x, then the reverse must be true. dx

# e x dx = e x + C To find the indefinite integral (primitive function) when the function of a function rule is involved, look at the derivative first.

EXAMPLE Differentiate e 2x + 1. Hence find Find

# 2e 2x + 1 dx.

# e 2x + 1 dx.

Solution d 2x + 1 (e ) = 2 e 2x + 1 dx

`

# 2e 2x + 1 dx = e 2x + 1 + C # e 2x + 1 dx = 12 # 2e 2x + 1 dx =

1 2x + 1 e +C 2

Integration is the inverse of differentiation.

142

Maths In Focus Mathematics HSC Course

In general

# e ax + b dx = 1a e ax + b + C Proof d ax + b (e ) = ae ax + b dx

` # ae ax + b dx = e ax + b + C # e ax + b dx = 1a # ae ax + b dx 1 = a e ax + b + C

EXAMPLES 1. Find

# (e 2x − e − x) dx.

Solution

# (e 2x − e − x) dx = 12 e 2x − (−11) e − x + C =

1 2x e + e−x + C 2

2. Find the exact area enclosed between the curve y = e 3x, the x-axis and the lines x = 0 and x = 2.

Solution Area = # e 3x dx 2

0

2 1 = ; e 3x E 3 0 1 6 1 0 = e − e 3 3 1 6 = (e − e 0) 3 1 = (e 6 − 1) units 2 3

3. Find the volume of the solid of revolution formed when the curve y = e x is rotated about the x-axis from x = 0 to x = 2.

Solution Use index laws to simplify (e x) 2.

y = ex ` y 2 = ( e x) 2 = e 2x

Chapter 4 Exponential and Logarithmic Functions

V = π # y2 dx b

a

= π # e2x dx 2

0

2 1 = π ; e2x E 2 0 1 4 1 0 =πc e − e m 2 2 1 4 1 =πc e − m 2 2 π 4 = (e − 1) units3 2

4.3 Exercises 1.

Find these indefinite integrals. (a) (b) (c) (d) (e) (f) (g) (h) (i) (j)

2.

3.

# e dx # e 4x dx # e − x dx # e 5x dx # e − 2x dx # e 4x + 1 dx # − 3e 5x dx # e 2t dt # (e 7x − 2) dx # (e x − 3 + x) dx 2x

#0

1

(b)

#0

2

(c)

#1

4

(d)

#2

3

(e)

#0

2

(f)

#1

2

(g)

#0

3

e 5x dx

(a)

#1

3

(b)

#0

2

2e 3y dy

#0

1

(e 3t + 4 − t) dt

(e)

#1

2

(e 4x + e 2x) dx

6.

Find the area enclosed by the curve y = x + e − x, the x-axis and the lines x = 0 and x = 2, correct to 2 decimal places.

7.

Find the area bounded by the curve y = e 5x, the x-axis and the lines x = 0 and x = 1, correct to 3 significant figures.

8.

Find the exact volume of the solid of revolution formed when the curve y = e x is rotated about the x-axis from x = 0 to x = 3.

9.

Find the volume of the solid formed when the curve y = e − x + 1 is rotated about the x-axis from x = 1 to x = 2, correct to 1 decimal place.

(e x − x) dx

e − x dx

(d)

Find the exact area bounded by the curve y = e 4x − 3, the x-axis and the lines x = 0 and x = 1.

(e 2x + 1) dx

Evaluate correct to 2 decimal places.

(ex + 5 + 2x − 3) dx

5.

2e 3x + 4 dx

(e 2x − e − x) dx

6

Find the exact area enclosed by the curve y = 2e 2x, the x-axis and the lines x = 1 and x = 2.

− e− x dx

(3x 2 − e 2x) dx

#5

4.

Evaluate in exact form. (a)

(c)

143

144

Maths In Focus Mathematics HSC Course

10. Use Simpson’s rule with 3 function values to find an approximation to

#1

2

xe x dx,

correct to 1 decimal place. 11. (a) Differentiate x 2 e x .

(b) Hence find # x(2 + x)e dx. x

12. The curve y = e x + 1 is rotated about the x-axis from x = 0 to x = 1. Find the exact volume of the solid formed. 13. Find the exact area enclosed between the curve y = e 2x and the lines y = 1 and x = 2.

Application

You will study these formulae in Chapter 6.

The exponential function occurs in many fields, such as science and economics. P = P0 e kt is a general formula that describes exponential growth. P = P0 e

- kt

is a general formula that describes exponential decay.

Logarithms ‘Logarithm’ is another name for the index or power of a number. Logarithms are related to exponential functions, and allow us to solve equations like 2 x = 5. They also allow us to change the subject of exponential equations such as y = e x to x.

Definition If y = a x, then x is called the logarithm of y to the base a.

If y = a x, then x = loga y

Logarithm keys log is used for log 10 x In is used for log e x

Chapter 4 Exponential and Logarithmic Functions

145

EXAMPLES 1. Find log 10 5.3 correct to 1 decimal place.

Solution log 10 5.3 = 0.724275869 = 0.7 correct to 1 decimal place

Use the log key.

2. Evaluate log e 80 correct to 3 significant figures.

Solution log e 80 = 4.382026634 = 4.38 correct to 3 significant figures 3. Evaluate log 3 81.

Solution Let

log3 81 = x

Then 3 x = 81 i.e. 3x = 34 x=4 ` So log3 81 = 4.

(by definition)

4. Find the value of log 2

Solution 1 =x 4 1 2x = Then 4 1 = 2 2 = 2−2 x = −2 ` 1 So log2 = − 2. 4 Let

log2

1 . 4

Use the In key.

146

Maths In Focus Mathematics HSC Course

Class Investigation 1. Sketch the graph of y = log 2 x. There is no calculator key for logarithms to the base 2. Use the definition of a logarithm to change the equation into index form, and the table of values: x y

–3

–2

–1

0

1

2

3

2. On the same set of axes, sketch the curve y = 2 x and the line y = x. What do you notice?

4.4 Exercises 1.

2.

3.

Evaluate (a) log2 16 (b) log4 16 (c) log5 125 (d) log3 3 (e) log7 49 (f) log7 7 (g) log5 1 (h) log2 128 Evaluate (a) 3 log2 8 (b) log5 25 + 1 (c) 3 – log3 81 (d) 4 log3 27 (e) 2 log10 10 000 (f) 1 + log4 64 (g) 3 log4 64 + 5 (h) 2 + 4 log6 216 log3 9 (i) 2 log8 64 + 4 (j) log2 8 Evaluate 1 (a) log 2 2 (b) log 3 3 (c) log4 2

(d) log 5

1 25

(e) log 7 4 7 1 (f) log3 3 3 1 (g) log4 2 (h) log8 2 (i) log 6 6 6 (j) log2 4.

2 4

Evaluate correct to 2 decimal places. (a) log10 1200 (b) log10 875 (c) loge 25 (d) ln 140 (e) 5 ln 8 (f) log10 350 + 4.5 log10 15 (g) 2 (h) ln 9.8 + log10 17 log10 30 (i) loge 30 (j) 4 ln 10 – 7

Chapter 4 Exponential and Logarithmic Functions

5.

Write in logarithmic form. (a) 3 x = y (b) 5 x = z (c) x 2 = y (d) 2 b = a (e) b 3 = d (f) y = 8 x (g) y = 6 x (h) y = e x (i) y = a x (j) Q = e x

6.

Write in index form. (a) log3 5 = x (b) loga 7 = x (c) log3 a = b (d) logx y = 9 (e) loga b = y (f) y = log 2 6 (g) y = log 3 x (h) y = log 10 9 (i) y = ln 4 (j) y = log 7 x

7.

Solve for x, correct to 1 decimal place where necessary. (a) log 10 x = 6 (b) log 3 x = 5 (c) logx 343 = 3 (d) logx 64 = 6 (e) log 5

1 =x 5

(f) log x

3=

1 2

(g) ln x = 3.8 (h) 3 log 10 x − 2 = 10 3 2 1 (j) log x 4 = 3 (i) log 4 x =

8.

Evaluate y given that log y 125 = 3.

9.

If log 10 x = 1.65, evaluate x correct to 1 decimal place.

10. Evaluate b to 3 significant figures if log e b = 0.894. 11. Find the value of log2 1. What is the value of loga 1? 12. Evaluate log5 5. What is the value of loga a? 13. (a) Evaluate ln e without a calculator. (b) Using a calculator, evaluate (i) loge e3 (ii) loge e2 (iii) loge e5 (iv) loge e 1 (v) loge e (vi) eln 2 (vii) eln 3 (viii) eln 5 (ix) eln 7 (x) eln 1 (xi) eln e 14. Sketch the graph of y = log e x. What is its domain and range? 15. Sketch y = 10 x, y = log10 x and y = x on the same number plane. What do you notice about the relationship of the curves to the line? 16. Change the subject of y = log e x to x.

147

148

Maths In Focus Mathematics HSC Course

Class Discussion 1. Investigate these questions on the calculator. Can you see some patterns? (a) loge e (b) loge e2 (c) loge e3 (d) loge e4 (e) loge e5 Can you write a rule for loge ex? 2. Evaluate using a calculator. Can you write a rule to show this pattern? (a) eln 1 (b) eln 2 (c) eln 3 (d) eln 4 (e) eln 5 Can you write a rule for eln x? 3. Do these rules work if x is negative?

Logarithm laws

This corresponds to the law a m ´ a n = a m + n from Chapter 1 of the Preliminary Course book.

Because logarithms are closely related to indices there are logarithm laws that correspond to the index laws. loga (xy) = loga x + loga y

Proof x = a m and y = a n m = log a x and n = log a y xy = a m ´ a n = am + n ` loga (xy) = m + n (by definition) = loga x + loga y

Let Then

This corresponds to the law a m ÷ a n = a m − n .

x log a b y l = log a x − log a y

Chapter 4 Exponential and Logarithmic Functions

149

Proof Let Then

x = a m and y = a n m = loga x and n = loga y

x m n y =a ÷a = am − n x ` loga b y l = m − n = loga x − loga y

(by definition)

This corresponds to the law (a m) n = a mn .

loga x n = n loga x

Proof Let Then

`

x = am m = loga x

x n = (a m) n = a mn n loga x = mn = n loga x

(by definition)

EXAMPLES 1. Evaluate log6 3 + log6 12.

Solution log6 3 + log6 12 = log6 (3 ´ 12) = log6 36 =2

log 6 36 = 2, since 6 2 = 36.

2. Given log5 3 = 0.68 and log5 4 = 0.86, find (a) log 5 12 (b) log5 0.75 (c) log 5 9 (d) log 5 20 CONTINUED

150

Maths In Focus Mathematics HSC Course

Solution (a) log5 12 = log5 (3 ´ 4) = log5 3 + log5 4 = 0.68 + 0.86 = 1.54 3 (b) log 5 0.75 = log 5 4 = log 5 3 − log 5 4 = 0.68 − 0.86 = − 0.18 (c) log 5 9 = log 5 3 2 = 2 log 5 3 = 2 ´ 0.68 = 1.36 (d) log 5 20 = log 5 (5 ´ 4) = log 5 5 + log 5 4

log 5 5 = 1, since 5 1 = 5.

= 1 + 0.86 = 1.86 3. Solve log2 12 = log2 3 + log2 x.

Solution log 2 12 = log 2 3 + log 2 x = log 2 3x So 12 = 3x 4=x

4.5 Exercises 1.

Use the logarithm laws to simplify (a) loga 4 + loga y (b) loga 4 + loga 5 (c) loga 12 – loga 3 (d) loga b – loga 5 (e) 3 logx y + logx z (f) 2 logk 3 + 3 logk y (g) 5 loga x – 2 loga y (h) loga x + loga y – loga z (i) log10 a + 4 log10 b + 3 log10 c (j) 3 log3 p + log3 q – 2 log3 r

2.

Given log7 2 = 0.36 and log7 5 = 0.83, find (a) log7 10 (b) log7 0.4 (c) log7 20 (d) log7 25 (e) log7 8 (f) log7 14 (g) log7 50 (h) log7 35 (i) log7 98 (j) log7 70

Chapter 4 Exponential and Logarithmic Functions

3.

4.

5.

Use the logarithm laws to evaluate (a) log5 50 – log5 2 (b) log2 16 + log2 4 (c) log4 2 + log4 8 (d) log5 500 – log5 4 (e) log9 117 – log9 13 (f) log8 32 + log8 16 (g) 3 log2 2 + 2 log2 4 (h) 2 log4 6 – (2 log4 3 + log4 2) (i) log6 4 – 2 log6 12 (j) 2 log3 6 + log3 18 – 3 log3 2 If loga 3 = x and loga 5 = y, find an expression in terms of x and y for (a) loga 15 (b) loga 0.6 (c) loga 27 (d) loga 25 (e) loga 9 (f) loga 75 (g) loga 3a a (h) loga 5 (i) loga 9a 125 (j) loga a If loga x = p and loga y = q, find, in terms of p and q. (a) loga xy (b) loga y3 y (c) loga x

(d) loga x2 (e) loga xy5 x2 (f) loga y (g) loga ax a (h) loga 2 y (i) loga a3y x (j) loga ay 6.

If loga b = 3.4 and loga c = 4.7, evaluate c (a) loga b (b) loga bc2 (c) loga (bc)2 (d) loga abc (e) loga a2c (f) loga b7 a (g) loga c (h) loga a3 (i) loga bc4 (j) loga b4c2

7.

Solve (a) log4 12 = log4 x + log4 3 (b) log3 4 = log3 y – log3 7 (c) loga 6 = loga x – 3 loga 2 (d) log2 81 = 4 log2 x (e) logx 54 = logx k + 2 logx 3

Change of base Sometimes we need to evaluate logarithms such as log2 7. We use a change of base formula.

log a x =

log b x log b a

151

152

Maths In Focus Mathematics HSC Course

Proof Let y = log a x Then x = a y Take logarithms to the base b of both sides of the equation: logb x = logb a y = y logb a logb x =y ` logb a = loga x You can use the change of base formula to find the logarithm of any number, such as log 5 2. You change it to either log 10 x or log e x, and use a calculator.

EXAMPLE Find the value of log 5 2, correct to 2 decimal places.

Solution log 5 2 = You can use either log or In

log 2

log 5 Z 0.430676558 = 0.43

(by change of base)

Exponential equations You can also use the change of base formula to solve exponential equations such as 5 x = 7. You studied exponential equations such as 2 x = 8 in the Preliminary Course. Exponential equations such as 2 x = 9 can be solved by taking logarithms of both sides, or by using the definition of a logarithm and the change of base formula.

Chapter 4 Exponential and Logarithmic Functions

153

EXAMPLES 1. Solve 5 x = 7 correct to 1 decimal place.

Solution 5x = 7 Using the definition of a logarithm, this means: log 5 7 = x log 7

=x

(using change of base formula)

log 5 1.2 = x

You can use either log or ln.

If you do not like to solve the equation this way, you can use the logarithm laws instead. Taking logs of both sides: log 5 x = log 7 x log 5 = log 7 `

x=

n

Use loga x = n loga x

log 7

log 5 = 1.2 correct to 1 decimal place

2. Solve 4 y − 3 = 9 correct to 2 decimal places.

Solution 4y − 3 = 9 Using the logarithm definition and change of base: log 4 9 = y − 3 log 9 =y−3 log 4 log 9 +3=y log 4 4.58 = y Using the logarithm laws: Taking logs of both sides: log 4 y − 3 = log 9 (y − 3) log 4 = log 9 y−3= y=

log 9 log 4 log 9

+3 log 4 = 4.58 correct to 2 decimal place

154

Maths In Focus Mathematics HSC Course

4.6 Exercises 1.

2.

3.

Use the change of base formula to evaluate to 2 decimal places. (a) log4 9 (b) log6 25 (c) log9 200 (d) log2 12 (e) log3 23 (f) log8 250 (g) log5 9.5 (h) 2 log4 23.4 (i) 7 – log7 108 (j) 3 log11 340 By writing each equation as a logarithm and changing the base, solve the equation correct to 2 significant figures. (a) 4 x = 9 (b) 3 x = 5 (c) 7 x = 14 (d) 2 x = 15 (e) 5 x = 34 (f) 6 x = 60 (g) 2 x = 76 (h) 4 x = 50 (i) 3 x = 23 (j) 9 x = 210 Solve, correct to 2 decimal places. (a) 2 x = 6 (b) 5 y = 15 (c) 3 x = 20

(d) (e) (f) (g) (h) (i) (j)

7 m = 32 4 k = 50 3t = 4 8 x = 11 2 p = 57 4 x = 81.3 6 n = 102.6

4.

Solve, to 1 decimal place. (a) 3 x + 1 = 8 (b) 53n = 71 (c) 2 x − 3 = 12 (d) 4 2n − 1 = 7 (e) 7 5x + 2 = 11 (f) 8 3 − n = 5.7 (g) 2 x + 2 = 18.3 (h) 37xk − 3 = 32.9 (i) 9 2 = 50 (j) 6 2y + 1 = 61.3

5.

Solve each equation correct to 3 significant figures. (a) e x = 200 (b) e 3t = 5 (c) 2e t = 75 (d) 45 = e x (e) 3000 = 100e n (f) 100 = 20e 3t (g) 2000 = 50e 0.15t (h) 15 000 = 2000e 0.03k (i) 3Q = Qe 0.02t (j) 0.5M = Me 0.016k

Chapter 4 Exponential and Logarithmic Functions

155

Derivative of the Logarithmic Function Drawing the derivative (gradient) function of a logarithm function gives a hyperbola.

EXAMPLE Sketch the derivative function of y = log 2 x.

Solution The gradient is always positive but is decreasing.

If y = log e x then

Proof

dy dx

1 = x

dy

dy

dx

1 dx dx dy Given y = log e x Then x = e y dx = ey dy dy 1 ` = dx e y 1 =x =

=

1 dx

is a special result that

dy can be proved by differentiating from first principles.

156

Maths In Focus Mathematics HSC Course

Function of a function rule

If y = log e f (x), then

Proof Let Then ` Also

u = f (x) y = log e u dy 1 = du u du = f l(x) dx dy dy du . = dx du dx 1 = u . f l(x) 1 . = f l(x) f (x)

EXAMPLES 1. Differentiate log e (x 2 − 3x + 1) .

Solution d 2x − 3 [loge (x 2 − 3x + 1)] = 2 dx x − 3x + 1 2. Differentiate log e

x+1 . 3x − 4

Solution x+1 3x − 4 = loge (x + 1) − loge (3x − 4)

Let y = loge dy dx

3 1 − x + 1 3x − 4 1 (3x − 4) − 3 (x + 1) = (x + 1) (3x − 4) 3x − 4 − 3x − 3 = (x + 1) (3x − 4) −7 = (x + 1) (3x − 4) =

dy dx

= f l(x) .

f l(x) 1 = f (x) f (x)

Chapter 4 Exponential and Logarithmic Functions

157

3. Find the gradient of the normal to the curve y = loge (x 3 − 5) at the point where x = 2.

Solution dy

3x 2 dx x 3 − 5 When x = 2, dy 3 (2) 2 = dx 2 3 − 5 m1 = 4 =

The normal is perpendicular to the tangent i.e. m1 m2 = − 1 4m 2 = − 1 1 ` m2 = − 4 4. Differentiate y = log 2 x.

Solution y = log 2 x log e x = log e 2 1 = ´ log e x log e 2 dy 1 1 = ´ dx log e 2 x 1 = x log e 2 5. Find the derivative of 2x.

Solution 2 = e ln 2 ` 2 x = (e ln 2) x = e x ln 2 dy = ln 2e x ln 2 dx = ln 2 ´ 2 x = 2 x ln 2

This result comes from the Preliminary Course.

158

Maths In Focus Mathematics HSC Course

4.7 Exercises 1.

Differentiate

6.

Find the gradient of the normal to the curve y = log e (x 4 + x) at the point (1, log e 2).

7.

Find the exact equation of the normal to the curve y = log e x at the point where x = 5.

8.

Find the equation of the tangent to the curve y = log e (5x + 4) at the point where x = 3.

9.

Find the point of inflexion on the curve y = x loge x − x 2 .

(a) x + log e x (b) 1 − log e 3x (c) ln (3x + 1) (d) loge (x 2 − 4) (e) ln (5x 3 + 3x − 9) (f) loge (5x + 1) + x 2 (g) 3x 2 + 5x − 5 + ln 4x (h) log e (8x − 9) + 2 (i) log e (2x + 4) (3x − 1) 4x + 1 (j) log e 2x − 7 (k) (1 + loge x) 5 (l) (ln x − x) 9 (m) (loge x) 4 (n) (x 2 + loge x) 6 (o) x log e x

10. Find the stationary point on the ln x curve y = x and determine its nature.

(r) x 3 log e (x + 1)

11. Sketch, showing any stationary points and inflexions. (a) y = x − log e x (b) y = (log e x − 1) 3 (c) y = x ln x

(s) log e (log e x)

12. Find the derivative of log3 (2x + 5).

log e x x (q) (2x + 1) log e x

(p)

ln x (t) x−2 (u)

2x

e loge x

(v) e x ln x (w) 5 (loge x) 2 2.

If f (x) = log e 2 − x , find f l(1).

3.

Find the derivative of log 10 x.

4.

Find the equation of the tangent to the curve y = log e x at the point (2, log e 2).

5.

Find the equation of the tangent to the curve y = log e (x − 1) at the point where x = 2.

13. Differentiate (a) 3 x (b) 10 x (c) 2 3x − 4 14. Find the equation of the tangent to the curve y = 4 x + 1 at the point (0, 4). 15. Find the equation of the normal to the curve y = log 3 x at the point where x = 3.

Chapter 4 Exponential and Logarithmic Functions

159

Integration and the Logarithmic Function 1 # dx x = # x dx = log e x + C

#

f l(x) f (x)

Integration is the inverse of differentiation.

dx = log e f (x) + C

EXAMPLES 1 1. Find the area enclosed between the hyperbola y = x , the x-axis and the lines x = 1 and x = 2, giving the exact value.

Solution

21 A = # x dx 1 = 7 loge x A 21 = loge 2 − loge 1 = loge 2

So area is log e 2 units2. 2. Find

#

Solution

#

x2 dx. x3 + 7

3x 2 x2 1 dx = # 3 dx 3 x +7 x +7 1 = loge (x 3 + 7) + C 3 3

CONTINUED

160

Maths In Focus Mathematics HSC Course

3. Find

#

Solution

#

x+1 dx. x2 + x + 4

2 (x + 1) x+1 1 dx = # 2 dx 2 x + 2x + 4 x + 2x + 4 2x + 2 1 = # 2 dx 2 x + 2x + 4 1 = loge (x 2 + 2x + 4) + C 2 2

4.8 Exercises 1.

2.

Find the indefinite integral (primitive function) of 2 (a) 2x + 5 4x (b) 2x 2 + 1 5x 4 (c) 5 x −2 1 (d) 2x 2 (e) x 5 (f) 3x 2x − 3 (g) 2 x − 3x x (h) 2 x +2 3x (i) 2 x +7 x+1 (j) 2 x + 2x − 5

3.

(a) (b) (c) (d) (e)

# 4x4− 1 dx

(b)

# x dx +3

(c)

#

(d)

#

(e)

#

x2 dx 2x − 7 x5 dx 2x 6 + 5 x+3 dx x 2 + 6x + 2 3

2 dx 2x + 5 5 #2 x dx +1 2 7 #1 3x dx x +2 3 #0 42 x + 1 dx 2x + x + 1 4 x − 1 dx #3 2 x − 2x

#1

3

4.

Find the exact area between the 1 curve y = x , the x-axis and the lines x = 2 and x = 3.

5.

Find the exact area bounded by 1 the curve y = , the x-axis and x−1 the lines x = 4 and x = 7.

6.

Find the exact area between the 1 curve y = x , the x-axis and the lines y = x and x = 2 in the first quadrant.

7.

Find the area bounded by the x curve y = 2 , the x-axis and x +1 the lines x = 2 and x = 4, correct to 2 decimal places.

Find (a)

Evaluate correct to 1 decimal place.

Chapter 4 Exponential and Logarithmic Functions

8.

Find the exact volume of the solid formed when the curve 1 is rotated about the x-axis y= x from x = 1 to x = 3.

12. (a) Show that 3x + 3 1 2 . = + x2 − 9 x + 3 x − 3 3x + 3 (b) Hence find # 2 dx. x −9

9.

Find the volume of the solid formed when the curve 2 y= is rotated about the 2x − 1 x-axis from x = 1 to x = 5, giving an exact answer.

13. (a) Show that

10. Find the area between the curve y = ln x, the y-axis and the lines y = 2 and y = 4, correct to 3 significant figures. 11. Find the exact volume of the solid formed when the curve y = log e x is rotated about the y-axis from y = 1 to y = 3.

x−6 5 =1− . x−1 x−1 x−6 (b) Hence find # dx. x−1

14. Find the indefinite integral (primitive function) of 3 2x − 1. 15. Find, correct to 2 decimal places, the area enclosed by the curve y = log 2 x, the x-axis and the lines x = 1 and x = 3 by using Simpson’s rule with 3 function values.

161

162

Maths In Focus Mathematics HSC Course

Test Yourself 4 1.

2.

3.

4.

5.

6.

Evaluate to 3 significant figures. (a) e 2 − 1 (b) log 10 95 (c) log e 26 (d) log 4 7 (e) log 4 3 (f) ln 50 (g) e + 3 5e 3 (h) ln 4 (i) e ln 6 (j) e ln 2

7.

Find the volume of the solid formed if the area bounded by y = e 3x, the x-axis and the lines x = 1 and x = 2 is rotated about the x-axis.

8.

If log 7 2 = 0.36 and log 7 3 = 0.56, find the value of (a) log 7 6 (b) log 7 8 (c) log 7 1.5 (d) log 7 14 (e) log 7 3.5

Differentiate (a) e 5x (b) 2e 1 − x (c) log e 4x (d) ln (4x + 5) (e) xe x ln x (f) x (g) (e x + 1)10

9.

Find the area enclosed between the curve y = ln x, the y-axis and the lines y = 1 and y = 3.

Find the indefinite integral (primitive function) of (a) e 4x x (b) 2 x −9 (c) e − x 1 (d) x+4 Find the equation of the tangent to the curve y = 2 + e 3x at the point where x = 0. Find the exact gradient of the normal to the curve y = x − e − x at the point where x = 2. Find the exact area bounded by the curve y = e 2x, the x-axis and the lines x = 2 and x = 5.

10. (a) Use Simpson’s rule with 3 function values to find the area bounded by the curve y = ln x, the x-axis and the lines x = 2 and x = 4. (b) Change the subject of y = ln x to x. (c) Hence find the exact area in part (a). 11. Solve (a) 3 x = 8 (b) 2 3x − 4 = 3 (c) logx 81 = 4 (d) log6 x = 2 (e) 12 = 10e 0.01t 12. Evaluate (a)

#0

1

3e 2x dx

dx 3x − 2 2 2x 3 − x 2 + 5x + 3 (c) # dx x 1 (b)

#1

4

13. Find the equation of the tangent to the curve y = e x at the point (4, e 4). 14. Evaluate log 9 8 to 1 decimal place.

Chapter 4 Exponential and Logarithmic Functions

15. (a) Find the area bounded by the curve y = e x, the x-axis and the lines x = 1 and x = 2. (b) This area is rotated about the x-axis. Find the volume of the solid of revolution formed. 16. Simplify (a) 5 loga x + 3 loga y (b) 2 log x k − log x 3 + log x p

18. Find the stationary points on the curve y = x 3 e x and determine their nature. 19. Use the trapezoidal rule with 4 strips to find the area bounded by the curve y = ln (x 2 − 1), the x-axis and the lines x = 3 and x = 5. 20. Evaluate to 2 significant figures (a) log 10 4.5 (b) ln 3.7

17. Find the equation of the normal to the curve y = ln x at the point (2, ln 2).

Challenge Exercise 4 loge x

1.

Differentiate

2.

Find the exact gradient of the tangent to the curve y = e x + log x at the point where x = 1.

e 2x + x

.

Find the derivative of loge

7.

Use Simpson’s rule with 5 function values to find the volume of the solid formed when the curve y = e x is rotated about the y-axis from y = 3 to y = 5, correct to 2 significant figures.

8.

Differentiate 5 x.

9.

Show that

e

3.

If log b 2 = 0.6 and log b 3 = 1.2, find (a) log b 6b (b) log b 8 (c) log b 1.5b 2

1 . 2x − 3

6.

d 2 (x loge x) = x (1 + 2 loge x). dx

4.

Differentiate (e 4x + loge x) 9.

Hence evaluate

5.

Find the shaded area, correct to 2 decimal places.

giving an exact answer. 10. Find

#1

3

2x (1 + 2 loge x) dx,

# 3 x dx.

11. (a) Find the point of intersection of the curves y = log e x and y = log 10 x. (b) Find the exact equations of the tangents to the two curves at this point of intersection. (c) Find the exact length of the interval XY where X and Y are the y-intercepts of the tangents.

163

164

Maths In Focus Mathematics HSC Course

12. Use Simpson’s rule with 3 function values to find the area enclosed by the curve y = e 2x, the y-axis and the line y = 3, correct to 3 significant figures. x loge x

13. Find the derivative of

14. If y = e x + e − x, show

ex

d2y dx

2

= y.

.

15. Prove

d2y 2

−4

dx 5x y = 3e − 2 .

dy dx

− 5y − 10 = 0, given

16. Find the equation of the curve that has f m(x) = 12e 2x and a stationary point at ( 0 , 3) . 17. Sketch y = loge (x − x 2).

5

Trigonometric Functions TERMINOLOGY T ERMINOLOGY Amplitude: Maximum displacement from the mean or centre position in vibrating or oscillating motion

Radian: A unit of angular measurement defined as the length of arc of one unit that an angle subtended at the centre of a unit circle cuts off

Circular measure: The measurement of an angle as the length of an arc cut off by an angle at the centre of a unit circle. The units are called radians

Sector: Part of a circle bounded by the centre and arc of a circle cut off by two radii

Period: One complete cycle of a wave or other recurring function

Segment: Part of a circle bounded by a chord and the circumference of a circle

Chapter 5 Trigonometric Functions

167

INTRODUCTION IN THIS CHAPTER YOU will learn about circular measure, which uses radians

rather than degrees. Circular measure is very useful in solving problems involving properties of the circle, such as arc length and areas of sectors and segments, which you will study in this chapter. You will also study trigonometric graphs in greater detail than in the Preliminary Course. These graphs have many applications in the movement of such things as sound, light and waves.

Circular measure is also useful in calculus, and you will learn about differentiation and integration of trigonometric functions in this chapter.

Circular Measure Radians We are used to degrees being used in geometry and trigonometry. However, they are limited. For example, when we try to draw the graph of a trigonometric function, it is difficult to mark degrees along the x- or y-axis as there is no concept of how large a degree is along a straight line. This is one of the reasons we use radians. Other reasons for using radians are that they make solving circle problems simple and they are used in calculus. A radian is based on the length of an arc in a unit circle (a circle with radius 1 unit), so we can visualise a radian as a number that can be measured along a line.

1 unit 1c

1 unitt

The symbol for 1 radian is 1c but we usually don’t use the symbol as radians are simply distances, or arc lengths.

168

Maths In Focus Mathematics HSC Course

One radian is the angle that an arc of 1 unit subtends at the centre of a circle of radius 1 unit.

π radians = 180°

Proof Circumference of the circle with radius 1 unit is given by: C = 2π r = 2π (1) = 2π The arc length of the whole circle is 2π. ` there are 2π radians in a whole circle. But there are 360° in a whole circle (angle of revolution). So 2π = 360° π = 180°

EXAMPLES 1. Convert

3π into degrees. 2

Solution Since π = 180°, 3π 3 (180°) = 2 2 = 270° 2. Change 60° to radians, leaving your answer in terms in π.

Solution 180° = π radians π So 1° = radians 180 π 60° = ´ 60 180 60π = 180 π = 3

2π 1 unit

Chapter 5 Trigonometric Functions

169

3. Convert 50° into radians, correct to 2 decimal places.

Solution 180° = π radians π So 1° = radians 180 π 50° = ´ 50 180 50π = 180 Z 0.87 4. Change 1.145 radians into degrees, to the nearest minute.

Solution π radians = 180° 180° ` 1 radian = π 180° 1.145 radians = π ´ 1.145 = 65.6° = 65° 36l Press 180 ' SHIFT ° , ,,

π ´ 1.145 =

5. Convert 38° 41l into radians, correct to 3 decimal places.

Solution 180° = π radians π 1° = 180° π 38° 41l = ´ 38° 41l 180° = 0.675 6. Evaluate cos 1.145 correct to 2 decimal places.

Solution For cos 1.145, use radian mode on your calculator. cos 1.145 = 0.413046135 = 0.41 correct to 2 decimal places

The calculator may give the answer in degrees and minutes. Simply change this into a decimal.

170

Maths In Focus Mathematics HSC Course

π 180 and 1 radian = π . You can use 180 these as conversions rather than starting each time from π = 180°. Notice from the examples that 1° =

180 To change from radians to degrees: Multiply by π . To change from degrees to radians: Multiply by

π . 180

π 180 = 0.017 radians 180 Also 1 radian = π = 57° 18l Notice that 1° =

While we can convert between degrees and radians for any angle, there are some special angles that we use regularly in this course. It is easier if you know these without having to convert each time.

π 2 π 3π 2 2π π 4 π 3 π 6

= 90° = 180° = 270° = 360° = 45° = 60° = 30°

5.1 Exercises 1.

Change into degrees. π (a) 5 2π (b) 3 5π (c) 4 7π (d) 6 (e) 3π

(f) (g) (h) (i) (j)

7π 9 4π 3 7π 3 π 9 5π 18

Chapter 5 Trigonometric Functions

2.

Convert into radians in terms of π. (a) 135° (b) 30° (c) 150° (d) 240° (e) 300° (f) 63° (g) 15° (h) 450° (i) 225° (j) 120°

3.

Change into radians, correct to 2 decimal places. (a) 56° (b) 68° (c) 127° (d) 289° (e) 312°

4.

Change into radians, to 2 decimal places. (a) 18° 34l (b) 35° 12l (c) 101° 56l (d) 88° 29l (e) 50° 39l

5.

Change these radians into degrees and minutes, to the nearest minute. (a) 1.09 (b) 0.768 (c) 1.16 (d) 0.99 (e) 0.32 (f) 3.2 (g) 2.7 (h) 4.31 (i) 5.6 (j) 0.11

6.

Find correct to 2 decimal places. (a) sin 0.342 (b) cos 1.5 (c) tan 0.056 (d) cos 0.589 (e) tan 2.29 (f) sin 2.8 (g) tan 5.3 (h) cos 4.77 (i) cos 3.9 (j) sin 2.98

Trigonometric Results All the trigonometry that you studied in the Preliminary Course can be done using radians instead of degrees.

Special triangles The two triangles that give exact trigonometric ratios can be drawn using radians rather than degrees. π 6 π 4 2

2

3

1

π 3

π 4 1

1

171

The calculator may give the answer in degrees and minutes. Simply change this into a decimal.

172

Maths In Focus Mathematics HSC Course

Using trigonometric ratios and these special triangles gives the results:

π 1 = 4 2 π 1 cos = 4 2 π tan = 1 4 sin

3 π = 3 2 π 1 cos = 3 2 π tan = 3 3 sin

π 1 = 6 2 3 π cos = 6 2 π 1 tan = 6 3 sin

EXAMPLE Find the exact value of 3 cos

π π − cosec . 4 6

Solution π π − cosec 4 6 π 1 = 3 cos − 6 sin π 4 3n 2 = 3d − 2 1 3 3 = − 2 2 3 3 2 2 = − 2 2 3 3 −2 2 = 2

3 cos

Angles of any magnitude The results for angles of any magnitude can also be looked at using radians. If we change degrees for radians, the ASTC rule looks like this:

Chapter 5 Trigonometric Functions

y

π 2

2nd quadrant

1st quadrant

π-θ

θ

S

A

π

0 2π

T

x

C 2π - θ

π+θ 3rd quadrant

4th quadrant 3π 2

We can summarise the trigonometric ratios for angles of any magnitude in radians as follows: First quadrant: Angle θ : sin θ is positive cos θ is positive tan θ is positive Second quadrant: Angle π − θ : sin (π − θ ) = sin θ cos (π − θ ) = − cos θ tan (π − θ ) = − tan θ Third quadrant: Angle π + θ : sin (π + θ ) = − sin θ cos (π + θ ) = − cos θ tan (π + θ ) = tan θ Fourth quadrant: Angle 2π − θ : sin (2π − θ ) = − sin θ cos (2π − θ ) = cos θ tan (2π − θ ) = − tan θ

173

174

Maths In Focus Mathematics HSC Course

EXAMPLES 1. Find the exact value of sin

5π . 4

Solution 4π 4 5π 4π π So = + 4 4 4 π =π+ 4 π=

In the 3rd quadrant, angles are in the form π + θ, so the angle is in the 3rd quadrant, and sin θ is negative in the 3rd quadrant. sin c

5π π m = sin c π + m 4 4 π = − sin 4 1 =− 2

π 4 2

1

π 4 1

2. Find the exact value of cos

11π . 6

Solution 12π 6 11π 12π π So = − 6 6 6 π = 2π − 6 2π =

In the 4th quadrant, angles are in the form 2π − θ, so the angle is in the 4th quadrant, and cos θ is positive. cos c

11π π m = cos c 2π − m 6 6 π = cos 6 3 = 2

π 6

2

3

π 3 1

Chapter 5 Trigonometric Functions

3. Find the exact value of tan c −

175

4π m. 3

Solution 3π 3 4π 3π π So − = −c + m 3 3 3 π = − cπ + m 3 π=

In the 2nd quadrant, angles are in the form − (π + θ ), so the angle is in the 2nd quadrant, and tan θ is negative in the 2nd quadrant. π 6

4π π m = − tan ; − c π + m E tan c − 3 3 π = − tan 3 =− 3

2

3

π 3 1

We can use the ASTC rule to solve trigonometric equations. You studied these in the Preliminary Course using degrees.

EXAMPLES 1. Solve cos x = 0.34 for 0 ≤ x ≤ 2π .

Solution cos is positive in the 1st and 4th quadrants. Using a calculator (with radian mode) gives x = 1.224. i.e. cos 1.224 = 0.34 π This is an angle in the 1st quadrant since 1.224 < . 2 In the 4th quadrant, angles are in the form of 2π − θ. So the angle in the 4th quadrant will be 2π − 1.224. So x = 1.224, 2π − 1.224 = 1.224, 5.06. 2. Solve sin α = −

1 in the domain 0 ≤ α ≤ 2π. 2

Solution Here the sin of the angle is negative. Since sin is positive in the 1st and 2nd quadrants, it is negative in the 3rd and 4th quadrants. CONTINUED

You could change the radians into degrees before finding these ratios.

You could solve all equations in degrees then convert into radians where necessary.

176

Maths In Focus Mathematics HSC Course

π 1 = 4 2 In the 3rd quadrant, angles are π + θ and in the 4th quadrant, 2π − θ.

sin

π π So α = π + , 2π − 4 4 4π π 8π π = + , − 4 4 4 4 5π 7π , = 4 4

π 4 2

1

π 4 1

5.2 Exercises 1.

Copy and complete the table, giving exact values. π 3

π 4

3.

Find the exact value, with rational denominator where relevant. π π (a) cos2 + sin2 4 6 π π π π (b) sin cos − cos sin 4 4 3 3 π π π π (c) cos cos + sin sin 6 3 6 3 π π (d) sin2 + cos2 4 4 π π (e) sec2 + sin2 3 6

4.

Find the exact value with rational denominator of π π π π (a) sin cos + sin cos 4 4 3 3 π π tan − tan 4 3 (b) π π 1 + tan tan 4 3

5.

Show that π π π π cos cos + sin sin = 4 4 3 3 π π π π sin cos + cos sin . 4 4 6 6 3π π (a) Show that =π− . 4 4 (b) Which quadrant is the angle

π 6

sin cos tan cosec sec cot 2. Remember that 2 2 tan θ = (tanθ )

Find the exact value, with rational denominator where relevant. π 6 π 2 c sin m 4 π 3 c cos m 6 π π tan + tan 3 6 π π sin − cos 4 4 π π tan + cos 3 3 π π sec − tan 4 3 π π cos + cot 4 4 π π 2 c cos − tan m 4 4 π π 2 c sin + cos m 6 6

(a) tan 2 (b) (c) (d) (e) (f) (g) (h) (i) (j)

6.

3π in? 4 (c) Find the exact value of cos

3π . 4

Chapter 5 Trigonometric Functions

7.

8.

9.

5π π =π− . 6 6 (b) Which quadrant is the angle 5π in? 6 5π (c) Find the exact value of sin . 6 7π π (a) Show that = 2π − . 4 4 (b) Which quadrant is the angle 7π in? 4 7π (c) Find the exact value of tan . 4 4π π (a) Show that =π+ . 3 3 (b) Which quadrant is the angle (a) Show that

4π in? 3 (c) Find the exact value of 4π cos . 3 5π π 10. (a) Show that = 2π − . 3 3 (b) Which quadrant is the angle 5π in? 3 5π (c) Find the exact value of sin . 3 11. Find the exact value of each ratio. 3π (a) tan 4 11π (b) cos 6 2π (c) tan 3 5π (d) sin 4 7π (e) tan 6 13π π 12. (a) (i) Show that = 2π + . 6 6 (ii) Which quadrant is the 13π angle in? 6 (iii) Find the exact value of cos

13π . 6

(b) Find the exact value of 9π (i) sin 4 7π (ii) tan 3 11π (iii) cos 4 19π (iv) tan 6 10π (v) sin 3 13. Solve for 0 ≤ x ≤ 2π 1 (a) cos x = 2 1 (b) sin x = − 2 (c) tan x = 1 (d) tan x = 3 3 (e) cos x = − 2 14. Simplify (a) sin ] π − θ g (b) tan (2π − x) (c) cos ] π + α g π (d) cos c − x m 2 π (e) tan c − θ m 2 15. Simplify 1 − sin2 c

π − θm 2

π 3 and < x < π , find 4 2 the value of cos x and sin x.

16. If tan x = −

17. Solve tan2 x – 1 = 0 for 0 ≤ x ≤ 2π

177

178

Maths In Focus Mathematics HSC Course

Circle Results The area and circumference of a circle are useful in this section.

Area of a circle

A = π r 2 where r is the radius of the circle

Circumference of a circle

C = 2π r = π d We use equal ratios to find the other circle results. The working out is simpler when using radians instead of degrees.

Length of arc

l = rθ (θ is in radians)

Proof arc length l

Using 360° instead of 2π gives a different formula. Can you find it?

=

angle θ

circumference whole revolution θ l = 2 π r 2π θ 2π r ` l= 2π = rθ

Chapter 5 Trigonometric Functions

EXAMPLES π 1. Find the length of the arc formed if an angle of is subtended at the 4 centre of a circle of radius 5 m.

Solution l = rθ π = 5c m 4 5π = m 4 2. The area of a circle is 450 cm2 . Find, in degrees and minutes, the angle subtended at the centre of the circle by a 2.7 cm arc.

Solution A = π r2 450 = π r 2 450 2 π =r 450 π =r 11.97 = r Now l = r θ 2.7 = 11.97θ 2 .7 =θ 11.97 0.226 = θ π radians = 180° 180° 1 radian = π 180° 0.226 radians = π ´ 0.226 = 12.93° = 12° 56l So θ = 12° 56l.

179

180

Maths In Focus Mathematics HSC Course

5.3 Exercises 1.

2.

3.

4.

Find the exact arc length of a circle if (a) radius is 4 cm and angle subtended is π (b) radius is 3 m and angle π subtended is 3 (c) radius is 10 cm and angle 5π subtended is 6 (d) radius is 3 cm and angle subtended is 30° (e) radius is 7 mm and angle subtended is 45°. Find the arc length, correct to 2 decimal places, given (a) radius is 1.5 m and angle subtended is 0.43 (b) radius is 3.21 cm and angle subtended is 1.22 (c) radius is 7.2 mm and angle subtended is 55° (d) radius is 5.9 cm and angle subtended is 23° 12l (e) radius is 2.1 m and angle subtended is 82° 35l. The angle subtended at the centre of a circle of radius 3.4 m is 29° 51l. Find the length of the arc cut off by this angle, correct to 1 decimal place. The arc length when a sector of a circle is subtended by an angle of π 3π at the centre is m. Find the 5 2 radius of the circle.

5.

The radius of a circle is 3 cm and 2π an arc is cm long. Find the 7 angle subtended at the centre of the circle by the arc.

6.

The circumference of a circle is 300 mm. Find the length of the arc that is formed by an angle of π subtended at the centre of the 6 circle.

7.

A circle with area 60 cm2 has an arc 8 cm long. Find the angle that is subtended at the centre of the circle by the arc.

8.

A circle with circumference 124 mm has a chord cut off it that subtends an angle of 40° at the centre. Find the length of the arc cut off by the chord.

9.

A circle has a chord of 25 mm π with an angle of subtended 6 at the centre. Find, to 1 decimal place, the length of the arc cut off by the chord.

10. A sector of a circle with radius π 5 cm and an angle of 3 subtended at the centre is cut out of cardboard. It is then curved around to form a cone. Find its exact surface area and volume.

Chapter 5 Trigonometric Functions

Area of sector

1 2 r θ 2 (θ is in radians) A=

Proof angle θ area of sector A = whole revolution area of circle θ A = π r 2 2π θπr2 A= ` 2π 1 2 = r θ 2

EXAMPLES π 1. Find the area of the sector formed if an angle of is subtended at the 4 centre of a circle of radius 5 m.

Solution 1 2 r θ 2 π 2 1 = ^5h c m 4 2 25π 2 = m 8

A=

CONTINUED

181

182

Maths In Focus Mathematics HSC Course

6π cm2. Find the 5 angle, in degrees, that is subtended at the centre of the circle. 2. The area of the sector of a circle with radius 4 cm is

Solution 1 2 r θ 2 6π 1 ] g 2 = 4 θ 5 2 = 8θ 6π =θ 40 3π =θ 20 3 ] 180° g =θ 20 27° = θ A=

Remember that π = 180 °.

5.4 Exercises 1.

2.

Find the exact area of the sector of a circle if (a) radius is 4 cm and angle subtended is π (b) radius is 3 m and angle π subtended is 3 (c) radius is 10 cm and angle 5π subtended is 6 (d) radius is 3 cm and angle subtended is 30° (e) radius is 7 mm and angle subtended is 45°. Find the area of the sector, correct to 2 decimal places, given (a) radius is 1.5 m and angle subtended is 0.43 (b) radius is 3.21 cm and angle subtended is 1.22 (c) radius is 7.2 mm and angle subtended is 55° (d) radius is 5.9 cm and angle subtended is 23° 12l (e) radius is 2.1 m and angle subtended is 82° 35l.

3.

Find the area, correct to 3 significant figures, of the sector of a circle with radius 4.3 m and an angle of 1.8 subtended at the centre.

4.

The area of a sector of a circle is 20 cm2. If the radius of the circle is 3 cm, find the angle subtended at the centre of the circle by the sector.

5.

The area of the sector of a circle that is subtended by an angle of π at the centre is 6π m2. Find the 3 radius of the circle.

6.

Find the (a) arc length (b) area of the sector of a circle with radius 7 cm if the sector is cut off by an angle of 30° subtended at the centre of the circle.

Chapter 5 Trigonometric Functions

183

3π cm2 10 and the arc length cut off by the π sector is cm. Find the angle 5 subtended at the centre of the circle and find the radius of the circle.

10. The area of a sector is

7.

A circle has a circumference of 185 mm. Find the area of the π sector cut off by an angle of 5 subtended at the centre.

8.

If the area of a circle is 200 cm2 and a sector is cut off by an angle 3π of at the centre, find the area 4 of the sector.

9.

Find the area of the sector of a circle with radius 5.7 cm if the length of the arc formed by this sector is 4.2 cm.

Area of minor segment

A=

1 2 r (θ − sin θ ) 2

(θ is in radians)

Proof Area of minor segment = area of sector − area of triangle 1 2 1 r θ − r 2 sin θ 2 2 1 2 = r (θ − sin θ ) 2 =

The area of the triangle is 1 given by A = 2 ab sin C.

184

Maths In Focus Mathematics HSC Course

EXAMPLES π 1. Find the area of the minor segment formed if an angle of is 4 subtended at the centre of a circle of radius 5 m.

Solution A= = = = = =

1 2] r θ − sin θ g 2 π π 2 1 ^ 5 h c − sin m 4 4 2 25 π 1 e − o 2 4 2 25π 25 − 8 2 2 25π 25 2 − 4 8 25π − 50 2 2 m 8

2. An 80 cm piece of wire is bent into a circle shape, and a 10 cm piece of wire is joined to it to form a chord. Find the area of the minor segment cut off by the chord, correct to 2 decimal places.

Solution

Don’t round off the radius. Use the full value in your calculator’s display to find θ.

C = 2 π r = 80 80 ` r= 2π 40 = π Z 12.7

Chapter 5 Trigonometric Functions

a2 + b2 − c2 2ab 12.72 + 12.72 − 102 cos θ = 2 ] 12.7 g ] 12.7 g = 0.69 ` θ = 0.807 1 A = r 2 ] θ − sin θ g 2 2 1 = ] 12.7 g ] 0.807 − sin 0.807 g 2 Z 6.88 cm2 cos C =

5.5 Exercises 1.

Find the exact area of the minor segment of a circle if (a) radius is 4 cm and the angle subtended is π (b) radius is 3 m and the angle π subtended is 3 (c) radius is 10 cm and the angle 5π subtended is 6 (d) radius is 3 cm and the angle subtended is 30° (e) radius is 7 mm and the angle subtended is 45°.

2.

3.

Find the area of the minor segment correct to 2 decimal places, given (a) radius is 1.5 m and the angle subtended is 0.43 (b) radius is 3.21 cm and the angle subtended is 1.22 (c) radius is 7.2 mm and the angle subtended is 55° (d) radius is 5.9 cm and the angle subtended is 23° 12l (e) radius is 2.1 m and the angle subtended is 82° 35l. Find the area of the minor segment formed by an angle of 40° subtended at the centre of a circle with radius 2.82 cm, correct to 2 significant figures.

4.

Find the (a) exact arc length (b) exact area of the sector (c) area of the minor segment, to 2 decimal places π is subtended at 7 the centre of a circle with radius 3 cm. if an angle of

5.

The area of the minor segment 2π cut off by an angle of is 9 2 500 cm . Find the radius of the circle, correct to 1 decimal place.

6.

Find the (a) length of the chord, to 1 decimal place (b) length of the arc (c) area of the minor segment, to 2 decimal places π 6 subtended at the centre of a circle with radius 5 cm. cut off by an angle of

7.

A chord 8 mm long is formed by an angle of 45° subtended at the centre of a circle. Find (a) the radius of the circle (b) the area of the minor segment cut off by the angle, correct to 1 decimal place.

185

186

Maths In Focus Mathematics HSC Course

8.

9.

π is subtended at the 8 centre of a circle. If this angle 5π cm, find cuts off an arc of 4 (a) the exact area of the sector (b) the area of the minor segment formed, correct to 1 decimal place. An angle of

11.

A triangle OAB is formed where O is the centre of a circle of radius 12 cm, and A and B are endpoints of a 15 cm chord.

An arc, centre C, cuts side AC in D. Find the exact (a) length of arc BD (b) area of ABD (c) perimeter of BDC 12.

(a) Find the angle (θ ) subtended at the centre of the circle, in degrees and minutes. (b) Find the area of ΔOAB, correct to 1 decimal place. (c) Find the area of the minor segment cut off by the chord, correct to 2 decimal places. (d) Find the area of the major segment cut off by the chord, correct to 2 decimal places. 10. The length of an arc is 8.9 cm and the area of the sector is 24.3 cm2 when an angle of θ is subtended at the centre of a circle. Find the area of the minor segment cut off by θ, correct to 1 decimal place.

The dartboard above has a radius of 23 cm. The shaded area has a height of 7 mm. If a player must hit the shaded area to win, find (a) the area of the shaded part, to the nearest square centimetre (b) the percentage of the shaded part in relation to the whole area of the dartboard, to 1 decimal place (c) the perimeter of the shaded area.

Chapter 5 Trigonometric Functions

13. The hour hand of a clock is 12 cm long. Find the exact (a) length of the arc through which the hand would turn in 5 hours (b) area through which the hand would pass in 2 hours. 14.

(a) the exact perimeter of sector ABC (b) the approximate ratio of the area of the minor segment to the area of the sector. 15. A wedge is cut so that its cross-sectional area is a sector of a circle, radius 15 cm and π subtending an angle of at the 6 centre. Find (a) the volume of the wedge (b) the surface area of the wedge.

Arc BC subtends an angle of 100° at the centre A of a circle with radius 4 cm. Find

Small Angles When we use radians of small angles, there are some interesting results.

EXAMPLES 1. Find sin 0.0023.

Solution Make sure your calculator is in radian mode sin 0.0023 = 0.002299997972 2. Find tan 0.0023.

Solution tan 0.0023 = 0.002300004056 3. Find cos 0.0023.

Solution cos 0.0023 = 0.999997355

187

188

Maths In Focus Mathematics HSC Course

Use radians for the small angles. You could use a computer spreadsheet to find these trigonometric ratios.

Class Investigation Use the calculator to explore other small angles and their trigonometric ratios. What do you notice?

B x O

Can you see why?

In ΔOAC, OC Z 1 when x is small. Arc AB = x ` AC Z x when x is small

1

C A

(definition of a radian)

AC OC x Z 1 =x OA cos x = OC 1 Z 1 =1 sin x =

AC OA x Z 1 =x

tan x =

If x (radians) is a small angle then sin x Z x tan x Z x cos x Z 1 and sin x < x < tan x

Chapter 5 Trigonometric Functions

189

Proof Area ΔOAB = = = Area sector OAB = = = Area ΔOAC = = = =

1 2 1 2 1 2 1 2 1 2 1 2 1 2 1 2 1 2 1 2

ab sin C ´ 1 ´ 1 ´ sin x sin x r2 ´ 12 ´ x x bh ´ 1 ´ AC

c tan x =

´ 1 ´ tan x

AC m 1

tan x

Area ΔOAB < area sector OAB < area ΔOAC 1 1 1 sin x < x < tan x 2 2 2 ` sin x < x < tan x `

Class Investigation π by using your calculator. 2 π Remember that x is in radians. Does this work for x > ? 2

Check that sin x < x < tan x for 0 < x <

These results give the following result:

lim x"0

Proof As x " 0, sin x " x sin x ` x "1

sin x x =1

tan x Also lim = 1. x "0 x Can you see why?

190

Maths In Focus Mathematics HSC Course

EXAMPLES 1. Evaluate lim x"0

sin 7x x .

Solution lim x"0

7 (sin 7x) sin 7x x = lim x"0 7x sin 7x = 7 lim x"0 7x = 7 (1) =7

2. The planet of Alpha Zeta in a faraway galaxy has a moon with a diameter of 145 000 km. If the distance between the centre of Alpha Zeta and the centre of its moon is 12.8 ´ 106 km, find the angle subtended by the moon at the centre of the planet, in seconds.

Solution

The radius of the moon is 72 500 km.

1 ´ 145 000 2 = 72 500

r=

1 θ 2 72 500 tan α = 12.8 ´ 106 = 0.00566 ` α = 0.00566 π = 180° 180° ` 1= π 180° 0.00566 = π ´ 0.00566 = 0.3245° ` θ = 0.649° = 39l α=

Chapter 5 Trigonometric Functions

5.6 Exercises 1.

Evaluate, correct to 3 decimal places. (a) sin 0.045 (b) tan 0.003 (c) cos 0.042 (d) sin 0.065 (e) tan 0.005

2.

Evaluate lim x"0

tan

θ 3

Find lim

4.

Find the diameter of the sun to the nearest kilometre if its distance from the Earth is 149000000 km and it subtends an angle of 31l at the Earth.

θ

Given that the wingspan of an aeroplane is 30 m, find the plane’s altitude to the nearest metre if the wingspan subtends an angle of 14l when it is directly overhead.

sin x . 4x

3.

θ"0

5.

.

Trigonometric Graphs You drew the graphs of trigonometric functions in the Preliminary Course, using degrees. You can also draw these graphs using radians. y = sin x

191

192

Maths In Focus Mathematics HSC Course

y = cos x

y = tan x

π 3π and since tan x is undefined at those 2 2 points. By finding values for x on each side of the asymptotes, we can see where the curve goes. We can also sketch the graphs of the reciprocal trigonometric functions, by finding the reciprocals of sin x, cos x and tan x at important points on the graphs. y = tan x has asymptotes at x =

Investigation 1. Use cosec x =

1 1 1 , sec x = cos x and cot x = to complete the tan x sin x

table below. x

0

cosec x sec x cot x e.g. cosec

π 1 = 2 sin π 2 1 = 1 =1

π 2

π

3π 2



Chapter 5 Trigonometric Functions

2. Find any asymptotes e.g. sec

π 1 = 2 cos π 2 1 = (undefined) 0

Discover what the values are on either side of the asymptotes. 3. Sketch each graph of the reciprocal trigonometric functions.

Here are the graphs of the reciprocal ratios. y

y = cosec x

1 y = sin x

π 2

3π 2

π



-1

y y = sec x

1 y = cos x

π 2 -1

π

3π 2



x

x

193

194

Maths In Focus Mathematics HSC Course

y

y = tan x

π 2

π

3π 2



x

y = cot x

We have sketched these functions in the domain 0 ≤ x ≤ 2π which gives all the angles in one revolution of a circle. However, we could turn around the circle again and have angles greater than 2π , as well as negative angles. These graphs all repeat at regular intervals. They are called periodic functions.

Investigation Here is a general sine function. Notice that the shape that occurs between 0 and 2π repeats as shown. y

1

−4π

−2π





x

−1

1. Draw a general cosine curve. How are the sine and cosine curves related? Do these curves have symmetry? 2. Draw a general tangent curve. Does it repeat at the same intervals as the sine and cosine curves? 3. Look at the reciprocal trigonometric curves and see if they repeat in a similar way.

Chapter 5 Trigonometric Functions

4. Use a graphics calculator or a graphing computer program to draw the graphs of trigonometric functions. Choose different values of k to sketch these families of trig functions. (Don’t forget to look at values where k is a fraction or negative.) (a) f (x) = k sin x (b) f (x) = k cos x (c) f (x) = k tan x (d) f (x) = sin kx (e) f (x) = cos kx (f) f (x) = tan kx (g) f (x) = sin x + k (h) f (x) = cos x + k (i) f (x) = tan x + k (j) f (x) = sin (x + k) (k) f (x) = cos (x + k) (l) f (x) = tan (x + k) Can you see patterns in these families? Could you predict what the graph of f (x) = a sin bx looks like?

The trig functions have period and amplitude. The period is the distance over which the curve moves along the x-axis before it repeats. The amplitude is the maximum distance that the graph stretches out from the centre of the graph on the y-axis.

y = sin x has amplitude 1 and period 2π y = cos x has amplitude 1 and period 2π y = tan x has no amplitude and period π

y = a sin bx has amplitude a and period

2π b

2π b π y = a tan bx has no amplitude and period b y = a cos bx has amplitude a and period

195

196

Maths In Focus Mathematics HSC Course

EXAMPLES 1. Sketch in the domain 0 ≤ x ≤ 2π. (a) y = 5 sin x for (b) y = sin 4x for (c) y = 5 sin 4x for

Solution (a) The graph of y = 5 sin x has amplitude 5 and period 2π. y

5

y = 5 sin x

π 2

π

3π 2



x

−5

2π π (b) The graph y = sin 4x has amplitude 1 and period or . 4 2 π This means that the curve repeats every , so in the domain 2 0 ≤ x ≤ 2π there will be 4 repetitions. y

1

y = sin 4x

π 4

−1

π 2

3π 4

π

5π 4

3π 2

7π 4



x

Chapter 5 Trigonometric Functions

(c) The graph y = 5 sin 4x has amplitude 5 and period

197

π . 2

y

5

y = 5 sin 4x

π 4

π 2

π

3π 4

5π 4

3π 2

7π 4



x

−5

2. Sketch f (x) = sin b x +

π l for 0 ≤ x ≤ 2π. 2

Solution Amplitude = 1 2π Period = 1 = 2π f (x) = sin (x + k) translates the curve k units to the left (see Investigation on page 194–5). π π So f (x) = sin c x + m will be moved units to the left. The graph is the 2 2 same as f (x) = sin x but starts in a different position. If you are not sure where the curve goes, you can draw a table of values. x

0

π 2

π

3π 2



y

1

0

−1

0

1

y

1

π 2

π

3π 2



x

−1 This is the same as the graph y = cos x.

CONTINUED

198

Maths In Focus Mathematics HSC Course

3. Sketch y = 2 tan

x for 0 ≤ x ≤ 2π. 2

Solution There is no amplitude. π Period = 1 2 = 2π y

π

π 2

3π 2

x



4. (a) Sketch y = 2 cos x and y = cos 2x on the same set of axes for 0 ≤ x ≤ 2π. (b) Hence or otherwise, sketch y = cos 2x + 2 cos x for 0 ≤ x ≤ 2π.

Solution (a) y = 2 cos x has amplitude 2 and period 2π 2π y = cos 2x has amplitude 1 and period or π 2 y 2 y = 2 cos x

1 y = cos 2x 2

π 4 −1

−2

π 2

3π 4

π

5π 4

3π 2

7π 4



x

Chapter 5 Trigonometric Functions

199

(b) Method 1: Use table 0

π 4

π 2

3π 4

π

5π 4

3π 2

7π 4



cos 2x

1

0

−1

0

1

0

−1

0

1

2 cos x

2

2

0

− 2

−2

− 2

0

2

2

cos 2x + 2 cos x

3

2

−1

− 2

−1

− 2

−1

2

3

x

Notice that the period of this graph is 2π .

Method 2: Add y values together on the graph itself e.g. when x = 0: y=1+2 =3 y 3

y = cos 2x 2 + 2 cos x

2 y = 2cos x 1

y = cos 2x 2

π 4

π 2

3π 4

π

5π 4

3π 2

7π 4



x

−1 −2

These graphs can be sketched more accurately on a graphics calculator or in a computer program such as Autograph.

−3

5.7 Exercises 1.

Sketch for 0 ≤ x ≤ 2π (a) y = cos x (b) f (x) = 2 sin x (c) y = 1 + sin x (d) y = 2 − sin x (e) f (x) = − 3 cos x (f) y = 4 sin x (g) f (x) = cos x + 3 (h) y = 5 tan x (i) f (x) = tan x + 3 (j) y = 1 − 2 tan x

2.

Sketch for 0 ≤ x ≤ 2π (a) y = cos 2x (b) y = tan 2x (c) y = sin 3x (d) f (x) = 3 cos 4x (e) y = 6 cos 3x x (f) y = tan 2 (g) f (x) = 2 tan 3x x (h) y = 3 cos 2 x (i) y = 2 sin 2 x (j) f (x) = 4 cos 4

200

Maths In Focus Mathematics HSC Course

3.

4.

Sketch for − π ≤ x ≤ π (a) y = sin 2x (b) y = 7 cos 4x (c) f (x) = tan 4x (d) y = 5 sin 4x (e) f (x) = 2 cos 2x Sketch y = 8 sin

Sketch for − 2 ≤ x ≤ 2 (a) y = sin π x (b) y = 3 cos 2π x

7.

Sketch in the domain 0 ≤ x ≤ 2π (a) y = sin x and y = sin 2x on the same set of axes (b) y = sin x + sin 2x

8.

Sketch for 0 ≤ x ≤ 2π (a) y = 2 cos x and y = 3 sin x on the same set of axes (b) y = 2 cos x + 3 sin x

9.

By sketching y = cos x and y = cos 2x on the same set of axes for 0 ≤ x ≤ 2π , sketch the graph of y = cos 2x − cos x.

x in the domain 2

0 ≤ x ≤ 4π. 5.

6.

Sketch for 0 ≤ x ≤ 2π (a) y = sin (x + π ) π (b) y = tan b x + l 2 (c) f (x) = cos (x − π ) π (d) y = 3 sin b x − l 2 π (e) f (x) = 2 cos b x + l 2 π (f) y = 4 sin b 2x + l 2 π (g) y = cos b x − l 4 π (h) y = tan b x + l 4 π (i) f (x) = 2 cos b x + l + 1 2 π (j) y = 2 − 3 sin b x – l 2

10. Sketch the graph of (a) y = cos x + sin x (b) y = sin 2x − sin x (c) y = sin x + 2 cos 2x (d) y = 3 cos x − cos 2x x (e) y = sin x − sin 2

Applications The sine and cosine curves are used in many applications including the study of waves. There are many different types of waves, including water, light and sound waves. Oscilloscopes display patterns of electrical waves on the screen of a cathode-ray tube. Search waves and the oscilloscope on the Internet for further information. Simple harmonic motion (such as the movement of a pendulum) is a wave-like or oscillatory motion when graphed against time. In 1581, when he was 17 years old, Gallileo noticed a lamp swinging backwards and forwards in Pisa cathedral. He found that the lamp took the same time to swing to and fro, no matter how much weight it had on it. This led him to discover the pendulum. Gallileo also invented the telescope. Find out more information about Gallileo’s life and his other discoveries.

Chapter 5 Trigonometric Functions

Some trigonometric equations are difficult to solve algebraically, but can be solved graphically by finding the points of intersection between two graphs.

EXAMPLES 1. Find approximate solutions to the equation sin x = x − 1 by sketching the graphs y = sin x and y = x − 1 on a Cartesian plane.

Solution When sketching the two graphs together, we use π ≈ 3.14, 2π ≈ 6.28 and so on to label the x-axis as shown. To sketch y = x − 1, find the gradient and y-intercept or find the x- and y-intercepts. x-intercept (where y = 0) is 1 and y-intercept (where x = 0) is –1. y y = x−1

1

1

π 2

2

3

π

4

3π 5 2

−1

6



x

y = sin x

The solution to sin x = x − 1 is at the point of intersection of the two graphs. ` x ≈ 2. 2. How many solutions are there for cos 2x =

x in the domain 0 ≤ x ≤ 2π ? 4

Solution x on the same set of axes. 4 y = cos 2x has amplitude 1 and period π x y = has x-intercept 0 and y-intercept 0. We can find another point on the line 4 e.g. When x = 4 4 y= 4 =1 Sketch y = cos 2x and y =

CONTINUED

201

Maths In Focus Mathematics HSC Course

y y=

x 4

1

1

π 2

2

3

π

4

3π 5 2

6



y = cos 2x 2 −1

There are 3 points of intersection of the graphs, so the equation x cos 2x = has 3 solutions. 4

We can also look at applications of trigonometric graphs in real life situations.

EXAMPLE The table shows the highest average monthly temperatures in Sydney. Jan °C

Feb

26.1 26.1

Mar

Apr

May

25.1

22.8 19.8

Jun

Jul

Aug

17.4

16.8 18.0

Sep

Oct

20.1 22.2

Nov

Dec

23.9

25.6

(a) Draw a graph of this data, by hand or on a graphics calculator or computer. (b) Is it periodic? Why would you expect it to be periodic? (c) What is the period and amplitude?

Solution (a)

Temperature 30 25 20 15 10 5 0

Ja nu Fe ary br ua r M y ar ch A pr il M ay Ju ne Ju A ly Se ugu pt st em b O er c N tob ov er e D mbe ec em r be r

202

Months

Chapter 5 Trigonometric Functions

203

(b) The graph looks like it is periodic, and we would expect it to be, since the temperature varies with the seasons. It goes up and down, and reaches a maximum in summer and a minimum in winter. (c) This curve is approximately a cosine curve with one full period, so the period is 12 months. The maximum temperature is around 26° and the minimum is around 18°, so the centre of the graph is 22° with 4° either side. So the amplitude is 4.

5.8 Exercises Show graphically that x sin x = has 2 (a) 2 solutions for 0 ≤ x ≤ 2π (b) 3 solutions for − π ≤ x ≤ π .

Solve cos x = 2x − 3 for 0 ≤ x ≤ 2π by finding the points of intersection of the graphs y = cos x and y = 2x − 3.

4.

Solve tan x = x graphically in the domain 0 ≤ x ≤ 2π .

5.

Solve by graphical means sin 2x = x for 0 ≤ x ≤ 2π .

6.

Draw the graphs of y = sin x and y = cos x for 0 ≤ x ≤ 2π on the same set of axes. Use your graphs to solve the equation sin x = cos x for 0 ≤ x ≤ 2π .

10 9 8 7 6 5 4 3 2 1 0 y M ar ch M ay Se Jul y pt em N be ov r em b Ja er nu ar y M ar ch M ay Se Jul y pt em N be ov r em be r

3.

Sunsets

Time (pm)

Solve sin x = x for 0 ≤ x ≤ 2π graphically by sketching y = sin x and y = x on the same number plane.

The graph below show the times of sunsets in a city over a period of 2 years.

ar

2.

7.

Ja nu

1.

(a) Find the period and amplitude of the graph. (b) At approximately what time would you expect the sun to set in July?

To find the centre, find the average of 18 and 26 which 18 + 26 . is 2

204

Maths In Focus Mathematics HSC Course

8.

The graph shows the incidence of crimes committed over 24 years in Gotham City. 1800 1600 1400 1200 1000 800 600 400 200 0 1 2 3 4 5 6 7 8 9 10 11 12 13 14 15 16 17 18 19 20 21 22 23 24

(a) Approximately how many crimes were committed in the 10th year? (b) What was the (i) highest and (ii) lowest number of crimes? (c) Find the amplitude and the period of the graph. 9.

Below is a table showing the average daylight hours over several months. Month

Jan

Feb

Mar

Apr

May June July

Daylight hours

15.3 14.7 13.2 13.1 12.7 12.2

Aug

12.5 13.8

(a) Draw a graph to show this data. (b) Is it periodic? If so, what is the period? (c) Find the amplitude. 10. The table below shows the high and low tides over a three-day period. Day

Friday

Saturday

Sunday

Time

6.20

11.55 6.15

11.48 6.20 11.55 6.15 11.48

6.20

11.55

6.15

11.48

am

am

pm

pm

am

am

pm

pm

am

am

pm

pm

Tide (m) 3.2

1.1

3.4

1.3

3.2

1.2

3.5

1.1

3.4

1.2

3.5

1.3

(a) Draw a graph showing the tides. (b) Find the period and amplitude. (c) Estimate the height of the tide at around 8 am on Friday.

Chapter 5 Trigonometric Functions

205

Differentiation of Trigonometric Functions Derivative of sin x We can sketch the gradient (tangent) function of y = sin x.

The gradient function is y = cos x.

d (sin x) = cos x dx

Proof Let f ] x g = sin x Then f (x + h) = sin (x + h) f (x + h) − f (x) f l(x) = lim h"0 h sin (x + h) − sin x = lim h"0 h (sin x cos h + cos x sin h) − sin x = lim h"0 h sin x (cos h − 1) + cos x sin h = lim h"0 h cos h − 1 sin h m + cos x c m = lim sin x c h"0 h h

This result is a formula in the Extension 1 Course and you do not need to know it.

= lim sin x(0) + cos x(1) c since cos h " 1 as h " 0 and lim h"0

h"0

= cos x. Notice that the result lim h"0

radians when using calculus.

sin h = 1m h

sin h = 1 only works for radians. We always use h

206

Maths In Focus Mathematics HSC Course

FUNCTION OF A FUNCTION RULE d [sin f (x)] = f l(x) cos f (x) dx

Proof Let u = f (x) Then y = sin u dy du = f l(x) and = cos u dx du dy dy du = $ dx du dx = cos u $ f l(x) = f l(x) cos f (x)

Derivative of cos x We can sketch the gradient function of y = cos x.

The gradient function is y = − sin x.

d (cos x) = − sin x dx

Proof π − xm 2 π d d < sin c − x m F (cos x) = 2 dx dx π = − 1 ´ cos c − x m 2 = − sin x

Since cos x = sin c

Chapter 5 Trigonometric Functions

207

FUNCTION OF A FUNCTION RULE d [cos f (x)] = − f l(x) sin f (x) dx

Derivative of tan x The gradient function is harder to sketch for y = tan x.

The gradient function is y = sec2 x.

sin x It is easier to see the rule for differentiating y = tan x by using tan x = cos x and the quotient rule.

d (tan x) = sec2 x dx

Proof sin x tan x = cos x d d b sin x l (tan x) = dx dx cos x ulv − vlu = v2 cos x (cos x) − (−sin x) sin x = cos2 x cos2 x + sin2 x = cos2 x 1 = cos2 x = sec2 x

This result can be proven the same way as for sin f(x).

208

Maths In Focus Mathematics HSC Course

FUNCTION OF A FUNCTION RULE d [tan f (x)] = f l(x) sec2 f (x) . dx

This result can be proven the same way as for sin f(x).

EXAMPLES 1. Differentiate sin (5x) .

Solution d [sin f (x)] = f l(x) cos f (x) dx d ` [sin (5x)] = 5 cos (5x) dx 2. Differentiate sin x°.

Solution πx d d c sin m (sin x°) = 180 dx dx π πx = cos 180 180 π = cos x° 180 3. Find the exact value of the gradient of the tangent to the curve π y = x2 sin x at the point where x = . 4

Solution dy dx

= ulv + vlu = cos x (x2) + 2x (sin x) = x2 cos x + 2x sin x π , 4 dy π 2 π π π = c m cos + 2 c m sin 4 4 4 4 dx 2 π π 1 1 = ´ + ´ 16 2 2 2 2 π π = + 16 2 2 2 π 2 + 8π = 16 2 π 2 (π + 8) = 32

When x =

Chapter 5 Trigonometric Functions

5.9 Exercises 1.

Differentiate (a) sin 4x (b) cos 3x (c) tan 5x (d) tan (3x + 1) (e) cos (− x) (f) 3 sin x (g) 4 cos (5x − 3) (h) 2 cos (x3) (i) 7 tan (x2 + 5) (j) sin 3x + cos 8x (k) tan (π + x) + x 2 (l) x tan x (m) sin 2x tan 3x sin x (n) 2x 3x + 4 (o) sin 5x (p) (2x + tan 7x)9 (q) sin2 x (r) 3 cos3 5x (s) ex − cos 2x (t) sin (1 − loge x) (u) sin (ex + x) (v) loge (sin x) (w) e3x cos 2x e2x (x) tan 7x

5.

Differentiate log e (cos x) .

6.

Find the exact gradient of the normal to the curve y = sin 3x at π the point where x = . 18

7.

Differentiate etan x.

8.

Find the equation of the normal to the curve y = 3 sin 2x at the point where x =

9.

Show that

d2 y

dx2 y = 2 cos 5x.

π , in exact form. 8

= − 25y if

10. Given f (x) = − 2 sin x, show that f m(x) = − f (x) . 11. Show that d [loge (tan x)] = tan x + cot x. dx 12. Find the coordinates of the stationary points on the curve y = 2 sin x − x for 0 ≤ x ≤ 2π .

2.

Find the derivative of cos x sin4 x.

3.

Find the gradient of the tangent to the curve y = tan 3x at the π point where x = . 9

4.

Find the equation of the tangent to the curve y = sin (π − x) at the π 1 point c , m , in exact form. 6 2

13. Differentiate (a) tan x° (b) 3 cos x° sin x° (c) 5 14. If y = 2 sin 3x − 5 cos 3x, show d2 y that = − 9 y. dx2 15. Find values of a and b if d2 y = ae3x cos 4x + be3x sin 4x, dx2 given y = e3x cos 4x.

Integration of Trigonometric Functions The integrals of the trigonometric functions are their primitive functions.

# cos x dx = sin x + C

209

210

Maths In Focus Mathematics HSC Course

Integration is the inverse of differentiation.

# − sin x dx = cos x + C ` # sin x dx = − cos x + C # sin x dx = − cos x + C # sec2 x dx = tan x + C Function of a function rule

# cos (ax + b) dx = 1a sin (ax + b) + C Proof d [sin (ax + b)] = a cos (ax + b) dx

`

# a cos (ax + b) dx = sin (ax + b) + C # cos (ax + b) dx = 1a # a cos (ax + b) dx 1 = a sin (ax + b) + C Similarly,

# sin (ax + b) dx = − 1a cos (ax + b) + C

# sec2 (ax + b) dx = 1a tan(ax + b) + C

EXAMPLES 1. Find

# sin (3x) dx.

Solution

# sin (3x) dx = − 13 cos (3x) + C

Chapter 5 Trigonometric Functions

2. Evaluate

#

π 2

211

sin x dx.

0

Solution

#

0

π 2

sin x dx = ; − cos x E

π 2 0

π = − cos − (− cos 0) 2 = 0 − ( − 1) =1 3. Find # cos x° dx.

Solution πx dx # cos x° dx = # cos 180 πx 1 π sin 180 + C 180 180 = π sin x° + C =

4. Find the area enclosed between the curve y = cos x, the x-axis and the π 3π . lines x = and x = 2 2

Solution

You studied areas in Chapter 3. The definite integral is negative as the area is below the x-axis.

#

π 2

3π 2

3π 2

cos x dx = ; sin x E π

2

You could use

3π π = sin − sin 2 2 = −1 − 1 = −2

Area =

#

π 2

= −2 = 2.

` area is 2 units2.

CONTINUED

3π 2

cos x dx

212

Maths In Focus Mathematics HSC Course

5. Find the volume of the solid formed if the curve y = sec x is rotated π about the x-axis from x = 0 to x = . 4

Solution

y = sec x ` y2 = sec2 x V=π

The volume formula comes from Chapter 3.

#

b

#

π 4

y 2 dx

a



sec 2 x dx

0

= π ; tan x E

π 4 0

π = π c tan − tan 0 m 4 = π (1 − 0) =π So the volume is π units 3 .

5.10 Exercises 1.

Find the indefinite integral (primitive function) of (a) cos x (b) sin x (c) sec2 x sin x° (d) 4 (e) sin 3x (f) − sin 7x (g) sec2 5x (h) cos (x + 1) (i) sin (2x − 3) (j) cos (2x − 1) (k) sin (π − x) (l) cos (x + π ) (m) 2 sec2 7x

(n) 4 sin

x 2

(o) 3 sec2

x 3

(p) − sin (3 − x) 2.

Evaluate, giving exact answers where appropriate. (a)

# #

π 2

cos x dx

0

(b)

π 3

π 6

(c)

#

(d)

#

π

π 2

0

π 2

sec 2 x dx sin

x dx 2

cos 3x dx

Chapter 5 Trigonometric Functions

#

1 2

# (g) # (h) #

π 8

(e)

by rotating the curve y = sin 2x about the x-axis from x = 0 to π x= . 6

sin π x dx

0

(f)

sec 2 2x dx

0 π 12

8.

Find the exact area enclosed by the curve y = sin x and the line 1 y = for 0 ≤ x ≤ 2π . 2

9.

Find the exact area bounded by the curves y = sin x and y = cos x in the domain 0 ≤ x ≤ 2π .

3 cos 2x dx

0 π 10

− sin 5x dx

0

3.

Find the area enclosed between the curve y = sin x and the x-axis in the domain 0 ≤ x ≤ 2π .

4.

Find the exact area bounded by the curve y = cos 3x, the x-axis π and the lines x = 0 and x = . 12

5.

Find the area enclosed between x the curve y = sec2 , the x-axis 4 π π and the lines x = and x = , 4 2 correct to 2 decimal places.

6.

7.

Find the volume, correct to 2 decimal places, of the solid formed when the curve y = sec π x is rotated about the x-axis from x = 0 to x = 0.15. Find, in exact form, the volume of the solid of revolution formed

10. (a) Show that the volume of the solid formed by rotating the curve y = cos x about the π x axis between x = 0 and x = is 2 π units3 . (b) Use the trapezoidal rule with 4 subintervals to find an approximation to the volume of the solid formed by rotating the curve y = cos x about the x-axis π from x = 0 to x = . 2 d 2y

= 18 sin 3x and a dx2 π stationary point at c , − 2 m . Find 6 the equation of the curve.

11. A curve has

213

214

Maths In Focus Mathematics HSC Course

Test Yourself 5 4 1.

A circle with radius 5 cm has an angle of π subtended at the centre. Find 6 (a) the exact arc length (b) the exact area of the sector (c) the area of the minor segment to 3 significant figures.

2.

Find the exact value of π (a) tan 3 π (b) cos 6 2π (c) sin 3 3π (d) cos 4

3.

Solve for 0 ≤ x ≤ 2π (a) tan x = − 1 (b) 2 sin x = 1

4.

Sketch for 0 ≤ x ≤ 2π (a) y = 3 cos 2x x (b) y = 7 sin 2

5.

6.

Differentiate (a) cos x (b) 2 sin x (c) tan x + 1 (d) x sin x tan x (e) x (f) cos 3x (g) tan 5x Find the indefinite integral (primitive function) of (a) sin 2x (b) 3 cos x (c) sec2 5x (d) 1 + sin x

7.

Evaluate (a)

# #

π 4

cos x dx

0

(b)

π 3

π 6

sec 2 x dx

8.

Find the equation of the tangent to the π 1 curve y = sin 3x at the point e , o. 4 2 d2 x 9. If x = cos 2t, show that = − 4x. dt2 10. Find the exact area bounded by the curve π y = sin x, the x-axis and the lines x = 4 π and x = . 2 11. Find the volume of the solid formed if the curve y = sec x is rotated about the π x-axis from x = 0 to x = . 6 12. Simplify (a) lim

sin 5x x

(b) lim

2 tan θ θ

x"0

θ"0

13. Find the gradient of the tangent to the curve y = 3 cos 2x at the point where π x= . 6 14. A circle has a circumference of 8π cm. If π an angle of is subtended at the centre 7 of the circle, find (a) the exact area of the sector (b) the area of the minor segment, to 2 decimal places. 2x on the 3 same set of axes for 0 ≤ x ≤ 2π . 2x (b) Solve cos 2x = for 0 ≤ x ≤ 2π . 3

15. (a) Sketch y = cos 2x and y =

Chapter 5 Trigonometric Functions

16. Find the exact area with rational denominator bounded by the curve

19. A curve has

= 6 sin 2x, and passes

π through the point c , 3 m . Find the 2 equation of the curve.

π y = sin x, the x-axis and the lines x = 6 π and x = . 4 17. Find the area bounded by the curve y = cos 2x, the x-axis and the lines x = 0 to x = π .

dy dx

20. (a) Sketch y = 7 sin 3x and y = 2x − 1 on the same number plane for 0 ≤ x ≤ 2π . (b) Solve 7 sin 3x = 2x − 1 for 0 ≤ x ≤ 2π .

18. Find the equation of the normal to the π curve y = tan x at the point c , 1 m . 4

Challenge Exercise 5 1.

Use Simpson’s rule with 5 function values to find an approximation to

#

π 8

2. 3.

4.

5.

6.

π 4

tan x dx, correct to 2 decimal places.

Evaluate

#

π 8

π 12

7.

Find the derivative of tan x°.

8.

(a) Show that sec x cosec x =

sec 2 2x dx, in exact form.

The area of the sector of a circle is 4π units2 and the length of the arc π bounded by this sector is units. Find 8 the radius of the circle and the angle that is subtended at the centre. If f (x) = 3 cos π x (a) find the period and amplitude of the function (b) sketch f (x) for 0 ≤ x ≤ 4. d2 y Given = 9 sin 3x dx2 (a) find y if there is a stationary point at π c , 1m 2 d2 y (b) Show that + 9y = 0 . dx2 Sketch y = 5 sin ] x + π g for 0 ≤ x ≤ 2π .

sec2 x . tan x (b) Hence, or otherwise, find the exact

value of 9.

#

π 4

π 3

cosec x sec x dx.

Differentiate ex sin 2x.

10. (a) Find the stationary points on the curve y = sin 2x + 3 over the domain 0 ≤ x ≤ π. (b) What is the maximum value of the curve? (c) What is the amplitude? 11. The area of a sector in a circle of radius 8π cm2. Find the area of the 4 cm is 3 minor segment, in exact form. 12. Find

# sin x°dx.

13. Find the gradient of the normal to the curve y = x2 + cos π x at the point where x = 1.

215

216

Maths In Focus Mathematics HSC Course

14. Use the trapezoidal rule with 4 subintervals to find, correct to 3 decimal places, an approximation to the volume of the solid formed by rotating the curve y = sin x about the x-axis from x = 0.2 to x = 0.6. 15. Differentiate log e (sin x + cos x) . 16. Find the exact area of the minor segment cut off by a quarter of a circle with radius 3 cm.

17. Sketch y = tan c x −

π m for 0 ≤ x ≤ 2π . 4

18. Find all the points of inflexion on the π curve y = 3 cos c 2x + m for 0 ≤ x ≤ 2π . 4 19. Find the exact area bounded by the curve y = cos x, the x-axis and the lines π π x = and x = . 4 6 20. If f (x) = 2 cos 3x, show that f m(x) = − 9 f (x) .

6

Applications of Calculus to the Physical World TERMINOLOGY Acceleration: The rate of change of velocity with respect to time At rest: Stationary (zero velocity) Displacement: The movement of an object in relation to its original position Exponential growth: Growth, or increase in a quantity, where the rate of change of a quantity is in direct proportional to the quantity itself. The growth becomes more rapid over time

Exponential decay: Decay, or decrease in a quantity, where the rate of change of a quantity is in direct proportion to the quantity itself. The decay becomes less rapid over time Rate of change: The change of one variable with respect to another variable over time Velocity: The rate of change of displacement of an object with respect to time involving speed and direction

Chapter 6 Applications of Calculus to the Physical World

INTRODUCTION CALCULUS IS USED IN many situations involving rates of change, such as

physics or economics. This chapter looks at rates of change of a particle in motion, and exponential growth and decay. Both these types of rates of change involve calculus.

DID YOU KNOW? Galileo (1564–1642) was very interested in the behaviour of bodies in motion. He dropped stones from the Leaning Tower of Pisa to try to prove that they would fall with equal speed. He rolled balls down slopes to prove that they move with uniform speed until friction slows them down. He showed that a body moving through the air follows a curved path at a fairly constant speed. John Wallis (1616–1703) continued this study with his publication Mechanica, sive Tractatus de Motu Geometricus. He applied mathematical principles to the laws of motion and stimulated interest in the subject of mechanics. Soon after Wallis’ publication, Christiaan Huygens (1629−1695) wrote Horologium Oscillatorium sive de Motu Pendulorum, in which he described various mechanical principles. He invented the pendulum clock, improved the telescope and investigated circular motion and the descent of heavy bodies. These three mathematicians provided the foundations of mechanics. Sir Isaac Newton (1642–1727) used calculus to increase the understanding of the laws of motion. He also used these concepts as a basis for his theories on gravity and inertia.

Rates of Change Simple rates of change The gradient of a straight line measures the rate of change of y with respect to the change in x.

With a curve, the rate of change of y with respect to x is measured by the gradient of the tangent to the curve. That is, the derivative measures the rate of change.

Galileo

219

220

Maths In Focus Mathematics HSC Course

The rate of change of a quantity Q with respect to time t is measured by

dQ . dt

EXAMPLES 1. The number of bacteria in a culture increases according to the formula B = 2t 4 − t 2 + 2000, where t is time in hours. Find (a) the number of bacteria initially (b) the number of bacteria after 5 hours (c) the rate at which the number of bacteria will be increasing after 5 hours.

Solution (a)

B = 2t 4 − t 2 + 2000 Initially, t = 0 `

B = 2 (0) 4 − 0 2 + 2000 = 2000

So there are 2000 bacteria initially. (b) When t = 5, B = 2 (5)4 − 52 + 2000 = 3225 So there will be 3225 bacteria after 5 hours. (c) The rate of change is given by the derivative dB = 8t 3 − 2t dt When t = 5, dB = 8 (5)3 − 2 (5) dt = 990 So the rate of increase after 5 hours will be 990 bacteria per hour. 2. The volume flow rate of water into a pond is given by R = 4 + 3t 2 litres per hour. If there is initially no water in the pond, find the volume of water after 12 hours.

Chapter 6 Applications of Calculus to the Physical World

Solution

i.e.

R = 4 + 3t 2 dV = 4 + 3t 2 dt

Volume flow rate is the rate of change of volume over time.

V = # (4 + 3t 2) dt = 4t + t 3 + C When t = 0, V = 0 ` 0 =0+0+C =C

`

` V = 4t + t 3 When t = 12 V = 4 (12) + (12) 3 = 1776 L So there will be 1776 L of water in the pond after 12 hours.

6.1 Exercises 1.

2.

Find a formula for the rate of change in each question. (a) h = 20t − 4t 2 (b) D = 5t 3 + 2t 2 + 1 (c) A = 16x − 2x 2 (d) x = 3t 5 − x 4 + 2x − 3 (e) V = e t + 4 (f) S = 3 cos 5θ 50 (g) S = 2π r + 2 r 2 (h) D = x − 4 400 (i) S = 800r + r 4 (j) V = π r3 3 Find an original formula for each question given its rate of change. (a) R = 4t − 12t 2 is the rate at which the height h of an object changes over time t (b) R = 8x3 + 1 is the rate at which the area A of a figure changes with side x

(c) R = 4π r2 is the rate at which the volume V changes with radius r (d) R = 7 sin t is the rate at which the distance d of an object changes over time t (e) R = 8e 2t − 3 is the rate at which the speed s changes over time t. 3.

If h = t 3 − 7t + 5, find the rate of change of h when t = 3.

4.

Given f (t) = sin 2t, find the rate of π change when t = . 6

5.

A particle moves so that its distance x is given by x = 2e 3t . Find the exact rate of change when t = 4.

6.

The volume of water flowing through a pipe is given by V = t2 + 3t. Find the rate at which the water is flowing when t = 5.

221

222

Maths In Focus Mathematics HSC Course

7.

The rate of change of the angle sum S of a polygon with n sides is a constant 180. If S is 360 when n = 4, find S when n = 7.

8.

A particle moves so that the rate of change of distance D over time t is given by R = 2e t − 1. If D = 10 when t = 0, find D when t = 3.

9.

For a certain graph, the rate of change of y values with respect to its x values is given by R = 3x 2 − 2x + 1. If the graph passes through the point (−1, 3), find its equation.

10. The mass in grams of a melting iceblock is given by the formula M = t − 2t 2 + 100, where t is time in minutes. Show that the rate at which the iceblock is melting is given by R = 1 − 4t grams per minute and find the rate at which it will be melting after 5 minutes. 11. The rate of change in velocity over time is given by dy = 4t + t 2 − t 3. If the initial dt velocity is 2 cms − 1, find the velocity after 15 s. 12. The rate of flow of water into a dam is given by R = 500 + 20t Lh− 1 . If there is 15 000 L of water initially in the dam, how much water will there be in the dam after 10 hours? 13. The surface area in cm2 of a balloon being inflated is given by S = t3 − 2t2 + 5t + 2, where t is time in seconds. Find the rate of increase in the balloon’s surface area after 8 s.

14. According to Boyle’s Law, the pressure of a gas is given by k the formula P = , where k is a V constant and V is the volume of the gas. If k = 100 for a certain gas, find the rate of change in the pressure when V = 20. 15. A circular disc expands as it is heated. The area, in cm2, of the disc increases according to the formula A = 4t 2 + t, where t is time in minutes. Find the rate of increase in the area after 5 minutes. 16. A balloon is inflated so that its increase in volume is at a constant rate of 10 cm 3 s − 1 . If its volume is initially 1 cm3, find its volume after 3 s. 17. The number of people in a certain city is given by P = 200 000e 0.2t , where t is time in years. Find the rate at which the population of the city will be growing after 5 years. 18. A radioactive substance has a mass that decreases over time t according to the formula M = 200e − 0.1t . Find (a) the mass of the substance after 20 years, to the nearest gram (b) the rate of its decrease after 20 years. 19. If y = e 4x, show that the rate is given by R = 4y. 20. Given S = 2e 2t + 3, show that the rate of change is R = 2 (S − 3).

Chapter 6 Applications of Calculus to the Physical World

223

Exponential Growth and Decay Exponential growth or decay are terms that describe a special rate of change that occurs in many situations. Population growth and growth of bacteria in a culture are examples of exponential growth. The decay of radioactive substances, the cooling of a substance and change in pressure are examples of exponential decay. When a substance grows or decays exponentially, its rate of change is directly proportional to the amount of the substance itself. That is, the more of the substance there is, the faster it grows. For example, in a colony of rabbits, there is fairly slow growth in the numbers at first, but the more rabbits there are, the more rabbits will be born. In other species, such as koalas, the clearing of habitats and other factors may cause the population to decrease, or decay, exponentially.

The exponential function is a function where the rate of change (gradient) is increasing as the function increases.

This is why it is called exponential growth.

In mathematical symbols, exponential growth or decay can be written as dQ = kQ , where k is the growth or decay constant. dt

224

Maths In Focus Mathematics HSC Course

dQ = kQ is solved by the equation Q = Ae kt dt

When k is positive there is exponential growth. When k is negative there is exponential decay.

Proof dQ = kQ dt dt 1 ` = dQ kQ 1 t=# dQ kQ 1 = log e Q + k 1 k kt = log e Q + k 2 kt − k 2 = log e Q `

Q = e kt − k = e kt ´ e − k = Ae kt 2

2

A is always the initial quantity

Proof Initial means ‘at first’ when t = 0.

Given Q = Ae kt , suppose the initial quantity is Q 0 i.e. Q = Q 0 when t = 0 Substituting into the equation gives Q 0 = Ae k ´ 0 = Ae 0 =A ∴ A is the initial quantity. Sometimes the equation is written as Q = Q 0 e kt .

DID YOU KNOW? Malthus was an economist.

Thomas Malthus (1766–1834), at the beginning of the Industrial Revolution, developed a theory about population growth that we still use today. His theory states that under ideal conditions, dN

the birth rate is proportional to the size of the population. That is, = kN (Malthusian Law of dt Population Growth). Malthus was concerned that the growth rate of populations would be higher than the increase in food supplies, and that people would starve. Was he right? Is this happening? How could we prove this?

Chapter 6 Applications of Calculus to the Physical World

EXAMPLES 1. The population of a colony of rabbits over time t months is given by P = 1500e 0.046t . Find (a) the initial population (b) the population after 2 years (c) when the population reaches 5000.

Solution (a) Initially, t = 0 Substituting: P = 1500e 0.046t = 1500e 0.046 ´ 0 = 1500e 0 = 1500 (e 0 = 1) So the initial population is 1500 rabbits. (b) 2 years = 24 months So t = 24 Substituting: P = 1500e 0.046t = 1500e 0.046 ´ 24 = 1500e 1.104 = 4524.31 So the population after 2 years is 4524 rabbits. (c) P = 5000 Substituting: P = 1500e 0.046t 5000 = 1500e 0.046t 5000 = e 0.046t 1500 5000 = ln e 0.046t 1500 = 0.046t ln e = 0.046t 5000 ln 1500 =t 0.046 26.2 = t ln

(since ln e = 1)

So the population reaches 5000 after 26.2 months.

CONTINUED

225

226

Maths In Focus Mathematics HSC Course

2. The number of bacteria in a culture is given by N = Ae kt . If 6000 bacteria increase to 9000 after 8 hours, find (a) k correct to 3 significant figures (b) the number of bacteria after 2 days (c) the rate at which the bacteria will be increasing after 2 days (d) when the number of bacteria will reach 1 000 000 (e) the growth rate per hour as a percentage.

Solution (a) A gives the initial number of bacteria.

N = Ae kt When t = 0, N = 6000 `

6000 = Ae 0 =A

So N = 6000e kt When t = 8, N = 9000 9000 9000 6000 1 .5 log e 1.5

Don't round k off, but put it in the calculator’s memory to use again.

= 6000e 8k = e 8k = e 8k = log e e 8k = 8k log e e = 8k

log e 1.5

=k 8 0.0507 Z k

So N = 6000e 0.0507t . (b) When t = 48, (48 hours in 2 days) N = 6000e 0.0507 # 48 = 68 344 So there will be 68 344 bacteria after 2 days. dN (c) Rate: = 6000 (0.0507e 0.0507t ) dt = 304.1e 0.0507t When The rate of growth is the derivative.

t = 48, dN = 304.1e 0.0507 # 48 dt = 3464

So after 2 days the rate of growth will be 3464 bacteria per hour. Another method: dN = kN dt = 0.0507N When t = 48, N = 68 344 dN = 0.0507 ´ 68 344 = 3464 dt

Chapter 6 Applications of Calculus to the Physical World

(d)

227

When N = 1 000 000 1 000 000 1 000 000 6000 166.7 log e 166.7

= 6000e 0 .0507t = e 0.0507t = e 0.0507t = log e e 0.0507t = 0.0507t log e e = 0.0507t

log e 166.7

=t 0.0507 100.9 Z t

So the number of bacteria will be 1 000 000 after 100.9 hours. (e) dN = kN where k = growth constant dt k = 0.0507 = 5.07% So the growth rate is 5.07% per hour.

3. A 50 g mass of uranium decays to 35 g after 2 years. If the rate of decay of its mass is proportional to the mass itself, find the amount of uranium left after 25 years.

Solution Q = Ae kt When t = 0, Q = 50 `

50 = Ae 0 =A

So Q = 50e kt When t = 2, Q = 35 35 35 50 0 .7 log e 0.7

= 50e 2k = e 2k = e 2k = log e e 2k = 2k log e e = 2k

log e 0.7

=k 2 − 0.1783 Z k So Q = 50e − 0.1783t When t = 25 Q = 50e − 0.1783 # 25 Z 0.579 So there will be 0.579 grams, or 579 mg, left after 25 years.

Notice that k is negative for decay. You could use the formula Q = Ae − kt for decay. What would k be?

228

Maths In Focus Mathematics HSC Course

6.2 Exercises 1.

2.

3.

The number of birds in a colony is given by N = 80e 0.02t where t is in days. (a) How many birds are there in the colony initially? (b) How many birds will there be after 30 days? (c) After how many days will there be 500 birds? (d) Sketch the curve of the population over time.

The number of bacteria in a culture is given by N = N 0 e 0.32t , where t is time in hours. (a) If there are initially 20 000 bacteria, how many will there be after 5 hours? (b) How many hours, to the nearest hour, would it take for the number of bacteria to reach 200 000? The decay of radium is proportional to its mass. If 100 kg of radium takes 5 years to decay to 95 kg (a) show that the mass of radium is given by M = 100 e − 0.01t (b) find its mass after 10 years (c) find its half-life (the time taken for the radium to halve its mass).

4.

A chemical reaction causes the amount of chlorine to be reduced at a rate proportional to the amount of chlorine present at any one time. If the amount of chlorine is given by the formula A = A 0 e − kt , and 100 L reduces to 65 L after 5 minutes, find (a) the amount of chlorine after 12 minutes (b) how long it will take for the chlorine to reduce to 10 L.

5.

The production output in a factory increases according to the equation P = P0 e kt , where t is in years. (a) Find P0 if the initial output is 5000 units. (b) The factory produces 8000 units after 3 years. Find the value of k, to 3 decimal places. (c) How many units will the factory produce after 6 years? (d) The factory needs to produce 20 000 units to make a maximum profit. After how many years, correct to 1 decimal place, will this happen?

6.

The rate of depletion of rainforests can be estimated as proportional to the amount of rainforests. If 3 million m2 of rainforest is reduced to 2.7 million m2 after 20 years, find how much rainforest there will be after 50 years.

Chapter 6 Applications of Calculus to the Physical World

(b) the rate at which the sugar will be reducing after 8 hours (c) how long it will take to reduce to 50 kg.

7.

8.

9.

The annual population of a country is increasing at a rate of dP 6.9%; that is, = 0.069P. If the dt population is 50 000 in 2015, find (a) a formula for the population growth (b) the population in the year 2020 (c) the rate at which the population will be growing in the year 2020 (d) in which year the population will reach 300 000. An object is cooling down according to the formula T = T0 e − kt , where T is temperature in degrees Celsius and t is time in minutes. If the temperature is initially 90°C and the object cools down to 81°C after 10 minutes, find (a) its temperature after half an hour (b) how long (in hours and minutes) it will take to cool down to 30°C. In the process of the inversion of sugar, the amount of sugar present is given by the formula S = Ae kt . If 150 kg of sugar is reduced to 125 kg after 3 hours, find (a) the amount of sugar after 8 hours, to the nearest kilogram

10. The mass, in grams, of a radioactive substance is given by M = M 0 e − kt , where t is time in years. Find (a) M 0 and k if a mass of 200 kg decays to 195 kg after 10 years (b) mass after 15 years (c) the rate of decay after 15 years (d) the half-life of the substance (time taken to decay to half its mass). 11. The number of bacteria in a culture increases from 15 000 to 25 000 in 7 hours. If the rate of bacterial growth is proportional to the number of bacteria at that time, find (a) a formula for the number of bacteria (b) the number of bacteria after 12 hours (c) how long it will take for the culture to produce 5 000 000 bacteria. 12. A population in a certain city is growing at a rate proportional to the population itself. After 3 years the population increases by 20%. How long will it take for the population to double?

229

230

Maths In Focus Mathematics HSC Course

13. The half-life of radium is 1600 years. (a) Find the percentage of radium that will be decayed after 500 years. (b) Find the number of years that it will take for 75% of the radium to decay. 14. The population of a city is P(t) at any one time. The rate of decline in population is proportional to the population P(t), that is, dP (t) = − kP (t) . dt (a) Show that P(t) = P(t 0) e − kt is a solution of the differential dP (t) = − kP (t) . dt (b) What percentage decline in population will there be after 10 years, given a 10% decline in 4 years? (c) What will the percentage rate of decline in population be after 10 years? (d) When will the population fall by 20%? equation

15. The rate of leakage of water out of a container is proportional to the amount of water in the container at any one time. If the container is 60% empty after 5 minutes, find how long it will take for the container to be 90% empty. 16. Numbers of sheep in a certain district are dropping exponentially due to drought. A survey found that numbers had declined by 15% after 3 years. If the drought continues, how long would it take to halve the number of sheep in that district?

17. Anthony has a blood alcohol level of 150 mg/dL. The amount of alcohol in the bloodstream decays exponentially. If it decreases by 20% in the first hour, find (a) the amount of alcohol in Anthony’s blood after 3 hours (b) when the blood alcohol level reaches 20 mg/dL. 18. The current C flowing in a conductor dissipates according dC to the formula = − kC. If it dt dissipates by 40% in 5 seconds, how long will it take to dissipate to 20% of the original current? 19. Pollution levels in a city have been rising exponentially with a 10% increase in pollution levels in the past two years. At this rate, how long will it take for pollution levels to increase by 50%? 20. If

dQ = kQ , prove that Q = Ae kt dt

satisfies this equation by (a) differentiating Q = Ae kt (b) integrating

dQ = kQ . dt

Chapter 6 Applications of Calculus to the Physical World

Class Investigation Research some environmental issues. For example: 1. It is estimated that some animals, such as pandas and koalas, will be extinct soon. How soon will pandas be extinct? Can we do anything to stop this extinction? 2. How do the greenhouse effect and global warming affect the earth? How soon will they have a noticeable effect on us? 3. How long do radioactive substances such as radium and plutonium take to decay? What are some of the issues concerning the storage of radioactive waste? 4. The erosion and salination of Australian soil are problems that may affect our farming in the future. Find out about this issue, and some possible solutions. 5. The effect of blue-green algae in some of our rivers is becoming a major problem. What steps have been taken to remedy this situation? Look at the mathematical aspects of these issues. For example, what formulae are used to make predictions? What sorts of time scales are involved in these issues? Research other issues, relating to growth and decay, that affect our environment.

Motion of a Particle in a Straight Line In this study of the motion of a particle moving along a straight line, we ignore friction, gravity and other influences on the motion. We can find the particle’s displacement, velocity and acceleration at any one time. The term ‘particle’, or ‘body’, can refer to something quite large (e.g. a car). It describes any moving object.

DID YOU KNOW? Calculus was developed in the 17th century as a solution to problems about the study of motion. Some problems of the time included finding the speed and acceleration of planets, calculating the lengths of their orbits, finding maximum and minimum values of functions, finding the direction in which an object is moving at any one time and calculating the areas and volumes of certain figures.

231

232

Maths In Focus Mathematics HSC Course

Displacement Displacement (x) measures the distance of a particle from a fixed point (origin). Unlike distance, displacement can be positive or negative, according to which side of the origin it is on. Usually, on the right-hand side it will be positive and on the left it will be negative. When the particle is at the origin, its displacement is zero. That is, x = 0.

Velocity Velocity (v) is a measure of the rate of change of displacement with respect to time. Since the rate of change is the gradient of the graph of displacement against time, dx . dt . Velocity can also be written as x. v=

Velocity (unlike speed) can be positive or negative, according to which direction the object is travelling in. If the particle is moving to the right, velocity is positive. If it is moving to the left, velocity is negative. When the object is not moving, we say that it is at rest. That is, v = 0.

Acceleration Acceleration is the measure of the rate of change of velocity with respect to time. This gives the gradient of the graph of velocity against time. That is, dv d dx d2 x = c m= 2 . dt dt dt dt .. Acceleration can also be written as x. Acceleration can be positive or negative, according to which direction it is in. If the acceleration is to the right, it is positive. If the acceleration is to the left, it is negative. If the acceleration is in the same direction as the velocity, the particle is speeding up (accelerating). If the acceleration is in the opposite direction from the velocity, the particle is slowing down (decelerating). If the object is travelling at a constant velocity, there is no acceleration. That is, a = 0. a=

Motion graphs You can describe the velocity of a particle by looking at the gradient function of a displacement graph. The acceleration is the gradient function of a velocity graph.

Chapter 6 Applications of Calculus to the Physical World

EXAMPLES 1. The graph below shows the displacement of a particle from the origin as it moves in a straight line.

(a) When is the particle (i) at rest? (ii) at the origin? (b) When is the particle moving at its greatest velocity?

Solution dx = 0. dt So the particle is at rest at the stationary points t 3 and t 5 . (ii) The particle is at the origin when x = 0, i.e. on the t-axis. So the particle is at the origin at t2, t4 and t 6 . (b) The greatest velocity is at t4 (the curve is at its steepest). Notice that this is where there is a point of inflexion. (a) (i)

When the particle is at rest, velocity is zero. i.e.

2. The graph below shows the displacement x of a particle over time t. x

t1

t2

t

CONTINUED

233

234

Maths In Focus Mathematics HSC Course

(a) Draw a sketch of its velocity (b) Sketch its acceleration (c) Find values of t for which the particle is (i) at the origin and (ii) at rest.

Solution dx . By noting dt where the gradient of the tangent is positive, negative and zero, we draw the velocity graph.

(a) The velocity is the rate of change of displacement, or

x + + + + + t1

-

t2 +

t

+ 0

v

t1

t2

t

d2 x . By noting where dt 2 the gradient of the tangent is positive, negative and zero, we draw the acceleration graph.

(b) Acceleration is the rate of change of velocity, or

Chapter 6 Applications of Calculus to the Physical World

235

v + + + +

t1

t2

t

The gradient is a constant, so acceleration is constant.

a

t1

t2

t

(c) (i) The particle is at the origin when x = 0. This is at 0 and t2. (ii) The particle is at rest when v = 0. This is at t1 (on the t-axis on the velocity graph or the stationary point on the displacement graph). 3. The graph below is the velocity of a particle. v

t1

t2

t3

t4

t5

t

CONTINUED

236

Maths In Focus Mathematics HSC Course

(a) Draw a sketch showing the displacement of the particle. (b) Find the times when the particle is (i) at rest (ii) at maximum velocity. (c) Explain why you can’t state accurately when the particle is at the origin.

Solution (a) For 0 − t 1 and between t3 and t5, the velocity is negative (below the dx < 0. t-axis), so the displacement has dt Between t1 and t3, and all points > t5, the velocity is positive (above dx > 0. the t-axis), so the displacement has dt dx = 0 and there is a At t1, t3 and t5, the velocity is zero, so dt stationary point. v +

+

+ +

+

+ 0 t1

0 t2

-

0

t3 -

t4

t5

-

t

-

-

-

Notice that at t1 and t5, LHS < 0 and RHS > 0 so they are minimum turning points. At t3, LHS > 0 and RHS < 0 so it is a maximum turning point. Here is one graph that could describe the shape of the displacement. x

t1

t2

(b) (i) At rest, v = 0. This occurs at t1, t3 and t5.

t3

t4

t5

t

Chapter 6 Applications of Calculus to the Physical World

(ii) Maximum velocity is at t4 since it is furthest from the t-axis. Notice also that there is a point of inflexion at t4 on the displacement graph. (c) You can’t tell when the particle is at the origin as the graph is not accurate. For example, the graph in (a) is at the origin at t4 and another point to the right of t5. However, when sketching displacement from the velocity (or the original function given the gradient or derivative function) there are many possible graphs, forming a family of graphs. x

t1

t2

t3

t4

t

t5

Notice that these cross the t-axis at various places, so we can’t tell where the displacement graph shows the particle at the origin.

6.3 Exercises 1.

The graphs at right and overleaf show the displacement of an object. Sketch the graphs for velocity and acceleration.

(a)

x

t

237

238

Maths In Focus Mathematics HSC Course

(b)

When is the particle (a) at the origin? (b) at rest? (c) travelling with constant acceleration? (d) furthest from the origin?

x

t

3. (c)

x

The graph below shows a particle travelling at a constant acceleration.

t

(d) Sketch the graphs that could represent (a) its velocity and (b) its displacement. (e)

4.

The graph below shows the velocity of a particle. x

t t1

2.

The graph below shows the displacement of a particle as it moves along a straight line.

t2

t3

t4

t5

t6

(a) When is the particle at rest? (b) When is the acceleration zero? (c) When is the velocity the greatest? (d) Describe the motion of the particle at (i) t2 and (ii) t3.

Chapter 6 Applications of Calculus to the Physical World

5.

The graph below shows the displacement of a pendulum.

(b)

(c)

(a) When is the pendulum at rest? (b) When is the pendulum in its equilibrium position (at the origin)? 6.

(d)

Describe the motion of the particle at t 1 for each displacement graph. (a) (e)

Motion and Differentiation If displacement is given, then the velocity is the first derivative and the acceleration is the second derivative.

Displacement x . dx Velocity x = dt Acceleration px =

dv d 2 x = 2 dt dt

239

240

Maths In Focus Mathematics HSC Course

EXAMPLES 1. The displacement x cm of a particle over time t seconds is given by x = 4t − t 2 . (a) Find any times when the particle is at rest. (b) How far does the particle move in the first 3 seconds?

Solution (a) At rest means that the velocity c

dx m is zero. dt

dx = 4 − 2t dt dx When =0 dt 4 – 2t = 0 4 = 2t 2=t So the particle is at rest after 2 seconds. Notice that there is a stationary point at around at this time.

dx = 0, so the particle turns dt

(b) Initially, when t = 0: x = 4(0) − 0 2 =0 So the particle is at the origin. After 3 seconds, t = 3: x = 4(3) − 3 2 =3 So the particle is 3 cm from the origin. However, after 2 seconds, the particle turns around, so it hasn’t simply moved from the origin to 3 cm away. We need to find where it is when it turns around. When t = 2: x = 4 (2) − 2 2 =4 So the particle is 4 cm from the origin. The particle moves from the origin (x = 0) to 4 cm away in the first 2 seconds, so it has travelled 4 cm. It then turns and goes back to 3 cm from the origin. So in the 3rd second it travels 1 cm back.

Chapter 6 Applications of Calculus to the Physical World

0

1

2

3

4

241

5

Total distance travelled in the first 3 seconds is 4 + 1 or 5 cm. 2. The displacement of a particle is given by x = − t 2 + 2t + 3 cm, where t is in seconds. (a) Find the initial velocity of the particle. (b) Show that the particle has constant acceleration. (c) Find when the particle will be at the origin. (d) Find the particle’s maximum displacement from the origin. (e) Sketch the graph showing the particle’s motion.

Solution (a)

dx dt = − 2t + 2 Initially, t = 0 ` v = − 2 (0) + 2 =2 v=

So the initial velocity is 2 cms− 1. d2 x (b) a = 2 dt =−2 ∴ the particle has a constant acceleration of − 2 cms− 2. (c) At the origin x = 0 i.e. − t2 + 2t + 3 = 0 − (t + 1)(t − 3) = 0 ` t = − 1 or 3 So the particle will be at the origin after 3 s.

A time of –1 does not exist.

(d) For maximum displacement, dx =0 dt − 2t + 2 = 0 2 = 2t 1=t (Maximum displacement occurs when t = 1.) When t = 1, x = − (1) 2 + 2 (1) + 3 =4 So maximum displacement is 4 cm. CONTINUED

242

Maths In Focus Mathematics HSC Course

(e)

This is the graph of a parabola. Notice that x can be positive or negative, but t is never negative.

3. The displacement of a particle is given by the formula x = 1 + 3 sin 2t, where x is in metres and t is in seconds. (a) Sketch the graph of x as a function of t. (b) Find the times when the particle will be at rest. (c) Find the position of the particle at those times.

Solution (a) Graph has period π and amplitude 3.

ds dt = 6 cos 2t

(b) v =

When the particle is at rest v=0 i.e. 6 cos 2t = 0 cos 2t = 0 π 3π 5π 2t = , , ,... 2 2 2 π 3π 5π ` t= , , ,... 4 4 4

Chapter 6 Applications of Calculus to the Physical World

(c) x = 1 + 3 sin 2t π When t = 4 x = 1 + 3 sin

π 2

= 1 + 3 (1) =4 3π When t = 4 3π 2 = 1 + 3 (− 1) = −2 5π 7π Similarly, when t = , , …, x moves between 4 m and –2 m from 4 4 the origin. x = 1 + 3 sin

Investigation In physics, equations for displacement and velocity are given by s = ut + 1 at 2 and v = u + at, where u is the initial velocity and a is the 2 acceleration. Differentiate the displacement formula to show the formula for velocity. What happens when you differentiate again?

6.4 Exercises 1.

The displacement of a particle is given by x = t 3 − 9t cm, where t is time in seconds. (a) Find the velocity of the particle after 3 s. (b) Find the acceleration after 2 s. (c) Show that the particle is initially at the origin, and find any other times that the particle will be at the origin. (d) Find after what time the acceleration will be 30 cms − 2 .

2.

A particle is moving such that its displacement is given by s = 2t 2 − 8t + 3, where s is in metres and t is in seconds. (a) Find the initial velocity. (b) Show that acceleration is constant and find its value. (c) Find the displacement after 5 s. (d) Find when the particle will be at rest. (e) What will the particle’s displacement be at that time? (f) Sketch the graph of the displacement against time.

243

244

Maths In Focus Mathematics HSC Course

3.

4.

5.

6.

A projectile is fired into the air and its height in metres is given by h = 40t − 5t 2 + 4, where t is in seconds. (a) Find the initial height. (b) Find the initial velocity. (c) Find the height after 1 s. (d) What is the maximum height of the projectile? (e) Sketch the graph of the height against time. The displacement in cm after time t s of a particle moving in a straight line is given by x = 2 − t − t 2. (a) Find the initial displacement. (b) Find when the particle will be at the origin. (c) Find the displacement after 2 s. (d) How far will the particle move in the first 2 seconds? (e) Find its velocity after 3 s. The equation for displacement of a particle is given by x = e 2t + 1 m after time t seconds. (a) Find the initial velocity. (b) Find the exact acceleration after 1 s. (c) Show that the acceleration is always double the velocity. (d) Sketch the graph of velocity over time t. The displacement of a pendulum is given by x = cos 2t cm after time t seconds. (a) Find the equation for the velocity of the pendulum. (b) Find the equation for its acceleration. (c) Find the initial displacement.

(d) Find the times when the pendulum will be at rest. (e) What will the displacement be at these times? (f) When will there be zero displacement? (g) Show that acceleration a = − 4x. 7.

An object is travelling along a straight line over time t seconds, with displacement according to the formula x = t 3 + 6t 2 − 2t + 1 m. (a) Find the equations of its velocity and acceleration. (b) What will its displacement be after 5 s? (c) What will its velocity be after 5 s? (d) Find its acceleration after 5 s.

8.

The displacement, in centimetres, of a body is given by x = (4t − 3)5 , where t is time in seconds. . (a) Find equations for velocity x .. and acceleration x . . .. (b) Find the values of x, x and x after 1 s. (c) Describe the motion of the body after 1 s.

9.

The displacement of a particle, in metres, over time t seconds 1 is s = ut + gt 2, where u = 5 and 2 g = − 10. (a) Find the equation of the velocity of the particle. (b) Find the velocity after 10 s. (c) Show that the acceleration is equal to g.

10. The displacement in metres after 2t − 5 t seconds is given by s = . 3t + 1 Find the equations for velocity and acceleration.

Chapter 6 Applications of Calculus to the Physical World

11. The displacement of a particle is given by x = log e (t + 1), where x is in centimetres and t is in seconds. (a) Find the initial position of the particle. (b) Find the velocity after 5 s. (c) Find the acceleration after 5 s. (d) Describe the motion of the particle at 5 s. (e) Find the exact time when the particle will be 3 cm to the right of the origin. 12. Displacement of a particle is given by x = t 3 − 4t 2 + 3t, where s is in metres and t is in seconds. (a) Find the initial velocity. (b) Find the times when the particle will be at the origin. (c) Find the acceleration after 3 s. 13. The displacement of a particle moving in simple harmonic motion is given by x = 3 sin 2t, where x is in centimetres and t is in seconds. (a) Sketch the graph of the displacement of the particle from t = 0 to t = 2π . (b) Find equations for the .. . velocity x and acceleration x . (c) Find the exact acceleration π after s. 3 .. (d) Show that x = − 4x. 14. The height of a projectile is given by h = 7 + 6t − t 2, where height is in metres and time is in seconds. (a) Find the initial height. (b) Find the maximum height reached.

(c) When will the projectile reach the ground? (d) Sketch the graph showing the height of the projectile over time t. (e) How far will the projectile travel in the first 4 s? 15. A ball is rolled up a slope at a distance from the base of the slope, after time t seconds, given by x = 15t − 3t 2 metres. (a) How far up the slope will the ball roll before it starts to roll back down? (b) What will its velocity be when it reaches the base of the slope? (c) How long will the motion of the ball take altogether? 16. The displacement of a particle is given by x = 2t 3 − 3t 2 + 42t. (a) Show that the particle is initially at the origin but never returns to the origin. (b) Show that the particle is never at rest. 17. A weight on the end of a spring has a height given by h = 3 sin 2t cm where t is time in seconds. (a) Find its initial height. (b) Find when the spring is at its maximum distance from the origin. (c) What is the acceleration at these times?

245

246

Maths In Focus Mathematics HSC Course

18. A particle moves so that its displacement x cm is given by x = e t at t seconds. (a) Find the exact velocity of the particle after 4 seconds. (b) When is the particle at rest? (c) Where is the particle at that time? 2

19. A particle is moving in a straight line so that its displacement x cm over time t seconds is given by x = t 49 − t2 . (a) For how many seconds does the particle travel? (b) Find the exact time at which the particle comes to rest. (c) How far does the particle move altogether?

Motion and Integration Displacement is the area under the velocity–time graph.

If the first derivative of the displacement of a particle gives its velocity, then the reverse is true. That is, the primitive function (indefinite integral) of the velocity gives the displacement. Similarly, the integral of the acceleration gives the velocity. x = # v dt

v = # a dt

EXAMPLES 1. The velocity of a particle is given by v = 3t 2 + 2t + 1. If initially the particle is 2 cm to the left of the origin, find the displacement after 5 s.

Solution x = # v dt

Initially means t = 0

= # (3t 2 + 2t + 1) dt = t3 + t2 + t + C When t = 0, x = − 2

`

− 2 = 03 + 02 + 0 + C =C

`

x = t3 + t2 + t − 2

When t = 5, x = 53 + 52 + 5 − 2 = 125 + 25 + 5 − 2 = 153 So after 5 s the particle will be 153 cm to the right of the origin.

Chapter 6 Applications of Calculus to the Physical World

247

2 ms − 2. If the 2 (t + 1) particle is initially at rest 1 m to the right of the origin, find its exact displacement after 9 s. 2. The acceleration of a particle is given by a = 6 −

Solution v= = =

# a dt # e 6 − (t +21) 2 o

dt

# [6 − 2 (t + 1)− 2 ] dt

(t + 1)− 1 = 6t − 2 +C −1 2 = 6t + +C t+1 When t = 0, v = 0 2 +C ` 0 = 6(0) + 0+1 =2+C ` C = −2 2 v = 6t + So −2 t+1 x = # v dt 2 − 2 m dt t +1 = 3t 2 + 2 log e (t + 1) − 2t + C = # c 6t +

When t = 0, x = 1 `

1 = 3(0)2 + 2 loge (0 + 1) − 2(0) + C =C

`

x = 3t2 + 2 loge (t + 1) − 2t + 1

When t = 9 x = 3 (9) 2 + 2 loge (9 + 1) − 2 (9) + 1 = 243 + 2 loge 10 − 18 + 1 = 226 + 2 loge 10 = 2 (113 + loge 10) So after 9 s the displacement will be 2(113 + log e 10) m.

You must find C before going on to find the displacement.

‘At rest’ means v = 0.

248

Maths In Focus Mathematics HSC Course

6.5 Exercises 1.

The velocity of a particle is given by v = 3t2 − 5 cms− 1. If the particle is at the origin initially, find its displacement after 3 s.

2.

A particle has velocity given by dx = 2t − 3 ms − 1. After 3 s the dt particle is at the origin. Find its displacement after 7 s.

3.

The velocity of a particle is given by v = 4 − 3t cms − 1. If the particle is 5 cm from the origin after 2 s, find the displacement after 7 s.

4.

The velocity of a particle is given by v = 8t3 − 3t2 cms− 1. If the particle is initially at the origin, find (a) its acceleration after 5 s (b) its displacement after 3 s (c) when it will be at the origin again.

5.

The velocity of a particle is given by x: = 3e3t cms− 1. Given that the particle is initially 2 cm to the right of the origin, find its exact displacement after 1 s.

6.

A particle has a constant acceleration of 12 ms− 2. If the particle has a velocity of 2 ms− 1 and is 3 m from the origin after 5 s, find its displacement after 10 s.

7.

8.

The acceleration of an object is given by 6t + 4 cms− 2. The particle is initially at rest at the origin. Find (a) its velocity after 5 s (b) its displacement after 5 s. A projectile is accelerating at a constant rate of − 9.8 ms− 2. If it is initially 2 m high and has

a velocity of 4 ms− 1, find the equation of the height of the projectile. 9.

A particle is accelerating according to the equation a = (3t + 1)2 ms− 2. If the particle is initially at rest 2 m to the left of 0, find its displacement after 4 s.

10. The velocity of a particle is given by v = e t − 1 ms − 1. If the particle is initially 3 m to the right of the origin, find its exact position after 5 s. 11. A particle accelerates according to the equation a = − e 2t . If the particle has initial velocity of zero cms− 1 and initial displacement of − 1 cm, find its displacement after 4 s, to the nearest centimetre. 12. The acceleration of a particle is given by a = − 9 sin 3t cms− 2. If the initial velocity is 5 cms− 1 and the particle is 3 cm to the left of the origin, find the exact displacement after π s. 13. The velocity of a particle is given t by x: = 2 ms − 1. If the particle t +3 is initially at the origin, find its displacement after 10 s, correct to 2 decimal places. 14. The acceleration of an object is given by a = e 3t ms − 2. If initially the object is at the origin with velocity − 2 ms − 1, find its displacement after 3 seconds, correct to 3 significant figures.

Chapter 6 Applications of Calculus to the Physical World

15. The velocity of a particle is given by v = 4 cos 2t ms − 1. If the particle is 3 m to the right of the origin after π s, find the exact π (a) displacement after s 6 π (b) acceleration after s. 6 16. The acceleration of a particle is 2 given by a = ms−2 and the 2 (t + 3) particle is initially at rest 2 ln 3 m to the left of the origin. (a) Evaluate its velocity after 2 seconds. (b) Find its displacement after n seconds. (c) Show that 0 ≤ v < 1 for all t.

17. The acceleration of a particle is .. x = 25 e 5t ms−2 and its velocity is 5 ms−1 initially. Its displacement is initially 1 m. (a) Find its velocity after 9 seconds. (b) Find its displacement after 6 seconds. (c) Show that the acceleration .. x = 25x. (d) Find the acceleration when the particle is 2 m to the right of the origin.

249

250

Maths In Focus Mathematics HSC Course

Test Yourself 6 1.

A particle moves so that its displacement after t seconds is x = 4t 2 − 5t 3 metres. Find (a) its initial displacement, velocity and acceleration (b) when the particle is at the origin (c) its maximum displacement.

2.

The velocity of a particle is given by V = 6t − 12t2 ms− 1 . Find its displacement and acceleration after 3 seconds if it is initially 5 m to the right of the origin.

3.

A city doubles its population in 25 years. If it is growing exponentially, when will it triple its population?

4.

The displacement of a particle after t seconds is x = 2e3t cm. (a) Find its initial velocity. (b) Find its acceleration after 3 seconds, to 1 decimal place. .. (c) Show that x = 9x.

5.

.. A particle accelerates at x = 8 cos 2t ms − 2. The particle is initially at rest at the origin. Find its displacement after π seconds. 6

6.

If a particle has displacement x = 2 sin 3t, show that its acceleration is given by .. x = –9x.

7.

A particle has displacement x = t3 − 12t2 + 36t − 9 cm at time t seconds. (a) When is the particle at rest? (b) What is the (i) displacement (ii) velocity (iii) acceleration after 1 s? (c) Describe the motion of the particle after 1 s.

8.

The acceleration of a particle is .. x = 6t − 12 ms− 2 . The particle is initially at rest 3 metres to the left of the origin. (a) Find the (i) acceleration (ii) velocity (iii) displacement after 5 seconds. (b) Describe the motion of the particle after 5 seconds.

9.

The graph below shows the displacement of a particle.

(a) When is the particle at the origin? (b) When is it at rest? (c) When is it travelling at its greatest speed? (d) Sketch the graph of its (i) velocity (ii) acceleration. 10. A radioactive substance decays by 10% after 80 years. (a) By how much will it decay after 500 years? (b) When will it decay to a quarter of its mass? 11. The equation for displacement of a particle over time t seconds is x = sin t metres. (a) When is the displacement 0.5 m? (b) When is the velocity 0.5 ms− 1 ? π (c) Find the acceleration after s. 4

Chapter 6 Applications of Calculus to the Physical World

12. The height of a ball is h = 20t − 5t2 metres after t seconds. (a) Find the height after 1 s. (b) What is the maximum height of the ball? (c) What is the time of flight of the ball?

14. The graph below shows the velocity of a particle.

13. A bird population of 8500 increases to 12 000 after 5 years. Find (a) the population after 10 years (b) the rate at which the population is increasing after 10 years (c) when the population reaches 30 000.

(a) Sketch a graph that shows (i) displacement (ii) acceleration (b) When is the particle at rest? 15. A particle is moving with acceleration a = 10e2t ms− 2. If it is initially 4 m to the right of the origin and has a velocity of − 2 ms− 1, find its displacement after 5 seconds, to the nearest metre.

Challenge Exercise 6 1.

(a) Find an equation for the displacement of a particle from the origin if its acceleration is given by dv = 6 cos 3t cms − 2 and initially the dt particle is at rest 2 cm to the right of the origin. (b) Write the acceleration of the particle in terms of x.

3.

(a) Find an equation for the displacement of a particle from the origin if its acceleration is given by dv = − 16 cos 4t cms− 2 and initially the dt particle is at rest 1 cm to the right of the origin. (b) Find the exact velocity of the particle when it is 0.5 cm from the origin.

2.

The displacement of a particle is given by x = (t3 + 1) 6, where x is in metres and t is in seconds. (a) Find its initial displacement and velocity. (b) Find its acceleration after 2 s in scientific notation, correct to 3 significant figures. (c) Show that the particle is never at the origin.

4.

The velocity, in ms− 1, of an object is given by v = 5 cos 5t. (a) Show that if the object is initially at the origin, its acceleration will always be –25 times its displacement. (b) Find the maximum acceleration of the object. (c) Find the acceleration when the object is 0.3 m to the right of the origin.

251

252

Maths In Focus Mathematics HSC Course

5.

The population of a flock of birds over t years is given by the formula P = P0 e 0.0151t . (a) How long will it take, correct to 1 decimal place, to increase the population by 35%? (b) What will be the percentage increase in population after 10 years, to the nearest per cent?

6.

The Logistic Law of Population Growth, first proposed by Verhulst in 1837, is dN given by = kN − bN2, where k and b dt are constants. Show that the equation kN0 N= is a solution of this bN0 + (k − bN0) e − kt differential equation (N0 is a constant).

7.

A particle moves so that its velocity is given by x: = tet cms− 1. Find its exact acceleration after 1 s. 2

7

Series TERMINOLOGY Annuity: An annuity is a fixed sum of money invested Terminology Bold Terminology text... every year that accumulates interest over a number of years Arithmetic sequence: A set of numbers that form a pattern where each successive term is a constant amount (positive or negative) more than the previous one Common difference: The constant amount in an arithmetic sequence that is added to each term to get the next term Common ratio: The constant amount in a geometric sequence that is multiplied to each term to get the next term Compound interest: Interest is added to the balance of a bank account so that the interest and balance both earn interest Geometric sequence: A set of numbers that form a pattern where each successive term is multiplied by a constant amount to get the next term Limiting sum: (or sum to infinity) The sum of infinite terms of a geometric series where the common ratio r obeys the condition −1 < r < 1

Partial sum: The sum of a certain finite number of terms of a sequence Sigma notation: The Greek letter stands for the sum of a sequence of a certain number of terms, called a partial sum Sequence: A set of numbers that form a pattern or obey a fixed rule Series: The sum of a sequence of n terms Superannuation: A sum of money that is invested every year (or more frequently) as part of a salary to provide a large amount of money when a person retires from the paid workforce Term: A term refers to the position of the number in a sequence. For example the first term is the first number in the sequence

Chapter 7 Series

259

INTRODUCTION THE INFINITE SUM OF a sequence of numbers (or terms) is called a series. Many

series occur in real life—think of the way plants grow, or the way money accumulates as a certain amount earns interest in a bank. You will look at series in general, arithmetic, geometric series and their applications.

General Series A sequence forms a pattern. Some patterns are easy to see and some are difficult. People are sometimes asked to identify sequences and their patterns in tests of intelligence.

DID YOU KNOW? Number patterns and series of numbers have been known since the very beginning of civilisation. The Rhind Papyrus, an Egyptian book written about 1650 BC, is one of the oldest books on mathematics. It was discovered and deciphered by Eisenlohr in 1877, and it showed that the ancient Egyptians had a vast knowledge of mathematics. The Rhind Papyrus describes the problem of dividing 100 loaves among 5 people in a way that shows the Egyptians were exploring arithmetic series. Around 500 BC the Pythagoreans explored different polygonal numbers: • triangular numbers: 1 + 2 + 3 + 4 + …

DID YOU KNOW? Box text...

1

1+ 2

1+ 2 + 3

1+ 2 + 3 + 4

1+ 3 + 5

1+ 3 + 5 + 7

• square numbers: 1 + 3 + 5 + 7 + …

1

1+ 3

Could you find a series for pentagonal or hexagonal numbers?

260

Maths In Focus Mathematics HSC Course

7.1 Exercises Find the next 3 terms in each sequence or series of numbers. 1.

5, 8, 11, ...

11. 1, − 2, 4, − 8, …

2.

8, 13, 18, ...

12. 3, − 6, 12, − 24, …

3.

11 + 22 + 33 + …

4.

100, 95, 90, ...

5.

7+5+3+ …

1 , 2 2 14. , 5

6.

99, 95, 91, ...

16. 1, 8, 27, 64, ...

7.

1 1 , 1, 1 , … 2 2

17. 0, 3, 8, 15, 24, ...

8.

1.3 + 1.9 + 2.5 + f

18. 3 + 6 + 11 + 18 + 27 + …

9.

2, 4, 8, ...

19. 2 + 9 + 28 + 65 + …

10. 4 + 12 + 36 + …

13.

1 1 , ,… 4 8 4 8 , ,… 15 45

15. 1 + 4 + 9 + 16 + 25 + …

20. 1, 1, 2, 3, 5, 8, 13, ...

DID YOU KNOW? The numbers 1, 1, 2, 3, 5, 8, … are called Fibonacci numbers after Leonardo Fibonacci (1170–1250). The Fibonacci numbers occur in natural situations. For example, when new leaves grow on a plant’s stem, they spiral around the stem. The ratio of the number of turns to the number of spaces between successive leaves gives the 1 1 2 3 5 8 13 21 34 55 89 series of fractions , , , , , , , , , , ,… 1 2 3 5 8 13 21 34 55 89 144

Chapter 7 Series

The Fibonacci ratio is the number of turns divided by the number of spaces.

Research Fibonacci numbers and find out where else they appear in nature.

Formula for the nth term of a series When a series follows a mathematical pattern, its terms can be described by a formula. We call any general or nth term of a series Tn where n stands for the number of the term and must be a positive integer.

EXAMPLE For the series 6 + 13 + 20 + … find (a) T1 (b) T2 (c) T3 (d) Tn

Solution (a) (b) (c) (d)

The first term is 6, so when n = 1: T1 = 6 The 2nd term is 13 so T2 = 13 The 3rd term is 20 so T3 = 20 Each term is 7 more than the previous term. The 1st term is 6 The 2nd term is 13 = 6 + 7 The 3rd term is 20 = 6 + 7 + 7 = 6 + 2 ´7 CONTINUED

261

262

Maths In Focus Mathematics HSC Course

Following this pattern:

Notice that the product with 7 in each term involves a number one less than the number of the term.

The 4th term = 6 + 7 + 7 + 7 = 6 + 3´7 and so on Tn = 6 + (n − 1) ´ 7 Tn = 6 + 7n − 7 = 7n − 1

If we are given a formula for the nth term, we can find out much more about the series.

EXAMPLES 1. The nth term of a series is given by the formula Tn = 3n + 4. Find the first 3 terms and hence write down the series.

Solution T1 = 3 (1) + 4 =7 T2 = 3 (2) + 4 = 10 T3 = 3 (3) + 4 = 13 So the series is 7 + 10 + 13 + … 2. The nth term of a series is given by tn = 5n − 1. Which term of the series is equal to 104?

Solution tn = 5n − 1 = 104 5n = 105 n = 21 So 104 is the 21st term of the series. 3. The nth term of a series is given by u n = 103 − 3n. (a) Find the first 3 terms. (b) Find the value of n for the first negative term in the series.

Chapter 7 Series

263

Solution (a) u1 = 103 − 3 (1) = 100 u2 = 103 − 3 (2) = 97 u3 = 103 − 3 (3) = 94 So the series is 100 + 97 + 94 + . . . . (b) For the first negative term, we want un < 0 i.e. 103 − 3n < 0 − 3n < −103 1 3 Since n is an integer, n = 35 ∴ the 35th term is the first negative term. n > 34

4. The nth term of a series is given by the formula tn = 2n − 1. Which term of the series is equal to 4095?

Solution tn = 2n − 1 We are given that the nth term is 4095 and need to find n. 4095 = 2n − 1 4096 = 2n log10 4096 = log10 2n = n log10 2 log10 4096 =n log10 2 12 = n

You could use base e instead of base 10. You could use trial and error to guess what power of 2 is equal to 4096 if you don’t want to use logarithms.

So 4096 is the 12th term of the sequence.

7.2 Exercises 1.

Find the first 3 terms of the series with nth term as follows (a) Tn = 8n − 5 (b) Tn = 2n + 3 (c) un = 6n − 1 (d) Tn = 8 − 5n (e) tn = 20 − n

(f) un = 3n (g) Qn = 2n + 7 (h) tn = 4n − 2n (i) Tn = 8n2 − n + 1 (j) Tn = n3 + n

264

Maths In Focus Mathematics HSC Course

2.

3.

4.

5.

6.

Find the first 3 terms of each series. (a) Tn = 3n − 2 (b) tn = 4n (c) Tn = n2 + n Find the 50th term of each series. (a) Tn = 7n − 1 (b) tn = 2n + 5 (c) Tn = 5n − 2 (d) Tn = 40 − 3n (e) Un = 8 − 7n Find the 10th term of each series. (a) Tn = 2n + 5 (b) tn = 10 − 3n (c) Tn = 2n − n (d) un = n2 − 5n + 3 (e) Tn = n3 + 2 Which of these are terms of the series Tn = 3n + 5? (a) 68 (b) 158 (c) 205 (d) 266 (e) 300 Which of these are terms of the series Tn = 5n − 1? (a) 3126 (b) 124 (c) 15 634 (d) 78 124 (e) 0

10. Which term of Tn = n2 − 3 is equal to 526? 11. For the series Tn = n3, find (a) the 12th term (b) n if the nth term is 15 625. 12. A series is given by the formula Tn = 3 + 2n − n2. (a) Find the 25th term. (b) Find n if the nth term is − 252. 13. Find the value of n that gives the first term of the series Tn = 3n + 2 greater than 100. 14. Find the value of n that gives the first term of the series Tn = 2n larger than 500. 15. Find the values of n where the series Tn = 4n − 1 is greater than 350. 16. Find the values of n for which the series Tn = 400 − 5n is less than 200. 17. Find the value of n that gives the first negative term of the series Tn = 1000 − 2n. 18. Find the value of n that gives the first positive term of the series Tn = 2n − 300.

7.

Which term of the series with nth term tn = 9n − 15 is equal to 129?

8.

Is 255 a term of the series with nth term Tn = 2n − 1?

19. Find (a) the value of n that gives the first negative term of the series un = 80 − 6n (b) the first negative term.

9.

Which term of the series with nth term un = n3 + 5 is 348?

20. Find the first negative term of the series Tn = 50 − 7n.

Chapter 7 Series

265

Sigma Notation Series are often written in sigma notation. The Greek letter sigma (Σ) is like an English ‘S’. Here it stands for the sum of a series.

Application The mean of a set of scores is given by x = Σfx Σf where Σf is the sum of frequencies and Σfx is the sum of the scores × frequencies.

EXAMPLES 1. Evaluate

5

/ r2

r =1

Solution 5

/ r2 means the sum of terms where the formula is r2 with r starting at 1

r =1

and ending at 5. So

5

/ r2 = 12 + 22 + 32 + 42 + 52

r =1

= 1 + 4 + 9 + 16 + 25 = 55 7

2. Evaluate / (2n + 5) 3

Solution 7

/ (2n + 5) means the sum of terms where the formula is 2n + 5 with n 3

starting at 3 and ending at 7. Can you work out how many terms there are in this series? 7

/ (2n + 5) = (2 ´ 3 + 5) + (2 ´ 4 + 5) + (2 ´ 5 + 5) + (2 ´ 6 + 5) + (2 ´ 7 + 5) 3

= 11 + 13 + 15 + 17 + 19 = 75 CONTINUED

Notice that there are 5 terms, since they include both the 3 and the 7. We work out the number of terms by 7 − 3 + 1.

266

Maths In Focus Mathematics HSC Course

3. Write 7 + 11 + 15 + … + (4k + 3) in sigma notation.

Solution In this question, the formula is 4n + 3 where the last term is at n = k. We need to find the number of terms. The first term is 7 so we guess that n = 1 for this term. When n = 1 4n + 3 = 4 ´ 1 + 3 =7 So the first term is at n = 1 and the last term is at n = k.

If n is not 1, we could try 2 then 3 and so on until we find the first term.

k

We can write the series as / (4n + 3). 1

4. How many terms are there in the series 100

(a)

/ (3n − 7) ? 1

50

(b)

/ 2n ? 5

Solution (a) The first term is when n = 1 and the last term is when n = 100. There are 100 terms. (b) The first term is when n = 5 and the last term is when n = 50. This is harder to work out. Look at example 2 where there are 5 terms. We can work out the number of terms by 50 − 5 + 1. There are 46 terms.

There are 100 − 1 + 1 terms.

p

The number of terms in the series / f (n) is p − q + 1 q

7.3 Exercises 1.

Evaluate (a) (b)

4

/ (3 n + 2)

n =1 5

/ n2

n =2 6

(c)

/ (5n − 6)

(f)

3

(g)

(d)

/ (3r + 1)

r =1

(e)

5

/ (k 3 − 1)

k =2

5

/ (n 2 − n)

n =1 4

(h) / | 3p − 2| 2

2

10

5

/ 1n

6

(i)

/ (2n)

n =1 6

(j)

/ (3 n − 2n − 5) 3

Chapter 7 Series

2.

How many terms are there in each series? 65

(a)

(i)

77

/ (2n − 3) 3

/ (7 ) n

(j)

n =1

55

/ (7 n + n ) 11

100

(b)

/ (n 2 + 1 )

n =2 80

(c)

/ (3n + 4) 5

200

(d)

/ (3r2)

r =1 20

(e)

/ (7n + 1) 10

(f)

45

/ (n 2 + 3n) 7

108

(g) (h)

/

n3

n = 12 74

/ 4n

n =9

3.

Write these series in sigma notation. (a) 1 + 3 + 5 + 7 + … + 11 (b) 7 + 14 + 21 + … + 70 (c) 1 + 8 + 27 + 64 + 125 (d) 2 + 8 + 14 + … + (6n − 4) (e) 9 + 16 + 25 + 36 + … + n2 (f) −1 − 2 − 3 − … − 50 (g) 3 + 6 + 12 + … + 3 ´ 2n 1 1 1 (h) 1 + + + … + 2 4 512 (i) a + (a + d ) + (a + 2d ) + … + (a + 5 n − 1 ? d ) (j) a + ar + ar2 + ar3 + … + ar n − 1

Arithmetic Series In an arithmetic series each term is a constant amount more than the previous term. The constant is called the common difference.

EXAMPLES 1. Find the common difference of the series 5 + 9 + 13 + 17 + 21 + …

Solution We can see that the common difference between the terms is 4. To check this, we notice that 9 − 5 = 4, 13 − 9 = 4, 17 − 13 = 4 and 21 − 17 = 4. 2. Find the common difference of the series 85 + 80 + 75 + 70 + 65 + …

Solution We can see that the common difference between the terms is − 5. To check this, we notice that 80 − 85 = − 5, 75 − 80 = − 5, 70 − 75 = − 5 and 65 − 70 = − 5.

267

268

Maths In Focus Mathematics HSC Course

If T1, T2 and T3 are consecutive terms of an arithmetic series then d = T2 − T1 = T3 − T2 Generally, if T1, T2, T3, … Tn − 1, Tn are consecutive terms of an arithmetic series then d = T2 − T1 = T3 − T2 = … = Tn − Tn − 1 .

EXAMPLES 1. If 5 + x + 31 + … is an arithmetic series, find x.

Solution

5 + 31

Notice that x = 2 which is the average of T1 and T2. We call x the arithmetic mean.

For an arithmetic series, T2 − T1 = T3 − T2 i.e. x − 5 = 31 − x 2x − 5 = 31 2x = 36 x = 18 ` 2. (a) Evaluate k if (k + 2) + (3k + 2) + (6k − 1) + … is an arithmetic series. (b) Write down the first 3 terms of the series. (c) Find the common difference d.

Solution (a) For an arithmetic series, T2 − T1 = T3 − T2 So (3k + 2) − (k + 2) = (6k − 1) − (3k + 2) 3k + 2 − k − 2 = 6k − 1 − 3k − 2 2k = 3k − 3 0 =k−3 3=k (b) The series is (k + 2) + (3k + 2) + (6k − 1) + … Substituting k = 3: T1 = k + 2 =3+2 =5 T2 = 3k + 2 =3´3+ 2 = 11 T3 = 6k − 1 = 6 ´ 3−1 = 17 (c) The sequence is 5 + 11 + 17 + . . . 11 − 5 = 17 − 11 = 6 So common difference d = 6.

Chapter 7 Series

Terms of an arithmetic series a + (a + d ) + (a + 2d ) + (a + 3d ) + … + [a + (n − 1) d ] + … is an arithmetic series with first term a, common difference d and nth term given by Tn = a + (n − 1) d

Proof Let the first term of an arithmetic series be a and the common difference d. Then first term is a second term is a + d third term is a + 2d fourth term is a + 3d, and so on. The nth term is a + (n − 1) d.

EXAMPLES 1. Find the 20th term of the series 3 + 10 + 17 + … .

Solution a = 3, d = 7, n = 20 Tn = a + (n − 1) d T20 = 3 + (20 − 1) 7 = 3 + 19 ´ 7 = 136 2. Find an expression for the nth term of the series 2 + 8 + 14 + . . . .

Solution a = 2, d = 6 Tn = a + (n − 1) d = 2 + (n − 1) 6 = 2 + 6n − 6 = 6n − 4 3. Find the first positive term of the series − 50 − 47 − 44 − … .

Solution a = − 50, d = 3 For the first positive term, Tn > 0 CONTINUED

269

270

Maths In Focus Mathematics HSC Course

i.e. a + (n − 1) d − 50 + (n − 1) 3 − 50 + 3n − 3 3n − 53

>0 >0 >0 >0

3n > 53 2 3 ` n = 18 gives the first positive term T18 = − 50 + (18 − 1) 3 = − 50 + 17 ´ 3 =1 So the first positive term is 1. n > 17

n must be a positive integer.

4. The 5th term of an arithmetic series is 37 and the 8th term is 55. Find the common difference and the first term of the series.

Solution Tn = a + (n − 1) d T5 = a + (5 − 1) d = 37 a + 4d = 37 T8 = a + (8 − 1) d = 55 i.e. a + 7d = 55 (2) − (1): a + 4d = 37

i.e.

( 1) (2)

3d = 18 d=6

Put d = 6 in (1): a + 4 (6) = 37 a + 24 = 37 a = 13 So d = 6 and a = 13.

7.4 Exercises 1.

The following series are arithmetic. Evaluate all pronumerals. (a) 5 + 9 + y + … (b) 8 + 2 + x + … (c) 45 + x + 99 + … (d) 16 + b + 6 + … (e) x + 14 + 21 + …

(f) 32 + (x − 1) + 51 + … (g) 3 + (2k + 3) + 21 + … (h) x + (x + 3) + (2x + 5) + … (i) (t − 5) + 3t + (3t + 1) + … (j) (2t − 3) + (3t + 1) + (5t + 2) + …

Chapter 7 Series

2.

3.

4.

Find the 15th term of each series. (a) 4 + 7 + 10 + … (b) 8 + 13 + 18 + … (c) 10 + 16 + 22 + … (d) 120 + 111 + 102 + … (e) − 3 + 2 + 7 + … Find the 100th term of each series. (a) − 4 + 2 + 8 + … (b) 41 + 32 + 23 + … (c) 18 + 22 + 26 + … (d) 125 + 140 + 155 + … (e) −1 − 5 − 9 − … What is the 25th term of each series? (a) −14 − 18 − 22 − … (b) 0.4 + 0.9 + 1.4 + … (c) 1.3 + 0.9 + 0.5 + … 1 (d) 1 + 2 + 4 + … 2 3 2 (e) 1 + 2 + 2 + … 5 5

5.

For the series 3 + 5 + 7 + … , write an expression for the nth term.

6.

Write an expression for the nth term of the following series. (a) 9 + 17 + 25 + … (b) 100 + 102 + 104 + … (c) 6 + 9 + 12 + … (d) 80 + 86 + 92 + … (e) − 21 − 17 − 13 − … (f) 15 + 10 + 5 + … 7 1 (g) + 1 + 1 + … 8 8 (h) − 30 − 32 − 34 − … (i) 3.2 + 4.4 + 5.6 + … 1 1 (j) + 1 + 2 +… 4 2

7.

Find which term of 3 + 7 + 11 + … is equal to 111.

8.

Which term of the series 1 + 5 + 9 + … is 213?

9.

Which term of the series 15 + 24 + 33 + … is 276?

10. Which term of the series 25 + 18 + 11 + … is equal to − 73? 11. Is zero a term of the series 48 + 45 + 42 + … ? 12. Is 270 a term of the series 3 + 11 + 19 + … ? 13. Is 405 a term of the series 0 + 3 + 6 +…? 14. Find the first value of n for which the terms of the series 100 + 93 + 86 + … become less than 20. 15. Find the values of n for which the terms of the series − 86 − 83 − 80 − … are positive. 16. Find the first negative term of 54 + 50 + 46 + … . 17. Find the first term that is greater than 100 in the series 3 + 7 + 11 + … . 18. The first term of an arithmetic series is -7 and the common difference is 8. Find the 100th term of the series. 19. The first term of an arithmetic series is 15 and the 3rd term is 31. (a) Find the common difference (b) Find the 10th term of the series. 20. The first term of an arithmetic series is 3 and the 5th term is 39. Find its common difference. 21. The 2nd term of an arithmetic series is 19 and the 7th term is 54. Find its first term and common difference.

271

272

Maths In Focus Mathematics HSC Course

22. Find the 20th term in an arithmetic series with 4th term 29 and 10th term 83. 23. The common difference of an arithmetic series is 6 and the 5th term is 29. Find the first term of the series. 24. If the 3rd term of an arithmetic series is 45 and the 9th term is 75, find the 50th term of the series. 25. The 7th term of an arithmetic series is 17 and the 10th term is 53. Find the 100th term of the series.

27. (a) Show that 3 + 12 + 27 + . . . is an arithmetic series. (b) Find the 50th term in simplest form. 28. Find the 25th term of log2 4 + log2 8 + log2 16 + … 29. Find the 40th term of 5b + 8b + 11b + … 30. Which term is 213y of the series 28y + 33y + 38y + …?

26. (a) Show that log5 x + log5 x2 + log5 x3 + … is an arithmetic series. (b) Find the 80th term. (c) If x = 4, evaluate the 10th term correct to 1 decimal place.

Partial sum of an arithmetic series The sum of the first n terms of an arithmetic series (nth partial sum) is given by the formula: Sn =

n (a + l) where l = last or nth term 2

Proof Let the last or nth term be l.

`

S n = a + ( a + d ) + ( a + 2d ) + … + l Sn = l + (l − d ) + (l − 2d ) + … + a 2Sn = (a + l ) + (a + l ) + (a + l ) + … + (a + l ) = n (a + l ) n S n = (a + l ) 2

In general,

Sn =

n [ 2 a + ( n − 1) d ] 2

( 1) (2) (1) + (2)

Chapter 7 Series

Proof Since l = n th term, l = a + ( n − 1) d n (a + l) 2 n = [ a + a + ( n − 1 ) d] 2 n = [2a + (n − 1) d] 2 We use this formula when the nth term is unknown. `

Sn =

EXAMPLES 1. Evaluate 9 + 14 + 19 + … + 224.

Solution a = 9, d = 5 First we find n. Tn = 224 ` Tn = a + (n − 1) d 224 = 9 + (n − 1) 5 = 9 + 5n − 5 = 5n + 4 220 = 5n 44 = n n ` S n = (a + l ) 2 44 S44 = (9 + 224) 2 = 22 ´ 233 = 5126 2. For what value of n is the sum of n terms of 2 + 11 + 20 + … equal to 618?

Solution a = 2, d = 9, Sn = 618 n Sn = [2a + (n − 1) d] 2 n 618 = [2 ´ 2 + (n − 1) 9] 2 1236 = n (4 + 9n − 9) = n (9n − 5) = 9n2 − 5n CONTINUED

273

274

Maths In Focus Mathematics HSC Course

n must be a positive integer.

0 = 9n2 − 5n − 1236 = (n − 12) (9n + 103) ` n = 12 or −11.4 But n cannot be negative, so n = 12. 3. The 6th term of an arithmetic series is 23 and the sum of the first 10 terms is 210. Find the sum of 20 terms.

Solution Tn = a + (n − 1) d T6 = a + (6 − 1) d = 23 a + 5d = 23 n S n = [ 2 a + ( n − 1 ) d] 2 10 [2a + (10 − 1) d] = 210 S10 = 2 5 (2a + 9d) = 210 2a + 9d = 42 (1) ´ (2):

2a + 10d = 46

(2) − (3):

−d = − 4 d=4 Substitute d = 4 in (1): a + 5 (4) = 23 a + 20 = 23 a=3 20 S20 = [2 (3) + (20 − 1) 4] 2 = 10 [6 + 19 (4)] = 10 ´ 82 = 820 50

4. Evaluate

/ 3r + 2.

r =1

Solution It would take a long time to add these up!

50

/ 3r + 2 = (3 ´ 1 + 2) + (3 ´ 2 + 2) + (3 ´ 3 + 2) + … + (3 ´ 50 + 2)

r =1

= 5 + 8 + 11 + … + 152

Arithmetic series with a = 5, d = 3, l = 152, n = 50 n S n = (a + l) 2 50 S50 = (5 + 152) 2 = 25 ´ 157 = 3925

(1)

(2 ) (3 )

Chapter 7 Series

7.5 Exercises 1.

2.

Find the sum of 15 terms of the series. (a) 4 + 7 + 10 + … (b) 2 + 7 + 12 + … (c) 60 + 56 + 52 + … Find the sum of 30 terms of the series. (a) 1 + 7 + 13 + … (b) 15 + 24 + 33 + … (c) 95 + 89 + 83 + …

3.

Find the sum of 25 terms of the series. (a) − 2 + 5 + 12 + … (b) 5 − 4 − 13 − …

4.

Find the sum of 50 terms of the series. (a) 50 + 44 + 38 + … (b) 11 + 14 + 17 + …

5.

Evaluate (a) 15 + 20 + 25 + … + 535 (b) 9 + 17 + 25 + … + 225 (c) 5 + 2 − 1 − … − 91 (d) 81 + 92 + 103 + … + 378 (e) 229 + 225 + 221 + … + 25 (f) − 2 + 6 + 14 + … + 94 (g) 0 − 9 − 18 − … − 216 (h) 79 + 81 + 83 + … + 229 (i) 14 + 11 + 8 + … − 43 3 1 1 (j) 1 + 1 + 2 + … + 25 4 4 2

6.

Evaluate

7.

How many terms of the series 45 + 47 + 49 + … give a sum of 1365?

8.

For what value of n is the sum of the arithmetic series 5 + 9 + 13 + … equal to 152?

9.

How many terms of the series 80 + 73 + 66 + … give a sum of 495?

10. The sum of the first 5 terms of an arithmetic series is 110 and the sum of the first 10 terms is 320. Find the first term and the common difference. 11. The sum of the first 5 terms of an arithmetic series is 35 and the sum of the next 5 terms is 160. Find the first term and the common difference. 12. Find S25, given an arithmetic series whose 8th term is 16 and whose 13th term is 81. 13. The sum of 12 terms of an arithmetic series is 186 and the 20th term is 83. Find the sum of 40 terms. 14. How many terms of the series 20 + 18 + 16 + … give a sum of 104?

20

(a)

/ 4n − 7

n =1

(b)

15

/ 5 − 3r

r =1 20

(c)

/ 4 − 6r

r =3 50

(d)

/ 5n + 3

n =1 40

(e)

/ 4 − 3n 5

15. The sum of the first 4 terms of an arithmetic series is 42 and the sum of the 3rd and 7th term is 46. Find the sum of the first 20 terms. 16. (a) Show that (x + 1) + (2x + 4) + (3x + 7) + … is an arithmetic series. (b) Find the sum of the first 50 terms of the series.

275

276

Maths In Focus Mathematics HSC Course

17. The 20th term of an arithmetic series is 131 and the sum of the 6th to 10th terms inclusive is 235. Find the sum of the first 20 terms. 18. The sum of 50 terms of an arithmetic series is 249 and the sum of 49 terms of the series is 233. Find the 50th term of the series.

19. Prove that Tn = Sn − Sn − 1 for any series. 20. Find the sum of all integers between 1 and 100 that are not multiples of 6.

Class Investigation Look at the working out of question 14 in the previous set of exercises. Why are there two values for n?

DID YOU KNOW? Here is an interesting series: π 1 1 1 1 1 = − + − + 4 5 7 1 3 9

− ….

Gottfried Wilhelm Leibniz (1646–1716) discovered this result. It is interesting that while π is an irrational number, it can be written as the sum of rational numbers.

Geometric Series In a geometric series each term is formed by multiplying the preceding term by a constant. The constant is called the common ratio.

EXAMPLE Find the common ratio of the series 3 + 6 + 12 + …

Solution By looking at this sequence, each term is multiplied by 2 to get the next term. If you can’t see this, we divide the terms as follows: 6 12 = =2 3 6

Chapter 7 Series

277

If T1 + T2 + T3 + … is a geometric series then T2 T3 = r= T1 T2 In general, if T1 + T2 + T3 + T4 + … + Tn − 1 + Tn is a geometric series then r=

T2 T1

=

T3 T2

=

T4 T3

=…=

Tn Tn − 1

.

EXAMPLES 1. Find x if 5 + x + 45 + … is a geometric series.

Solution For a geometric series x 45 = x 5 x2 = 225

T2 T1

=

T3 T2

x = ± 225 = ±15 If x = 15 the series is 5 + 15 + 45 + … (r = 3) . If x = −15, the series is 5 − 15 + 45 − … (r = − 3) .

2. Is

1 1 1 + + + … a geometric series? 4 6 18

Solution T2 T1

T3 T2

1 1 ÷ 6 4 1 4 = ´ 6 1 2 = 3 =

1 1 ÷ 18 6 6 1 = ´ 18 1 1 = 3 T2 ≠ T1 =

∴ the series is not geometric.

x is called the geometric mean.

When r is negative, the signs alternate.

278

Maths In Focus Mathematics HSC Course

Terms of a geometric series

a + ar + ar2 + ar3 + … + arn − 1 + … is a geometric series with first term a, common ratio r and nth term given by Tn = ar n − 1

Proof Let the first term of a geometric series be a and the common ratio be r. Then first term is a second term is ar third term is ar 2 fourth term is ar 3, and so on nth term is ar n − 1

EXAMPLES 1. (a) Find the 10th term of the series 3 + 6 + 12 + … (b) Write an expression for the nth term of the series.

Solution (a) This is a geometric series with a = 3 and r = 2. We want the 10th term, so n = 10. Tn = ar n − 1 T10 = 3 (2)10 −1 = 3 ´ 29 = 1536 (b) Tn = ar n − 1 = 3 ( 2) n − 1 8 2 4 2. Find the common ratio of + + + … and hence find the 8th 3 15 75 term in index form.

Solution 8 4 2 4 ÷ c= ÷ m 75 15 15 3 3 4 ´ = 15 2 2 = 5

r=

Chapter 7 Series

279

Tn = ar n − 1 2 2 8−1 c m 3 5 2 2 7 = c m 3 5 28 = 3 ´ 57

T8 =

Terms can become very large in geometric series.

3. Find the 10th term of the series − 5 + 10 − 20 + …

Solution a = − 5, r = − 2, n = 10 Tn = ar n − 1 T10 = − 5 (− 2)10 − 1 = − 5 (− 2)9 = − 5 (− 512) = 2560 4. Which term of the series 4 + 12 + 36 + … is equal to 78 732?

Solution This is a geometric series with a = 4 and r = 3. The nth term is 78 732. Tn = ar n − 1 78 732 = 4 (3)n − 1 19 683 = 3n − 1 log10 19 683 = log10 3n − 1 = (n − 1) log 3 log10 19 683 = n −1 log10 3 9 = n −1 10 = n

You could use logarithms to the base e or ln instead of logarithms to the base 10.

So the 10th term is 78 732. CONTINUED

280

Maths In Focus Mathematics HSC Course

5. The third term of a geometric series is 18 and the 7th term is 1458. Find the first term and the common ratio.

Solution Tn = ar n − 1 T3 = ar 3 − 1 = 18 ar 2 = 18 T7 = ar Always divide the equations in geometric series to eliminate a.

7−1

(1)

= 1458

ar = 1458 6

(2)

(2) ¸ (1): r4 = 81

Substitute

r = ± 4 81 = ±3 r = ±3 into (1):

a (±3)2 = 18 9a = 18 a=2 1 6. Find the first value of n for which the terms of the series + 1 + 5 + … 5 exceed 3000.

Solution 1 ,r = 5 5 For terms to exceed 3000, Tn > 3000

a=

ar n − 1 > 3000

n must be an integer.

1 n−1 (5 ) > 3000 5 5n − 1 > 15 000 log 5n − 1 > log 15 000 (n − 1) log 5 > log 15 000 log 15 000 n −1 > log 5 log 15 000 n> +1 log 5 n > 6.97 ` n=7 So the 7th term is the first term to exceed 3000.

Chapter 7 Series

7.6 Exercises 1.

2.

3.

Are the following series geometric? If they are, find the common ratio. (a) 5 + 20 + 60 + … 1 (b) − 4 + 3 − 2 + … 4 3 3 3 + +… (c) + 4 14 49 5 1 (d) 7 + 5 + 3 + … 6 3 (e) − 14 + 42 − 168 + … 8 1 8 (f) 1 + + +… 3 9 27 (g) 5.7 + 1.71 + 0.513 + … 81 1 7 (h) 2 − 1 + +… 4 20 100 7 (i) 63 + 9 + 1 + … 8 7 (j) −1 + 15 − 120 + … 8 Evaluate all pronumerals in these geometric series. (a) 4 + 28 + x + … (b) − 3 + 12 + y + … (c) 2 + a + 72 + … (d) y + 2 + 6 + … (e) x + 8 + 32 + … (f) 5 + p + 20 + … (g) 7 + y + 63 + … (h) − 3 + m − 12 + … (i) 3 + (x − 4) + 15 + … (j) 3 + (k − 1) + 21 + … 1 1 (k) + t + + , … 4 9 1 4 (l) + t + +,… 3 3 Write an expression for the nth term of the following series. (a) 1 + 5 + 25 + … (b) 1 + 1.02 + 1.0404 + … (c) 1 + 9 + 81 + … (d) 2 + 10 + 50 + … (e) 6 + 18 + 54 + … (f) 8 + 16 + 32 + … 1 (g) + 1 + 4 + … 4

(h) 1000 − 100 + 10 − … (i) − 3 + 9 − 27 + … 1 2 4 (j) + + +… 3 15 75 4.

Find the 6th term of each series. (a) 8 + 24 + 72 + … (b) 9 + 36 + 144 + … (c) 8 − 32 + 128 + … (d) −1 + 5 − 25 + … 8 2 4 (e) + + +… 3 9 27

5.

What is the 9th term of each series? (a) 1 + 2 + 4 + … (b) 4 + 12 + 36 + … (c) 1 + 1.04 + 1.0816 + … (d) − 3 + 6 − 12 + … 3 3 3 −… (e) − + 4 8 16

6.

Find the 8th term of each series. (a) 3 + 15 + 75 + … (b) 2.1 + 4.2 + 8.4 + … (c) 5 − 20 + 80 − … 3 9 1 (d) − + − +… 2 10 50 10 5 47 (e) 1 +2 + 3 +… 81 27 9

7.

Find the 20th term of each series, leaving the answer in index form. (a) 3 + 6 + 12 + … (b) 1 + 7 + 14 + … (c) 1.04 + 1.042 + 1.043 + … 1 1 1 (d) + + +… 4 8 16 3 9 27 (e) + + +… 4 16 64

281

282

Maths In Focus Mathematics HSC Course

8.

Find the 50th term of 1 + 11 + 121 + … in index form.

9.

Which term of the series 4 + 20 + 100 + … is equal to 12 500?

10. Which term of 6 + 36 + 216 + … is equal to 7776? 11. Is 1200 a term of the series 2 + 16 + 128 + …? 12. Which term of 3 + 21 + 147 + … is equal to 352 947? 13. Which term of the series 1 8 − 4 + 2 − … is ? 128 14. Which term of 54 + 18 + 6 + … 2 is ? 243 15. Find the value of n if the nth term of the series 81 1 1 . − 2 + 1 − 1 + … is − 2 128 8

16. The first term of a geometric series is 7 and the 6th term is 1701. Find the common ratio. 17. The 4th term of a geometric series is −648 and the 5th term is 3888. (a) Find the common ratio. (b) Find the 2nd term. 18. The 3rd term of a geometric series 3 2 is and the 5th term is 1 . Find 5 5 its first term and common ratio. 19. Find the value of n for the first term of the series 5000 + 1000 + 200 + … that is less than 1. 20. Find the first term of the series 2 6 4 + + 2 + … that is greater 7 7 7 than 100.

Partial sum of a geometric series The sum of the first n terms of a geometric series (nth partial sum) is given by the formula:

a ( r n − 1) for |r | > 1 r−1 a (1 − r n ) for |r | < 1 Sn = 1−r Sn =

You studied absolute values in the Preliminary Course | r | < 1 means − 1 < r < 1. What does | r | > 1 mean?

Proof The sum of a geometric series can be written Sn = a + ar + ar 2 + . . . + ar n − 1 rSn = ar + ar 2 + ar 3 + . . . + ar n (1) − (2):

Sn (1 − r) = a − ar n = a (1 − r n ) a (1 − r n ) Sn = 1−r

The two formulae are the same, just rearranged.

(2) − (1) gives the formula a (r n − 1 ) Sn = r−1

(1) (2)

Chapter 7 Series

283

EXAMPLES 1. Find the sum of the first 10 terms of the series 3 + 12 + 48 + …

Solution This is a geometric series with a = 3, r = 4 and we want n = 10. Since r > 1 we use the first formula. a (r n − 1 ) Sn = r−1 3 (410 − 1) = 4−1 3 (410 − 1) = 3 10 =4 −1 = 1 048 575 11

2. Evaluate / 2n . 3

Solution 11

/ 2n = 23 + 24 + 25 + . . . + 211 3

= 8 + 16 + 32 + . . . + 2048

Can you see why n = 9? Count the terms carefully.

Geometric series with a = 8, r = 2, n = 9 a (r n − 1 ) Sn = r−1 8 (2 9 − 1 ) S9 = 2−1 8 (512 − 1) = 1 = 8 ´ 511 = 4088 3. Evaluate 60 + 20 + 6

20 2 +…+ . 3 81

Solution 20 1 ,T = 3 n 81 20 Tn = ar n − 1 = 81 a = 60, r =

First find n.

CONTINUED

284

Maths In Focus Mathematics HSC Course

1 n − 1 20 60 c m = 3 81 1 n−1 1 c m = 3 243 1 1 = n−1 243 3 1 1 = 3n − 1 35 `n−1 =5 n=6 a (1 − r n ) 1−r 1 6 60 = 1 − c m G 3 S6 = 1 1− 3 1 60 c 1 − m 729 = 2 3 3 728 = 60 c m´ 729 2 71 = 89 81 Sn =

4. The sum of n terms of 1 + 4 + 16 + … is 21 845. Find the value of n.

Solution a = 1, r = 4, Sn = 21 845 a (r n − 1 ) r−1 1 (4 n − 1 ) 21 845 = 4−1 4n − 1 = 3 n 65 535 = 4 − 1 65 536 = 4n 48 = 4n ` n=8 So 8 terms gives a sum of 21 845. Sn =

Chapter 7 Series

7.7 Exercises 1.

2.

3.

4.

5.

Find the sum of 10 terms of the series. (a) 6 + 24 + 96 + … (b) 3 + 15 + 75 + … Find the sum of 8 terms of the series. (a) −1 + 7 − 49 + … (b) 8 + 24 + 72 + … Find the sum of 15 terms of the series. (a) 4 + 8 + 16 + … 3 3 3 −… (b) − + 4 8 16 Evaluate (a) 2 + 10 + 50 + … + 6250 9 1 (b) 18 + 9 + 4 + … + 2 64 (c) 3 + 21 + 147 + … + 7203 3 3 1 1 (d) + 2 + 6 + … + 182 4 4 4 4 (e) − 3 + 6 − 12 + … + 384 Evaluate (a)

8

/2

n−1

n =1 6

(b)

/ 31n

n =1 10

(c) (d) (e)

/ 5n − 3

n =2 8

/

r =1

1 2r − 1

7

/ 4k + 1

k =1

6.

Find the (a) 9th term (b) sum of 9 terms of the series 7 + 14 + 28 + …

7.

Find the sum of 30 terms of the series 1.09 + 1.092 + 1.093 + f , correct to 2 decimal places.

8.

Find the sum of 25 terms of the series 1 + 1.12 + 1.122 + f , correct to 2 decimal places.

9.

Find the value of n if the sum of n terms of the series 11 + 33 + 99 + … is equal to 108 251.

10. How many terms of the series 1 1 1 + + + … give a sum of 2 4 8 1023 ? 1024 11. The common ratio of a geometric series is 4 and the sum of the first 5 terms is 3069. Find the first term. 12. Find the number of terms needed to be added for the sum to exceed 1 000 000 in the series 4 + 16 + 64 + … 13. Lucia currently earns $25 000. Her wage increases by 5% each year. Find (a) her wage after 6 years (b) her total earnings (before tax) in 6 years. 14. Write down an expression for the series 2 − 10 + 50 − … + 2 (− 5k − 1 ) (a) in sigma notation (b) as a sum of n terms. 15. Find the sum of the first 10 terms of the series 3 + 7 + 13 + … + [2n + (2n − 1)] + …

285

286

Maths In Focus Mathematics HSC Course

PUZZLES

This is a hard one!

1. A poor girl saved a rich king from drowning one day. The king offered the girl a reward of sums of money in 30 daily payments. He gave the girl a choice of payments: Choice 1: $1 the first day, $2 the second day, $3 the third day and so on. Choice 2: 1 cent the first day, 2 cents the second day, 4 cents the third day and so on, the payment doubling each day. How much money would the girl receive for each choice? Which plan would give the girl more money? 2. Can you solve Fibonacci’s Problem? A man entered an orchard through 7 guarded gates and gathered a certain number of apples. As he left the orchard he gave the guard at the first gate half the apples he had and 1 apple more. He repeated this process for each of the remaining 6 guards and eventually left the orchard with 1 apple. How many apples did he gather? (He did not give away any half apples.)

Limiting sum (sum to infinity) In some geometric series the sum becomes very large as n increases. For example, 2 + 4 + 8 + 16 + 32 + 64 + 128 + … We say that this series diverges, or it has an infinite sum. In some geometric series, the sum does not increase greatly after a few terms. We say this series converges and it has a limiting sum (the sum is limited to a finite number).

EXAMPLE 1 1 1 1 + + + + … , notice that after a while the 2 4 8 16 terms are becoming closer and closer to zero and so will not add much to the sum of the whole series. (a) Evaluate, correct to 4 decimal places, the sum of (i) 10 terms (ii) 20 terms. (b) Estimate its limiting sum.

For the series 2 + 1 + Can you estimate the sum of this series?

Chapter 7 Series

Solution (a) a = 2, r =

1 2

(i) n = 10 Sn =

=

a (1 − r n ) 1−r 1 10 2e1 − c m o 2

1 2 1 2 c 1 − 10 m 2 = 1 2 = 3.9961 1−

(ii) n = 20 Sn =

=

a (1 − r n ) 1−r 1 20 2e1 − c m o 2

1 2 1 2 c 1 − 20 m 2 = 1 2 = 4.0000 (b) The limiting sum is 4. 1−

Can you see why the series 2 + 4 + 8 + 16 + … diverges and the series 1 1 2 + 1 + + + … converges? 2 4 The difference is in the common ratio. Only geometric series with common ratios | r | < 1 will converge and have a limiting sum.

a 1−r | r | < 1 is the necessary condition for the limiting sum to exist. S∞ =

287

288

Maths In Focus Mathematics HSC Course

Proof Sn =

a (1 − r n ) 1−r

For | r | < 1, as n increases, r n decreases and approaches zero 1 e.g. When r = : 2 1 10 1 1 20 1 and c m = c m = 2 1024 2 1 048 576 a (1 − 0) ` S∞ = 1−r a = 1−r

EXAMPLES 1. Find the limiting sum of 2 + 1 +

1 1 + +… 2 4

Solution In the previous example, we guessed that the limiting sum was 4. Here we will use the formula to find the limiting sum. 1 a = 2, r = 2 a S∞ = 1−r 2 = 1 1− 2 2 = 1 2 2 =2´ 1 = 4 2. Find the sum of the series 6 + 2 +

Solution 2 2 3 1 a = 6, r = = = 6 2 3

2 +… 3

Chapter 7 Series

a 1−r 6 = 1 1− 3 6 = 2 3 3 =6´ 2 =9

S∞ =

3. Which of the following series have a limiting sum? 3 15 11 (a) + +1 +… 4 16 64 (b) 100 + 50 + 25 + … (c) 3 + 2

1 11 +1 +… 4 16

Solution 15 11 1 64 16 (a) r = = 15 3 4 16 1 =1 4 >1 So this series does not have a limiting sum. 50 25 = 100 50 1 = 2 0, 2 > 0 dx dx

C

O

D

(b)

A

(c) (d)

B

Given O is the centre of the circle, triangles OAB and OCD can be proven to be congruent using the test (a) SSS (b) SAS (c) AAS (d) RHS. 48. The area bounded by the curve y = x3, the x-axis and the lines x = − 2 and x = 2 is (a) 8 units2 (b) 2 units2 (c) 0 units2 (d) 4 units2. 49.

y

x

dy dx dy dx dy dx

> 0, < 0, < 0,

d2 y dx2 d2 y dx2 d2 y dx2

0 − 1 (c) − 4 < x < 2 (d) x < − 4, x > 2 48. I borrow $10 000 over 5 years at 1.85% monthly interest. How much do I need to pay each month? 49. Find the probability of drawing out a blue and a white ball from a bag containing 7 blue and 5 white balls if the first ball is not replaced before taking out the second. 70 (a) 121 70 (b) 144 1225 (c) 17 424 70 (d) 132 50. The limiting sum of a geometric series exists when (a) | r | > 1 (b) | r | < 1 (c) | r | ≥ 1 (d) | r | ≤ 1

pat_3.indd 350

51. Find the limiting sum of the series 2 1 3 + + + ... 3 2 8 (a) 2 1 (b) 1 3 2 (c) 2 3 8 (d) 9 52. Which statement is the same as 3x = 7? There may be more than one answer. 7 (a) x = log 3 (b) log3 x = 7 (c) log3 7 = x log 7 (d) x = log 3 53. The nth term of the series 7 + 49 + 343 + … is (a) 7n (b) 7n − 1 (c) 7n − 1 (d) 7n 54. An amount of $6000 is invested at 3.5% p.a. with interest paid quarterly. Find the balance after 10 years. (a) $8501.45 (b) $8463.59 (c) $6546.16 (d) $6899.96 55. The limiting sum of

3 2 4 + + + . . . is 5 5 15

1 5 9 (b) 10 4 (c) 1 5 6 (d) 15 (a)

7/16/09 5:28:11 PM

PRACTICE ASSESSMENT TASK 3

56. In a group of 25 students, 19 catch a train to school and 21 catch a bus. If one of these students is chosen at random, find the probability that the student only catches a bus to school. 6 (a) 25 21 (b) 25 3 (c) 5 3 (d) 20

pat_3.indd 351

351

57. The formula for the sum 1 + 1.03 + 1.032 + . . . + 1.03n − 1 is 1.03(1.03n − 1 − 1) 1.03 − 1 1.03(1.03n − 1) (b) S = 1.03 − 1 1.03n − 1 − 1 (c) S = 1.03 − 1 1.03n − 1 (d) S = 1.03 − 1 (a) S =

7/16/09 5:28:13 PM

352

Maths In Focus Mathematics HSC Course

Sample Examination Papers This chapter contains two sample Mathematics HSC papers. They are designed to give you some practice in working through an examination. These papers contain work from the Year 11 Preliminary Course, as up to 20% of questions are allowed to contain this work. You may need to revise this work before you try these papers. If you can, set yourself a time limit and work under examination conditions. Give yourself 3 hours to do each Mathematics paper. Try not to look up any notes while working through these papers.

(e) Two cities are 3295 km apart. Write this number correct to 2 significant figures. (f) Solve | x + 3| < 7. QUESTION 2 (a)

(i) Copy the diagram into your examination booklet. (ii) Show that AC = CE. (iii) Find the length of DE.

MATHEMATICS—PAPER 1 Time allowed—Three hours (Plus 5 minutes’ reading time)

(b)

DIRECTIONS TO CANDIDATES • • •

All questions may be attempted. All questions are of equal value. All necessary working should be shown in every question, as marks are awarded for this. Badly arranged or careless work may not receive marks.

QUESTION 1 (a) Find, correct to 2 decimal places, the 28.3 . 15.7 ´ 2.4 (b) Factorise 3x2 - 11x + 6. x x−1 (c) Solve the equation − = 5. 2 3 (d) The volume of a cone is given by 1 V = π r 2. If a cone has volume 12 m3, 3 find its radius correct to 2 decimal places. value of

(i) Copy the diagram into your examination booklet. (ii) Use the sine rule to calculate MO correct to 1 decimal place. (iii) Find MP to the nearest metre. QUESTION 3 (a) Differentiate (i) x + 5x 3 + 1 1

(ii) 3 ln x + x (iii) (2x + 3)5 (b) Find (i) (ii)

# (x − e − x ) dx π #0 (sin θ + 1) dθ

SAMPLE EXAMINATION PAPERS

(c) (i)

Rationalise the denominator of

5 . 2−1 (ii) Find integers a and b such that 5 = a + b. 2−1 QUESTION 4 (a) (i) Plot points A(3, 2) and B(-1, 5) on a number plane. (ii) Show that line AB has equation 3x + 4y - 17 = 0. (iii) Find the perpendicular distance from the origin to line AB. (iv) Find the area of triangle OAB where O is the origin. (b)

QUESTION 6 2 (a) A plant has a probability of of 7 producing white flowers. If 3 plants are chosen at random, find the probability that (i) 2 plants will produce white flowers (ii) at least 1 plant will produce white flowers. 3 (b) Solve sin x = for 0° ≤ x ≤ 360°. 2 (c) A particle P moves so that it is initially at rest at the origin, and its acceleration is given by a = 6t + 4 cms-2. (i) Find the velocity of P when t = 5. (ii) Find the displacement of P when t = 2. QUESTION 7 (a)

Find the length of side BC using the cosine rule and give your answer correct to the nearest centimetre. QUESTION 5 (a) Kate earns $24 600 p.a. In the following year she receives a pay increase of $800. Each year after that, she receives a pay increase of $800. (i) What percentage increase did Kate receive in the first year? (ii) What will be Kate’s salary after 12 years? (iii) What will Kate’s total earnings be over the 12 years? (b) A function is given by y = x3 - 3x2 - 9x + 2. (i) Find the coordinates of the stationary points and determine their nature. (ii) Find the point of inflexion. (iii) Draw a sketch of the function.

ABCD is a parallelogram with AD = 1, AB = 2 and +ADX = 60°. AX is drawn so that it bisects DC. (i) Copy the diagram into your examination booklet. (ii) Show that ADX is an equilateral triangle. (iii) Show that AXB is a right-angled triangle. (iv) Find the exact length of side BX. (b) On a number plane, shade in the region given by the two conditions y ³ x2 and x + y £ 3. (c) The number of people P with chickenpox is increasing but the rate at which the disease is spreading is slowing down over t weeks. (i) Sketch a graph showing this information. dP d2 P (ii) Describe and 2 for this dt dt information.

353

354

Maths In Focus Mathematics HSC Course

QUESTION 8 (a) Consider the function given by y = log10 x. (i) Copy and complete the table, to 3 decimal places, in your examination booklet. x

1

y

0

2

3

4

(b) Kim invests $500 at the beginning of each year in a superannuation fund. The money earns 12% interest per annum. If she starts the fund at the beginning of 1996, what will the fund be worth at the end of 2025?

5

(ii) Apply the trapezoidal rule with 4 subintervals to find an approximation, to 2 decimal 5 places, of # log10 x dx.

QUESTION 10 (a)

1

(b) A population of mice at time t in weeks is given by P = P0ekt , where k is a constant and P0 is the population when t = 0. (i) Given that 20 mice increase to 100 after 6 weeks, calculate the value of k, to 3 decimal places. (ii) How many mice will there be after 10 weeks? (iii) After how many weeks will there be 500 mice? QUESTION 9 (a)

The surface area of a cylinder is given by the formula S = 2π r (r + h) . A cylinder is to have a surface area of 160 cm2. (i) Show that the volume is given by V = 80r − π r3. (ii) Find the value of r, to 2 decimal places, that gives the maximum volume. (iii) Find the maximum volume, to 1 decimal place.

(i) Find the exact point of intersection of the curve y = ex and the line y = 4. (ii) Find the exact shaded area enclosed between the curve and the line. (b) (i) The quadratic equation x2 + (k - 1)x + k = 0 has real and equal roots. Find the exact values of k in simplest surd form. (ii) If k = 5, show that x2 + (k - 1)x + k > 0 for all x. (c) A parabola has equation y = x2 + 2px + q. (i) Show that the coordinates of its vertex are (-p, q - p2). (ii) Find the coordinates of its focus. (iii) Find the distance between the point (m, 3m2 + q) and point P vertically below it on the parabola x2 = 8y when q > 0.

SAMPLE EXAMINATION PAPERS

y (m, 3m2 + q)

x2 = 8y

P x y = x2 + 2px + q

(iv) Find the minimum distance between these two points when m + q = 5.

MATHEMATICS—PAPER 2 Time allowed—Three hours (Plus 5 minutes’ reading time) QUESTION 1 (a) Solve | x − 3 | = 5. (b) If f (x) = 5 - x2, find x when f (x) = -4. 5π (c) Find the exact value of sin . 6 3 2 (d) Factorise fully a - 2a - 4a + 8. (e) Simplify 2 24 − 150 . (f) Evaluate loga 50 if loga 5 = 1.3 and loga 2 = 0.43. (g) Find the midpoint of (-3, 4) and (0, -2). QUESTION 2 1 1 Plot points A e 5, − 1 o , B e 1, 1 o and 2 2 C(-4, -1) on a number plane. (a) Show that the equation of line AB is given by 3x + 4y - 9 = 0. (b) Find the equation of the straight line l through C that is perpendicular to AB. (c) Find the point of intersection P of the two lines, AB and l. (d) Find the area of triangle ABC. (e) Find the coordinates of D such that ADCB is a rectangle.

QUESTION 3 (a) Differentiate (i) x cos x (ii) e5x (iii) loge (2x2 − 1) (b) Find the indefinite integral (primitive function) of (i) (3x - 2)4 (ii) 3 sin 2x (c) Evaluate

#0

3

(ex − e− x ) dx. d2y

= 18x − 6. If dx 2 there is a stationary point at (2, −1), find the equation of the curve. (d) For a certain curve,

QUESTION 4 (a) Mark plays a game of chance that has 2 a probability of of winning the game. 5 Hong plays a different game in which 3 there is a probability of of winning. 8 Find the probability that (i) both Mark and Hong win their games (ii) one of them wins the game (iii) at least one of them wins the game. (b) (i) On a number plane, shade the region where y ³ 0, x £ 3 and y £ x + 2. (ii) Find the area of this shaded region. (iii) This area is rotated about the x-axis. Find the volume of the solid formed. (c) (i) Sketch the graph of y = 2x − 1 on a number plane. (ii) Hence solve 2x − 1 < 3. QUESTION 5 (a) For the curve y = 2x3 - 9x2 + 12x - 7 (i) find any stationary points on the curve and determine their nature (ii) find any points of inflexion (iii) sketch the curve in the domain -3 £ x £ 3.

355

356

Maths In Focus Mathematics HSC Course

(b) A plane leaves Bankstown airport and flies for 850 km on a bearing of 100°. It then turns and flies for 1200 km on a bearing of 040°. (i) Draw a diagram showing this information. (ii) How far from the airport is the plane, to the nearest km? (c) Simplify (cosec θ + cot θ )(cosec θ − cot θ ). QUESTION 6 (a) (i) Find the equation of the normal to the curve y = x2 at point P(-2, 4). (ii) This normal cuts the parabola again at point Q. Find the coordinates of Q. (iii) Find the shaded area enclosed between the parabola and the normal, to 3 significant figures. (b) The graph below shows the displacement of a particle over time t.

(ii) Differentiate ln (cos x).

#

π

4

(iii) Hence evaluate tan x dx to 0 2 decimal places. (b) A bridge is 40 metres long, held by wires at A and C with angles of elevation of 47° and 23° as shown.

(i) Find the length of AD to 3 significant figures. (ii) Find the height of the bridge BD to 1 decimal place. (c) Triangle BEC is isosceles with BC = CE. Also +BEC = 50°, +ABE = 130°, and +ADC = 80°.

Prove that ABCD is a parallelogram.

(i) When is the particle at the origin? (ii) When is the particle at rest? (c) The temperature T of a metal is cooling exponentially over t minutes. It cools down from 97°C to 84°C after 5 minutes. Find (i) the temperature after 15 minutes (ii) when it cools down to 20°C. QUESTION 7 (a) (i) Find the area bounded by the curve y = tan x, the x-axis and the lines π x = 0 and x = , by using Simpson’s rule 4 with 5 function values (to 2 decimal places).

QUESTION 8 (a) (i) Sketch the curve y = 3 sin 2x for 0 ≤ x ≤ 2π . (ii) On the same set of axes, sketch x y= . 2 (iii) How many roots does the equation x 3 sin 2x = have in this domain? 2 (b) Solve 2 sin x - 1 = 0 for 0° £ x £ 360°. (c) If logx 3 = p and logx 2 = q, write in terms of p and q (i) logx 12 (ii) logx 2x (d) A stack of oranges has 1 orange at the top, 3 in the next row down, then each row has 2 more oranges than the previous one.

SAMPLE EXAMINATION PAPERS

(i) How many oranges are in the 12th row? (ii) If there are 289 oranges stacked altogether, how many rows are there?

(iii) Find the value of k if the tangent has an x-intercept of 2. (e) Find the area of the sector to 1 decimal place.

QUESTION 9 (a) The diagram below shows the graph of a function y = f (x).

QUESTION 10 (a) The graph of y = (x + 2)2 is drawn below. Copy the graph into your writing booklet.

(i) Copy this diagram into your writing booklet. (ii) On the same set of axes, draw a sketch of the derivative f l(x) of the function. (b) ‘A bag contains yellow, blue and white marbles. Therefore if I choose one marble at random from the bag, the 1 probability that it is blue is .’ 3 Is this statement true or false? Explain why in no more than one sentence. (c) Solve the equation 2 ln x = ln (2x + 3). (d)

(i) Shade the region bounded by the curve, the x-axis and the line x = 1. (ii) This area is rotated about the x-axis. Find the volume of the solid of revolution formed. (b) The accountant at Acme Business Solutions calculated that the hourly cost of running a business car is s2 + 7500 cents where s is the average speed of the car. The car travels on a 3000 km journey. (i) Show that the cost of the journey is given by C = 3000 b s +

The diagram shows the graph of y = ex and the tangent to the curve at x = k. (i) Find the gradient of the tangent at x = k. (ii) Find the equation of the tangent at x = k.

7500 s l.

(ii) Find the speed that minimises the cost of the journey. (iii) Find the cost of the trip to the nearest dollar.

357

358

Maths In Focus Mathematics HSC Course

Answers Chapter 1: Geometry 2

8.

Exercises 1.1 1.

2.

+ABE = 180° −+ABD +CBE = 180° −+CBD = 180° −+ABD = +ABE

(straight angle 180° ) (straight angle)

(corresponding sides in congruent Δs)

(+ABD =+CBD —given)

+DFB = 180° − (180 − x) ° (+AFB is a straight angle) =x ` +AFC = x (vertically opposite angles) +CFE = 180° − (x + 180° − 2x)

` OC bisects AB 9.

(+AFB is a straight angle)

=x ` +AFC = +CFE ` CD bisects +AFE 3.

+WBC + +BCY = 2x + 115 + 65 − 2x = 180° These are supplementary cointerior angles. `VW < XY

4.

x + y = 180° (given) ` +A + +D = 180° These are supplementary cointerior angles. ` AB < DC (similarly) Also +A = +B These are supplementary cointerior angles. ` AD < BC ` ABCD is a parallelogram.

5.

+ADB = +CDB = 110° (given) (BD bisects +ABC) +ABD = +CBD BD is common ` by AAS, Δ ABD / Δ CBD

6.

(a)

AB = AE ( given) +B = +E ( base angles of isosceles Δ) ( given) BC = DE ` by SAS, Δ ABC / Δ AED

10.

(corresponding angles in congruent Δs)

(corresponding angles in congruent Δs)

` AC bisects +DCB 11. (a) +NMO = +MOP (alternate angles, MN < PO) (alternate angles, PM < ON) +PMO = +MON MO is common ` by AAS, Δ MNO / Δ MPO (b) +PMO = +MON (alternate angles, PM < ON) (given) MN = NO (base angles of isosceles Δ) +MON = +NMO ` +PMO = +NMO i.e. +PMQ = +NMQ (c)

MN = NO PM = NO

(given)

(corresponding sides in congruent Δs)

`

PM = MN +PMQ = +NMQ MQ is common ` by SAS, ΔPMQ / ΔNMQ

+ACD = 180° − +BCA (BCD is a straight angle) = 180° − +EDA = +ADC ` since base angles are equal, Δ ACD is isosceles

(corresponding sides in congruent Δs)

AB = AD (given) (given) BC = DC AC is common ` by SSS, ΔABC / ΔADC +DAC = +BAC ` So AC bisects +DAB Also +BCA = +DCA

(corresponding angles in congruent Δs)

DC = BC ( given) ( given) +B = +D = 90° 1 ( given) DM = AD 2 1 and BN = AB 2 ` DM = BN ` by SAS, ΔMDC / ΔNBC ` MC = NC

+CDB = +BEC = 90° (altitudes given) (base angles of isosceles Δ) +ACB = +ABC CB is common ` by AAS, ΔCDB / ΔBEC ` CE = BD (corresponding sides in congruent Δs)

(b) +BCA = +EDA

7.

+OCA = +OCB = 90° (given) (equal radii) OA = OB OC is common ` by RHS, ΔOAC / ΔOBC ` AC = BC

(d)

(from (b))

+MQN = +MQP

(corresponding angles in congruent Δs)

But +MQN ++MQP = 180° (+PQN straight angle) ` +MQN = +MQP = 90° 12.

ANSWERS

Let ABCD be a parallelogram with diagonal AC. +DAC = +ACB (alternate angles, AD < BC) +BAC = +ACD (alternate angles, AB < DC) AC is common ` by AAS, ΔADC / ΔABC

19.

13.

Δ ADC / Δ ABC (see question 12) ` +ADC = +ABC (corresponding angles in congruent Δs) Similarly, by using diagonal BD we can prove +A = +C ` opposite angles are equal 14.

15.

17.

(corresponding sides in congruent Δs)

20.

AB = DC (opposite sides in < gram) (given) BM = DN ` AB − BM = DC − DN i.e. AM = NC Also AM < NC (ABCD is a < gram) Since one pair of sides is both parallel and equal, AMCN is a parallelogram. AD = BC BC = FE ` AD = FE

Let ABCD be a rectangle with +D = 90° `

+DEC = +DCE Also, +DEC = +ECB ` +DCE = +ECB ` CE bisects +BCD

= 90° +B = 180° − 90°

(+B and +C cointerior angles, AB < DC)

= 90° +A = 180° − 90°

(+A and +B cointerior angles, AD < BC)

= 90° ` all angles are right angles 21.

(base angles of isosceles Δ) (alternate angles, AD < BC)

AB = CD (given) (given) +BAC = +DCA AC is common ` by SAS, ΔABC / ΔADC ` AD = BC

+C = 180° − 90°

(+D and +C cointerior angles, AD < BC)

(opposite sides in < gram) (similarly)

Also AD < BC (ABCD is a < gram) (BCEF is a < gram) and BC < FE ` AD < FE Since one pair of sides is both parallel and equal, AFED is a parallelogram. 16.

Let ABCD be a rectangle (opposite sides in < gram) AD = BC +D = +C = 90° DC is common ` by SAS, Δ ADC / Δ BCD ` AC = DB

AD = CD (given) (opposite sides of < gram) AD = BC Also AB = CD (similarly) ` AB = AD = BC = CD ` all sides of the rhombus are equal

22.

(corresponding sides in congruent Δs)

Construct BE < AD (opposite sides of < gram) Then AD = BE (given) But AD = BC Then BE = BC (base angles of isosceles Δ) ` +BCD = +BEC (corresponding angles, AD < BE) Also, +ADC = +BEC ` +ADC = +BCD

Since two pairs of opposite sides are equal, ABCD is a parallelogram. 18. (a)

AE = EC

(diagonals bisect each other in < gram)

+AEB = +CEB = 90° EB is common ` by SAS, Δ ABE / ΔCBE AB = BC `

(given)

(corresponding sides in congruent Δs)

(b) +ABE = +CBE

(corresponding angles in congruent Δs)

23.

AD = AB (given) (given) DC = BC AC is common ` by SSS, ΔADC / ΔΑΒC ` +ADC = +ABC

(corresponding angles in congruent Δs)

359

360

Maths In Focus Mathematics HSC Course

24.

AD = BC (opposite sides of < gram) (given) +D = +C = 90° DE = EC (E is the midpoint—given) ` by SAS, ΔADE / ΔBCE ` AE = BE (corresponding sides in congruent Δs)

Exercises 1.3 1.

2.

AD = BC (opposite sides of < gram) AB = DC (similarly) DB is common ` by SSS, ΔADB / ΔBCD

25. (a)

3.

(a) AB = AC =

4.

Show m XY = mYZ =

5.

(a) Show AB = AD =

(b) +ABE = +CBE

(corresponding angles in congruent Δs)

+AEB = +BEC

(corresponding angles in congruent Δs)

But +AEB + +BEC = 180° (AEC is a straight angle) ` +AEB = +BEC = 90°

Exercises 1.2 1.

(a) 14 452 mm2 (b) 67 200 mm3

3.

V = 2x3 + 3x2 − 2x

4.

V = π r2 h 250 = π r2 h 250 = r2 πh 250 =r πh 6. A =

5.

V = 5π r3

8.

S = 24π h2

9.

V = x ] 3 − 2x g2 = 4x3 − 12x2 + 9x

2.

3

3b 2 2

7. V = ] x + 2 g3 = x3 + 6x2 + 12x + 8

16. V = 19. l = 20. y =

h2 + r2 4π

400 17. h = πr 2

850 − πr2 πr 810 000 − x2

r =1

7.

(a) 2x − 3y + 13 = 0 (b) Substitute (7, 9) into the equation (c) Isosceles

8.

+AOB OD OB OC OA OD ` OB

= +COD = 90° 4 = =2 2 14 = =2 7 OC = OA

(a) OB is common OA = BC = 5 AB = OC = 20 ` by SSS ΔOAB / ΔOCB (b) Show mOA = m BC = 1

1 (6x3 − 5x2 − 34x − 15) 3

x2 y

72 = 6 2,

6.

9.

14. (a) V = 18x3 − 12x2 + 2x (b) S = 54x2 − 30x + 4 15. l =

26 ,

Since 2 pairs of sides are in proportion and their included angles are equal, Δ OAB 0 for all x The function is monotonic increasing for all x.

5

16. (a)

21.

y

2 x 3

ANSWERS

22.

Exercises 2.3

y

x

-2

-1

1.

7x6 − 10x4 + 4x3 − 1; 42x5 − 40x3 + 12x2; 210x4 − 120x2 + 24x; 840x3 − 240x + 24

2.

f m(x) = 72x7

4.

f l(1) = 11, f m(− 2) = 168

5. 6.

2 13 23. (2, 0) and c , 3 m 3 81 24.

x 2 x+1

2 −2 3 p x+1 = ; f− , 9 3 2 x+1 3x + 2

+

25. a = −1.75 26.

1 2 x

!0

27.

7x6 − 12x5 + 16x3; 42x5 − 60x4 + 48x2; 210x4 − 240x3 + 96x dy dx

= 4x − 3,

d2 y

=4

dx2

7.

f l(−1) = −16, f m(2) = 40 8. − 4x− 5, 20x− 6

9.

g m(4) = −

11. x =

−3 !0 x4

3. f l(x) = 10x4 − 3x2, f m(x) = 40x3 − 6x

1 32

7 18

10.

12. x >

d2 h = 26 when t = 1 dt2

1 3

13. 20 (4x − 3)4 ; 320 (4x − 3)3 14. f l(x) = −

Exercises 2.2 1.

(0, −1); yl< 0 on LHS, yl> 0 on RHS

2.

(0, 0) minimum 3. (0, 2) inflexion

4.

(− 2, 11); show f l(x) > 0 on LHS and f l(x) < 0 on RHS.

5.

(−1, − 2) minimum

6. (4, 0) minimum

7.

(0, 5) maximum, (4, − 27) minimum

8.

f l(0) = 0, f l(x) > 0 on LHS and RHS

9.

(0, 5) maximum, (2, 1) minimum

f m(x) = −

1 2 2−x 1

15. f l^ x h = −

16.

;

4 (2 − x) 3 16 96 ; f m(x) = (3x − 1) 2 (3x − 1) 3

d2 v 2 = 24t + 16 17. b = 3 dt2

18. f m(2) =

3 4 2

=

3 2 8

19. f m(1) = 196 20. b = − 2.7

Exercises 2.4

10. (0, − 3) maximum, (1, − 4) minimum, (−1, − 4) minimum

1.

x>−

1 3

2. x < 3 3. y m = − 8 < 0 4. y m = 2 > 0

11. (1, 0) minimum, (−1, 4) maximum

5.

x 0 on RHS 9. − 2 < x < 1

12. m = − 6

1 12

13. x = − 3 minimum

14. x = 0 minimum, x = −1 maximum

15. x = 1 inflexion, x = 2 minimum dP 50 =2− 2 dx x (b) (5, 20) minimum, ^ − 5, − 20 h maximum

16. (a)

17. d 1,

10. (a) No—minimum at (0, 0) (b) Yes—inflexion at (0, 0) (c) Yes—inflexion at (0, 0) (d) Yes—inflexion at (0, 0) (e) No—minimum at (0, 0) 11.

y

1 n minimum 2

18. (2.06, 54.94) maximum, ^ − 20.6, − 54.94 h minimum 19. (4.37, 54.92) minimum, ^ − 4.37, − 54.92 h maximum 20. (a)

dA = dx =

3600 − x2 −

x2 3600 − x2

3600 − 2x2 3600 − x2

(b) (42.4, 1800) maximum, ^ − 42.4, −1800 h minimum

x

365

366

Maths In Focus Mathematics HSC Course

12.

3.

y

x 1 4.

(a)

13. None: (2, 31) is not an inflexion since concavity does not change. 12 x4

14. f m(x) =

x4 > 0 for all x ! 0 12 > 0 for all x ! 0 x4 So the function is concave upward for all x ! 0.

So

(b)

15. (a) (0, 7), (1, 0) and ^ −1, 14 h (b) (0, 7) 16. (a)

d2 y dx2

= 12x2 + 24

x2 ≥ 0 for all x So 12x2 ≥ 0 for all x 12x2 + 24 ≥ 24 So 12x2 + 24 ≠ 0 and there are no points of inflexion. (b) The curve is always concave upwards.

(c)

17. a = 2 18. p = 4 19. a = 3, b = − 3 20. (a) ^ 0, − 8 h, ^ 2, 2 h (b)

dy dx

= 6x5 − 15x4 + 21

At ^ 0, − 8 h:

At

^ 2, 2 h :

dy dx

dy dx

= 6 ] 0 g5 − 15 ] 0 g4 + 21 (d)

= 21 ≠0 = 6 ] 2 g5 − 15 ] 2 g4 + 21

= − 27 ≠0 So these points are not horizontal points of inflexion.

Exercises 2.5 1.

(a) (c) (e)

2.

(a)

dy dx dy dx dy dx

> 0,

d2 y dx2 d y

> 0 (b)

2

> 0, > 0,

dx2 d2 y dx2

< 0 (d)

dy dx dy dx

< 0, < 0,

d2 y dx2 d2 y dx2

5. 0

>0

dP d2 P > 0, < 0 (b) The rate is decreasing. dt dt2

ANSWERS

Exercises 2.7

6.

1.

7. 8.

dM d2 M < 0, >0 dt dt2 (a) The number of fish is decreasing. (b) The population rate is increasing. (c) 2.

9.

The level of education is increasing, but the rate is slowing down.

10. The population is decreasing, and the population rate is decreasing.

Exercises 2.6

3.

1.

(1, 0) minimum 2. (0,1) minimum

3.

(2, − 5), y m = 6 > 0 4. (0.5, 0.25), y m < 0 so maximum

5.

(0, − 5); f m(x) = 0 at (0, − 5), f m(x) < 0 on LHS, f m(x) > 0 RHS

6.

Yes—inflexion at (0, 3)

7.

^ − 2, −78 h minimum, ^ − 3, −77 h maximum

8.

(0, 1) maximum, ^ − 1, − 4 h minimum, ^ 2, − 31 h minimum

9.

(0, 1) maximum, (0.5, 0) minimum, ^ − 0.5, 0 h minimum

10. (a) (4, 176) maximum, (5, 175) minimum (b) (4.5, 175.5) 11. (3.67, 0.38) maximum 12. ^ 0, −1 h minimum, ^ − 2, 15 h maximum, ^ − 4, −1 h minimum 13. (a) a = − 14. m = − 5

2 3

1 2

(b) maximum, as y m < 0 15. a = 3, b = − 9

4.

367

368

Maths In Focus Mathematics HSC Course

9.

5.

6.

10.

7.

11.

8.

(a) ^ 0, −7 h minimum, ^ − 4, 25 h maximum (b) ^ − 2, 9 h (c) 12.

ANSWERS

13.

14.

dy dx

=

−2 ≠0 (1 + x) 2

15.

Exercises 2.8 1.

Maximum value is 4.

2.

Maximum value is 9, minimum value is −7.

3.

Maximum value is 25.

4.

Maximum value is 86, minimum value is −39.

5.

Maximum value is −2.

6.

1 Maximum value is 5, minimum value is − 16 . 3

7.

Absolute maximum 29, relative maximum −3, absolute minimum −35, relative minimum −35, −8

8.

Minimum −25, maximum 29

369

370

Maths In Focus Mathematics HSC Course

9.

Maximum 3, minimum 1

4.

V 400 400 π r2 S

= π r2h = π r2 h =h = 2π r2 + 2π rh = 2π r 2 + 2π r e = 2π r 2 +

5.

10. Maximum ∞, minimum −∞

400 o π r2

800 r

(a) x + y = 30 ` y = 30 − x (b) The perimeter of one square is x, so its side is 1 1 x. The other square has side y. 4 4 1 2 1 2 xn + d yn 4 4 y2 x2 = + 16 16 2 x + y2 = 16 x2 + (30 − x)2 = 16 x2 + 900 − 60x + x2 = 16 2x2 − 60x + 900 = 16 2 (x2 − 30x + 450) = 16 x2 − 30x + 450 = 8

A =d

Problem 30 cm. (This result uses Stewart’s 7 theorem—check this by research.) The disc has radius

6.

Exercises 2.9 1.

A 50 50 ` x P

y=

= xy = xy

= x 78 400 − x2

78 400 − x2

7.

= 2 x + 2y

= 2x +

3.

(b) A = xy

=y

= 2x + 2 ´

2.

(a) x2 + y2 = 2802 = 78 400 y2 = 78 400 − x2

2x + 2y 2y y A

50 x

= 120 = 120 − 2x = 60 − x = xy = x (60 − x) = 60x − x2

xy = 20 20 y= x S =x+y 20 =x+ x

V = x (10 − 2x) (7 − 2x) = x (70 − 20x − 14x + 4x2) = x (70 − 34x + 4x2) = 70x − 34x2 + 4x3

100 x

8.

Profit per person = Cost − Expenses = (900 − 100x) − (200 + 400x) = 900 − 100x − 200 − 400x = 700 − 500x For x people, P = x (700 − 500x) = 700x − 500x2

ANSWERS

(b) 63.2 m by 63.2 m (c) $12 332.89

9.

4 m by 4 m

8.

x = 1.25 m, y = 1.25 m

9.

(a) V = x (30 − 2x) (80 − 2x) = x (2400 − 220x + 4x2) = 2400x − 220x2 + 4x3 (b) x = 6

2 cm 3

(c) 7407.4 cm3

After t hours, Joel has travelled 75t km. He is 700 − 75t km from the town. After t hours, Nick has travelled 80t km. He is 680 − 80t km from the town. d= =

(700 − 75t)2 + (680 − 80t)2 490 000 − 105 000t + 5625t2 + 462 400 − 108 800t + 6400t2 = 952 400 − 213 800t + 12 025t2

10. The river is 500 m, or 0.5 km, wide Distance AB: d= =

x2 + 0.52 x2 + 0.25 distance Speed = time distance ` Time = speed x2 + 0.25 t= 5

10. V = πr2h = 54π 54π h= πr 2 54 = 2 r S = 2πr (r + h) 54 = 2π r d r + 2 n r 108π 2 = 2π r + r Radius is 3 m. 11. (a) S = 2π r2 +

17 200 r

(b) 2323.7 m2

12. 72 cm2 13. (a)

Distance BC: d =7−x distance Time = speed 7−x t= 4 So total time taken is: t=

6. 14 and 14 7. -2.5 and 2.5

5.

x2 + 0.25 7 − x + 5 4

xy = 400 400 ` y= x A = (y − 10) (x − 10) = xy − 10y − 10x + 100 400 400 m − 10 c m − 10x + 100 = xc x x 4000 = 400 − − 10x + 100 x 4000 = 500 − 10x − x

(b) 100 cm2

Exercises 2.10

14. 20 cm by 20 cm by 20 cm 15. 1.12 m2

1.

2 s, 16 m 2. 7.5 km

16. (a) 7.5 m by 7.5 m (b) 2.4 m

3.

2x + 2y 2y y A

= 60 = 60 − 2x = 30 − x = xy = x (30 − x) = 30x − x2

17. 301 cm2 18. 160 (1) from (1)

Max. area 225 m2 4.

(a) A = xy = 4000 4000 ` y= x P = 2x + 2 y 4000 m x 8000 = 2x + x

= 2x + 2 c

(1)

from (1)

1 cm2 6

19. 1.68 cm, 1.32 cm

20. d2 = (200 − 80t) 2 + (120 − 60t) 2 = 40 000 − 32 000t + 6400t2 + 14 400 − 14 400t + 3600t2 = 10 000t2 − 46 400t + 54 400 24 km 1 21. (a) d = ] x2 − 2x + 5 g − ] 4x − x2 g (b) unit 2 2 2 = x − 2 x + 5 − 4x + x 2 = 2x − 6x + 5

371

372

Maths In Focus Mathematics HSC Course

22. (a)

y 1 (2πr) where r = 2 2 y 1 1200 = 2x + y + e 2π ´ o 2 2 πy = 2x + y + 2 πy = 2x 1200 − y − 2 y πy 600 − − =x 2 4 2400 − 2y − πy =x 4

(b) 95 km/h (c) $2846

Perimeter = 2x + y +

1 2 πr 2 2400 − 2y − πy

Exercises 2.11 1.

(c)

2.

3.

d t d So t = s 1500 = s s=

Cost of trip taking t hours: C = ] s2 + 9000 g t 1500 = ] s2 + 9000 g s 9000 ´ 1500 = 1500s + s 9000 c m = 1500 s + s

3

x5 − x 3 + 7x + C 5 x3 − x 2 − 3x + C (d) f (x) = 3 (b) f (x) =

+C x− 3 + 2x − 1 + C 3 2 (d) y = − + C x

(b) y = −

x4 x3 − +C 20 3 x4 x2 − +x+C (e) y = 4 3 (c) y =

4.

(a)

2 x3 +C 3

(b) −

x− 2 +C 2 1

(c) −

1 +C 7x 7

(e) −

x− 6 + 2x − 1 + C 6

1

(d) 2x 2 + 6x 3 + C

x4 1 − x 3 + 5x − 4 4

5.

y=

7.

f (1) = 8 8. y = 2x − 3x2 + 19 9. x = 16

6. f (x) = 2x2 − 7x + 11 1 3

10. y = 4x2 − 8x + 7 11. y = 2x3 + 3x2 + x − 2 12. f (x) = x3 − x2 − x + 5 13. f (2) = 20.5

(b) a = 2, b = 4

25. (a)

2x 2

(a) y = x5 − 9x + C

2 =

24. 26 m

x2 +C 2

x2 − 2x + C 2

(e) f (x) =

Substitute ^ −1, 2 h b (−1) + b a b =− +b a 2a = − b + ab = b ] −1 + a g = b ]a − 1 g 2a =b a−1

(a) f (x) = 2x3 −

3

(c) x = 168 m, y = 336 m 23. (a) Equation AB: y = mx + b b = x+b a

x6 − x4 + C 6

(c) f (x) =

2

y oy + 1 π e o 4 2 2 2400y − 2y2 − πy2 πy2 = + 4 8 2 2 4800y − 4y − 2πy πy 2 = + 8 8 4800y − 4y2 − πy2 = 8

(b)

(e) 6x + C

(b) A = xy + =e

x3 + 4x 2 + x + C 3 x3 − x2 + x + C (d) 3

(a) x2 − 3x + C

14. y =

x3 x2 1 + − 12x + 24 3 2 2

16. y =

x3 2 − 2 x 2 + 3x − 4 3 3

18. y = 3x2 + 8x + 8

15. y =

4x 3 1 − 15x − 14 3 3

17. f (x) = x4 − x3 + 2x2 + 4x − 2

19. f ] − 2 g = 77 20. y = 0

Test yourself 2 1.

(− 3, −11) maximum, (−1, −15) minimum

2.

x >1

4.

(a) -8

6.

(0, 0) minimum 7. x > −1

1 6

3. y = 2x3 + 6x2 − 5x − 33 (b) 26

(c) -90

5. 50 m

ANSWERS

8.

15. y = x3 + 3x2 + 3

(a) (0, 1) maximum, (− 4, − 511) minimum, (2, −79) minimum

14. x < 3

(b)

17. For decreasing curve,

dy dx

16. 150 products

0 on LHS and RHS (b) y m < 0 on LHS, y m > 0 on RHS

2.

19. 21

1 cm3 3

(b) k = 2, 4, 6, 8, . . .

20. (a) (0, 1)

21. minimum −1; maximum −

1 5

22. 87 kmh−1

Chapter 3: Integration

Exercises 3.1 3.

1 x 4 2

4.

16 m2

5.

27; -20.25

6.

f l(0.6) = f m(0.6) = 0 and concavity changes

7.

Show sum of areas is least when r = s = 12.5

8.

25

9.

(a)

dx

2.5 2. 0.39

10

3.

7. 0.41

11. 0.94 12. 0.92

2.4 8.

4. 0.225 5.

1.08

9.

(a) 28 (b) 22

0.75

10. 0.65

13. 75.1 14. 16.5 15. 650.2

Exercises 3.2

5 6 dy

1. 6.

1. 48.7 2. 30.7 3. 1.1 4. 0.41 5. (a) 3.4475 (b) 3.4477 6. 2.75 7. 0.693 8. 1.93 9. 72 10. 5.25 11. 0.558 12. 0.347 13. 3.63 14. 7.87 15. 175.8

Exercises 3.3 =

1 2 x−1

≠0

(b) Domain: x ≥ 1; range: y ≥ 0 (c)

1.

8

2. 10 3. 125

8.

16

9. 50 10. 52

14. 4 20. 6

2 3

2 9

15. 1

1 4

21. 101

25. 0.0126

2 3 1 16. 4 3

1 4

-1

4.

11.

2 3

5.

10

12. 21

17. 0 18. 2

22. −12

3 4

23. 22

2 3

1 3

6. 54 1 4

13. 0 19. 0

24. 2

1 3

7.

3

1 3

ANSWERS

Exercises 3.4

2.

1.

x3 +C 3

5.

t3 − 7t + C 3

6.

h8 + 5h + C 8

7.

8.

x 2 + 4x + C

9.

b3 b2 + +C 3 2

10.

11.

x3 + x 2 + 5x + C 3

x6 +C 2

x5 x4 + +C 5 2

13. x6 +

15.

2.

3.

2x 5 +C 5

m2 +m+C 2

4. y2 2

(g) 4

− 3y + C

2 3

(h) −

1 8

1 4

(c) −

(i) 1

1 5

1 8

(j)

(d) 60

2 3

(e)

Exercises 3.6 1 units2 3

1 units2 6

x 8 3x 7 − − 9x + C 7 8

9.

8 units2 10. 24.25 units2 11. 2 units2 12. 9

a4 a2 − −a+C 4 2

13. 11

x6 x4 + + 4x + C 4 6

17. 5

3x 5 x 4 x 2 18. − + +C 5 2 2

20. 3x 4 5x 3 + − 4x + C 2 3

20. − x− 3 −

x− 2 − 2x − 1 + C 2

2. 36 units2 3. 4.5 units2 4. 10

6. 14.3 units2 7. 4 units2 8. 0.4 units2

2 units2 3

1 units2 3

14.

1 units2 6

2 units2 3

15.

16.

19. π = 3.14 units2

18. 18 units2

a4 units2 2

4

3x 3

24. x − 2x2 +

4x 3 +C 3

1 +C 2x 2

28. −

27. − 29.

y3 3

+

y− 6 6

4

2 x +C 5

23.

x4 − x3 + x2 + C 4

x 3 3x 2 + − 10x + C 3 2

26. −

3 4 7 −x+ 2 − 4 +C x 4x 2x t4 t3 30. − − 2t2 + 4t + C 4 3 3 3 x4 33. +C 4

1 32. − 4 + C 2t 35.

(b) (d) (f)

5

+C

(3y − 2)8 24 (7x + 8)13 91

(c)

+C

(4 + 3x)5 15

(g) −

(1 − x)7 7

(i) −

(j) − 3 (x + 7) − 1 + C

(k) −

(l)

(n)

3

3

(4x + 3)

4

16

+C

5 2 (t + 3) +C 5

(3x − 4)3 9

+C

50

+C

3

(ii)

(5x − 1)10

(e)

+C

(2x − 5)3

(h)

1 units2 3

1.

21

4.

1.5 units2 5. 1

7.

10

2 units2 3

2. 20 units2

8.

3. 4

2 units2 3

1 units2 4

6. 2

1 units2 3

1 units2 6

9. 3

7 units2 9

10. 2 units2 11. 11

1 units2 4

12. 60 units2

13. 4.5 units2 14. 1

1 units2 3

15. 1.9 units2

Exercises 3.8

(a) (i) 3x3 − 12x2 + 16x + C (x + 1)5

3 +C x

3

Exercises 3.5 1.

Exercises 3.7

2 x +x+C 3

5

34.

+C

25.

+ 5y + C

2 x3 31. +C 3

2 units2 3

5.

16.

+C +C

2 (3x + 1)− 3 9

+C

1 +C 16 (4x − 5) 2 1 2

(m) − 2 (2 − x) + C (o)

7 2 (5x + 2) +C 35

1 7

3 5

12. x4 − x3 + 4x2 − x + C

x4 x3 x2 + − − 2x + C 2 3 2

22.

(f)

1

14.

1 21. − 7 + C 7x

1 6

1.

4x 3 5x 2 17. − − 8x + C 3 2 19.

(a) 288.2 (b) − 1

+C

1 units2 3

2. 1

1 units2 3

1.

1

4.

10

7.

2 units2 3

8. 166

10.

2 units2 3

11.

2 units2 3

13. 36 units2

Problem 206π units3 15

5. 20

5 units2 6

2 units2 3

1 units2 12

14. 2

3.

2 units2 3

1 units2 6 6. 8 units2

9. 0.42 units2 12.

1 units2 3

15. (π − 2) units2

1 units2 3

1 units2 3

375

376

Maths In Focus Mathematics HSC Course

Exercises 3.9

6.

243π units3 5

1.

2.

3.

376π units3 15

4.

7.

2π units3 3

992π units3 5

8.

π units3 7

5.

39π units3 2

6.

5π units3 3

10.

9.

3 units2 4 206π units3 12. + C 13. 3 units2 14. (a) 84 15 π 2 (b) units3 15. (a) x = ± y − 3 (b) 3 units2 3 2

8.

485π units3 3 758π units3 3 9π units3 2

3 units2 7. 1.1 units2 2 units2 3 (7x + 3)12

(c)

11.

27π units3 2

12.

64π units3 3

13.

16 385π units3 7

16. 36

14.

25π units3 2

15.

65π units3 2

16.

1023π units3 5

19. (a)

5π units3 3 3π units3 20. 5 17.

23.

18. 13π units3 2π units3 5

21.

344π units3 27 72π units3 22. 5 19.

y = r2 − x2 ` y2 = r2 − x2 V = π # y2 dx b

a r

(− r)3 r3 oH n − e − r3 − 3 3 2r 3 − 2r 3 n =πd − 3 3 3 4π r = units3 3

11.

5π units3 2 17. 85

1 units2 3

(2x − 1)5

+C

2

18.

(b)

3π units3 5

x6 +C 8

1 units2 12

2.

(a) Show f (− x) = − f (x) (b) 0 (c) 12 units2

3.

27.2 units3 4. 9 units2 5. (a) 36x3 (x4 − 1)8

(x4 − 1)9

(b)

2π units3 35

(a)

(b)

= π > d r3 −

1 2

1.

−r

x3 r F 3 −r

10. 4

Challenge exercise 3

= π # ] r2 − x2 g dx = π < r2 x −

9. 9π units3

2

+C

36 − 22x (3x2 − 4)2

6.

(a)

9.

f (0) =

(b)

1 8

7. 7.35 units2 8.

2π units3 3

1 =∞ 0

10. (a)

Test yourself 3 1. 2.

(a) 0.535 (b) 0.5 3x 2 5x 2 + x + C (b) −x+C (a) 2 2 (d)

(2x + 5)8

3.

14.83

5.

(a)

16

(c)

2 x3 +C 3 (b) 3.08 units2

+C

4. (a) 2 (b) 0 (c) 2

1 5

11.

17 17 units2 6

12.

3x + 6

3 (x + 2)

13. (a)

2 x+3

14. (a) 6 15.

2 x+3

2 2 (b) 6 3 3

5 units2 12 8a 2 units2 3 (b) 2πa3 units3

16. (a) (b)

=

215π units3 6 (b)

2x x + 3 +C 3

ANSWERS

Practice assessment task set 1 1.

13. AB = AC (given) BD = CD (given) AD is common. ` by SSS ΔABD / ΔACD ` +ADB = +ADC (corresponding +s in congruent Δs)

But +ADB + +ADC = 180° (+BDC is a straight +)

Let ABCD be a parallelogram with diagonal AC. +ACD = +BAC (alternate +s, AB < DC) +DAC = +BCA (alternate +s, AD < BC) AC is common ` by AAS ΔACD / ΔACB ` AB = DC and AD = BC

` +ADB = +ADC = 90° ` AD = BC 14. (a) 78.7 units3 (b) 1.57 units3 15. x > −

7 9

16. f (1) = 3, f l(1) = − 2, f m(1) = 18; curve is decreasing and concave upwards at (1, 3)

(corresponding sides in congruent Δs)

` opposite sides are equal 1 2

2.

x<

6.

AC = FD (opposite sides of < gram equal) BC = FE (given) ` AB = AC − BC = FD − FE = ED

3. x3 − x2 + x + C

4. 24

5. 8 m

Also AB < ED (since ACDF is < gram) ` since AB = ED and AB < ED, ABDE is a parallelogram 7.

17. P = 8x + 4y = 4 4y = 4 − 8x y = 1 − 2x A = 3x 2 = 3x 2 = 3x 2 = 7x 2

+ y2 + ] 1 − 2x g2 + 1 − 4x + 4x 2 − 4x + 1 3 6 2 Rectangle m ´ m, square with sides m 7 7 7

x9 + 2x 2 + C 3

8.

18. f (−1) = 0 21.

19. 12

20. 0.837

AB2 = 242 = 576 BC2 = 322 = 1024 AC2 = 402 = 1600 AB2 + BC2 = 576 + 1024 = 1600 = AC2 ` ΔABC is right angled at +B (Pythagoras’ theorem)

22. 9.

198π (a) units3 7

1 10. 1 units2 3

96π (b) units3 5

11. f l(3) = 20, f m(− 2) = −16 12. 68

(3x + 5)8 24

+C

23. − 5

1 3

24. (1, 1)

25. (a) 1.11 (b) 1.17 26. ^ 0, 3 h maximum, (1, 2) minimum, (−1, 2) minimum 27. 2

8 15

28. (a) 1.58 units2

(b)

5π units3 2

377

378

Maths In Focus Mathematics HSC Course

29. 12

2 3

30. 10

43. (c), (d) 44. (a), (b)

2 units2 3

31. +B is common +BDC = +ACB = 90° ` ΔABC 0 on both LHS and RHS of (0, 0) 33. 2

45. (c)

37. f (x) = x3 − 4x2 − 3x + 20

Chapter 4: Exponential and logarithmic functions

Exercises 4.1 1.

(a) 4.48 (b) 0.14

2.

(a)

(c) 2.70 (d) 0.05 (e) -0.14

38.

(b)

39.

(c) Let ABCD be a rhombus with AC = x and BD = y. +AEB = 90° (diagonals perpendicular in rhombus)

1 DE = BE = y 2 (diagonals bisect each other)

1 1 1 ´ x ´ y = xy 4 2 2 1 1 1 ΔADC has area ´ x ´ y = xy 4 2 2 1 1 1 ` ABCD has area xy + xy = xy 4 4 2

ΔACB has area

40.

AB AG = AC AD AG AF = AD AE AB AF ` = AC AE

(equal ratios of intercepts, BG < CD)

3.

(b) − ex

(e) 3ex ] ex + 1 g 2 (equal ratios of intercepts, GF < DE)

(h) ex ] x + 1 g (i)

xn + 1 +C n+1 d (b) Since (C) = 0, the primitive function dx could include C.

(c) ex + 2x (d) 6x2 − 6x + 5 − ex (f) 7 ex (ex + 5)6 ex (x − 1) x2

(g) 4ex ] 2ex − 3 g

(j) xex ] x + 2 g

(k) (2x + 1)ex + 2ex = ex (2x + 3) (l)

2 41. f (2) = 1 3 42. (a)

(a) 9ex

(m)

ex (7x − 10) (7x − 3) 2

5ex − 5xex 5 (1 − x) = ex e 2x

4.

f l(1) = 6 − e; f m(1) = 6 − e

7.

19.81 8. ex + y = 0 9. x + e3 y − 3 − e6 = 0

5. e

6. − e− 5 = −

1 e5

ANSWERS

10. c −1, −

11. 12.

dy dx

1 m min e

= 7ex; dy

d2 y dx

2

= 7e x = y

= 2ex;

d2 y

dx dx2 x y = 2e + 1 ` y − 1 = 2e x d2 y =y−1 ` dx2

11.

y = 3e 2 x dy = 6e 2 x dx d2 y = 12e2x dx2 d2 y dy −3 + 2y LHS = 2 dx dx 2x = 12e − 3 (6e2x) + 2 (3e2x) = 12e2x − 18e2x + 6e2x =0 = RHS d2 y dy ` −3 + 2y = 0 dx dx2

12.

y = aebx dy = baebx dx d2 y = b2aebx dx2 = b2 y

= 2e x

13. n = − 15 14.

Exercises 4.2 1.

(a) 7e7x

(b) − e− x

(e) (3x + 5) e 2

(i) 2e2x + 1

(c) 6e6x − 2

(d) 2xex

(f) 5e

(g) − 2e

x 3 + 5x + 7

(j) 2x + 2 − e1 − x

(l) e2x (2x + 1) (m) (o)

5x

+1

− 2x

(h) 10e10x

(k) 5 (1 + 4e4x) (x + e4x)4

e3x (3x − 2) x3

(n) x2 e5x (5x + 3)

4e2x + 1 (x + 2)

15.

(2x + 5)2

2.

28e2x (e2x + 1)5 (7e2x + 1)

3.

f (1) = 3e; f m(0) = 9e− 2

5.

x + y − 1 = 0 6.

7.

y = 2ex − e

9.

(0, 0) min; (−1, e − 2) max

8.

2



4.

5

1 3e 3

f m(−1) = −18 − 4e2

Exercises 4.3 1.

(a)

1 2x e +C 2

(e) − (h) 10.

dy dx d2 y dx2

= 4e 4 x − 4e − 4 x = 16e4x + 16e− 4x = 16 (e4x + e− 4x) = 16y

2.

(b)

1 − 2x e +C 2

1 2t e +C 2

1 4x e +C 4

(f) (i)

(c) − e− x + C

1 4x + 1 +C e 4

1 7x e − 2x + C 7

(g) −

(d)

1 5x e +C 5

3 5x e +C 5

(j) ex − 3 +

x2 +C 2

1 5 1 2 (e − 1) (b) e− 2 − 1 = 2 − 1 (c) e7 (e9 − 1) 5 3 e 1 4 2 1 4 1 1 (d) 19 − e (e − 1) (e) e + 1 (f) e2 − e − 1 2 2 2 2 1 6 1 −3 (g) e + e − 1 2 2 (a)

379

380

Maths In Focus Mathematics HSC Course

3. 4. 6. 9.

15. Curves are symmetrical about the line y = x.

(a) 0.32 (b) 268.29 (c) 37 855.68 (d) 346.85 (e) 755.19 1 e4 − e2 = e2 (e2 − 1) units2 5. (e − e− 3) units2 4 π 2 2 2.86 units 7. 29.5 units 8. (e6 − 1) units3 2 4.8 units3 10. 7.4

11. (a) x (2 + x) ex

(b) x2 ex + C

12. πe units3 13.

1 4 (e − 5) units2 2

Exercises 4.4 1.

(a) 4 (b) 2 (c) 3 (d) 1 (e) 2

2.

(a) 9 (b) 3 (c) −1 (d) 12 (e) 8 (i) 1 (j) 2

3.

(h) 4.

5.

1 2 1 (i) 1 2

(a) −1 (b) 1 3

(c)

1 2

(d) −2 (e)

(j) − 1

(f ) 1 (g) 0

(h) 7

(f ) 4 (g) 14 (h) 14

1 4

Exercises 4.5 (f ) −

1 3

(g) −

1 2

1.

1 2

(a) 3.08 (b) 2.94 (c) 3.22 (d) 4.94 (e) 10.40 (f ) 7.04 (g) 0.59 (h) 3.51 (i) 0.43 (j) 2.21 (a) log3 y = x (b) log5 z = x (c) logx y = 2 (d) log2 a = b (e) logb d = 3 (f ) log8 y = x (g) log6 y = x (h) loge y = x (i) loga y = x (j) loge Q = x (a) 3x = 5 (b) ax = 7 (c) 3b = a (d) x9 = y (e) ay = b (f ) 2y = 6 (g) 3y = x (h) 10y = 9 (i) ey = 4 (j) 7y = x

7.

(a) x = 1 000 000 (b) x = 243 (c) x = 7 (d) x = 2 (e) x = − 1 (f ) x = 3 (g) x = 44.7 (h) x = 10 000 (i) x = 8 (j) x = 64 y = 5 9. 44.7

10. 2.44 11. 0

13. (a) 1 (b) (i) 3 (ii) 2 (vii) 3 (viii) 5

(iii) 5 (iv)

(ix) 7

12. 1 1 2

(x) 1 (xi) e

14. Domain: x > 0; range: all real y

(a) loga 4y

(b) loga 20 (c) loga 4 (d) loga

(e) logx y3 z

6.

8.

16. x = ey

(i) log10 ab4 c3

(f ) logk 9y3 (j) log3

(g) loga

x5 y2

b 5

(h) loga

xy z

p3 q r2

2.

(a) 1.19 (b) -0.47 (c) 1.55 (d) 1.66 (e) 1.08 (f ) 1.36 (g) 2.02 (h) 1.83 (i) 2.36 (j) 2.19

3.

(a) 2 (b) 6 (c) 2 (d) 3 (e) 1 1 (h) (i) -2 (j) 4 2

4.

(a) x + y (b) x − y (c) 3x (d) 2y (e) 2x (g) x + 1 (h) 1 − y (i) 2x + 1 (j) 3y − 1

5.

(a) p + q (b) 3q (c) q − p (d) 2p (e) p + 5q (f ) 2p − q (g) p + 1 (h) 1 − 2q (i) 3 + q (j) p − 1 − q

6.

(a) 1.3 (b) 12.8 (c) 16.2 (d) 9.1 (e) 6.7 (f ) 23.8 (g) -3.7 (h) 3 (i) 22.2 (j) 23

7.

(a) x = 4 (b) y = 28 (c) x = 48 (d) x = 3 (e) k = 6

(v) -1 (vi) 2

(f ) 3 (g) 7

(f ) x + 2y

Exercises 4.6 1.

(a) 1.58 (b) 1.80 (c) 2.41 (d) 3.58 (e) 2.85 (f ) 2.66 (g) 1.40 (h) 4.55 (i) 4.59 (j) 7.29

2.

(a) x = 1.6 (b) x = 1.5 (c) x = 1.4 (d) x = 3.9 (e) x = 2.2 (f ) x = 2.3 (g) x = 6.2 (h) x = 2.8 (i) x = 2.9 (j) x = 2.4

3.

(a) x = 2.58 (b) y = 1.68 (c) x = 2.73 (d) m = 1.78 (e) k = 2.82 (f ) t = 1.26 (g) x = 1.15 (h) p = 5.83 (i) x = 3.17 (j) n = 2.58

4.

(a) x = 0.9 (b) n = 0.9 (c) x = 6.6 (d) n = 1.2 (e) x = − 0.2 (f ) n = 2.2 (g) x = 2.2 (h) k = 0.9 (i) x = 3.6 (j) y = 0.6

5.

(a) x = 5.30 (b) t = 0.536 (c) t = 3.62 (d) x = 3.81 (e) n = 3.40 (f ) t = 0.536 (g) t = 24.6 (h) k = 67.2 (i) t = 54.9 (j) k = − 43.3

ANSWERS

Exercises 4.7 1.

(a) 1 +

1 x

(b) −

(c) 1 x

(c)

3 3x + 1

(d)

2x x2 − 4

15x2 + 3 10x2 + 2x + 5 5 (f) + 2x = 3 5x + 1 5x + 1 5 x + 3x − 9 8 6x + 5 1 (g) 6x + 5 + (h) (i) x 8x − 9 (x + 2) (3x − 1) − 30 4 2 (j) − = 4x + 1 2x − 7 (4x + 1) (2x − 7)

(e)

5 1 (1 + loge x)4 (l) 9 c − 1 m (ln x − x)8 x x 4 1 3 (m) (loge x) (n) 6 c 2x + m (x2 + loge x)5 x x 1 − loge x (o) 1 + loge x (p) x2

(k)

2x + 1 x3 + 3x2 loge (x + 1) + 2 loge x (r) x x+1 x − 2 − x loge x 1 (t) (s) x loge x x (x − 2) 2

(q)

(u)

12.

14. 4 ln 4 $ x − y + 4 = 0 15. 3 loge 3 $ x + y − 1 − 9 loge 3 = 0

Exercises 4.8 1.

(b) loge (2x2 + 1) + C 1 1 (c) ln (x5 − 2) + C (d) loge x + C or loge 2x + C 2 2 5 (e) 2 ln x + C (f) loge x + C (g) loge (x2 − 3x) + C 3 3 1 (h) ln (x2 + 2) + C (i) loge (x2 + 7) + C 2 2 1 2 (j) loge (x + 2x − 5) + C 2

2.

(a) ln (4x − 1) + C

e2x (2x loge x − 1) x (loge x) 2

1 (v) ex c + loge x m x (w)

10 loge x x

1 2. 3. 4. x − 2y − 2 + 2 loge 2 = 0 x loge 10 2 7. 5x + y − loge 5 − 25 = 0 5. y = x − 2 6. − 5 1 1 1 1 8. 5x − 19y + 19 loge 19 − 15 = 0 9. d , loge − n 2 2 2 4 1 10. c e, m maximum e

1 f l(1) = − 2

11. (a)

2 13. (a) 3x ln 3 (b) 10x ln 10 (2x + 5) loge 3 (c) 3 ln 2 ´ 23x − 4

(a) loge (2x + 5) + C

1 1 ln (2x3 − 7) + C (d) loge (2x6 + 5) + C 6 12 1 (e) loge (x2 + 6x + 2) + C 2

(c)

3.

(a) 0.5

(b) 0.7 (c) 1.6 (d) 3. 1 (e) 0.5

4.

loge 3 − loge 2 = loge 1.5 units2

6.

(0.5 + loge 2) units2

8.

π loge 3 units3

12. (a) RHS = =

5. loge 2 units2

7. 0.61 units2

9. 2π loge 9 units3

10. 47.2 units2 11.

(b)

(b) loge (x + 3) + C

π 2 4 e (e − 1) units3 2

1 2 + x+3 x−3 1 (x − 3)

(x + 3) (x − 3) 2 (x + 3) + (x + 3) (x − 3) x − 3 + 2x + 6 = x2 − 9 3x + 3 = 2 x −9 = LHS 3x + 3 1 2 ` 2 = + x −9 x+3 x−3 (b) loge (x + 3) + 2 loge (x − 3) + C

381

382

Maths In Focus Mathematics HSC Course

17. 2x + y − ln 2 − 4 = 0

5 x−1 x−1 5 = − x−1 x−1 x−6 = x−1 = LHS

13. (a) RHS = 1 −

`

18. (0, 0) point of inflexion, (− 3, − 27e− 3) minimum 19. 5.36 units2 20. (a) 0.65 (b) 1.3

Challenge exercise 4

x−6 5 =1− x−1 x−1

(e2x + x)

(b) x − 5 loge (x − 1) + C 14.

2x − 1

3 +C 2 loge 3

1.

15. 1.86 units2

Test yourself 4 1.

(a) 6.39 (b) 1.98 (c) 3.26 (d) 1.40 (e) 0.792 (f) 3.91 (g) 5.72 (h) 72.4 (i) 6 (j) 2

2.

(a) 5e5x

(b) − 2e1 − x

(c)

1 x

(d)

4 4x + 5

1 − ln x (g) 10ex (ex + 1)9 x2 1 1 (a) e4x + C (b) ln (x2 − 9) + C 4 2

(e) ex (x + 1)

(f) 3.

4. 7.

4.

9 c 4e 4 x +

6.



9.

d 2 3x (x loge x) = x (1 + 2 loge x); 18 loge 3 10. +C dx loge 3

9.

e (e2 − 1) units2

10. (a) 2.16 units2 (b) x = ey 11. (a) x = 1.9 (b) x = 1.9 (c) x = 3 (d) x = 36 (e) t = 18.2

13.

1 m 4x (e + loge x) 8 x

2 2x − 3

7.

12 units3

(b) x − y − 1 = 0; x − loge 10 $ y − 1 = 0

e 2x 1 + loge x − x loge x ex

y = ex + e− x dy = ex − e− x dx d2 y = ex − (− e− x) dx2 = ex + e− x =y

15.

y = 3e 5 x − 2 dy = 15e5x dx d2 y = 75e5x dx2 d2 y dy −4 − 5y − 10 LHS = dx dx2 5x = 75e − 4 (15e5x) − 5 (3e5x − 2) − 10 = 75e5x − 60e5x − 15e5x + 10 − 10 =0 = RHS

(c) 2.16 units2

3 2 (e − 1) 2 1 (b) ln 10 3 1 (c) 8 + 3 ln 2 6

13. e4 x − y − 3e4 = 0 14. 0.9

16. f (x) = 3e2x − 6x

15. (a) e (e − 1) units2 π (b) e2 (e2 − 1) units3 2

17.

(b) logx

k2 p 3

8. 5x loge 5

14.

12. (a)

16. (a) loga x5 y3

5. 0.42 units2

ex (1 + loge x) − xex loge x

=

(a) 0.92 (b) 1.08 (c) 0.2 (d) 1.36 (e) 0.64

(c) 2.6

1 (c) f 1 − p units 12. 0.645 units2 loge 10

(c) − e− x + C

8.

2e

(a) 2.8 (b) 1.8

11. (a) (1, 0)

1 6. e4 (e6 − 1) units2 2

2.

3.

(d) ln (x + 4) + C e2 3x − y + 3 = 0 5. − 2 e +1 π 6 6 3 e (e − 1) units 6

1 − (2e2x + 1) loge x x (e2x + x) 2

ANSWERS

Chapter 5: Trigonometric functions

5.

π π π π cos + sin sin 4 4 3 3 3 1 1 1 = ´ + ´ 2 2 2 2

LHS = cos

Exercises 5.1 1.

2.

(a) 36° (b) 120° (c) 225° (d) 210° (g) 240° (h) 420° (i) 20° (j) 50° π (b) 6

3π (a) 4 (h)

5π 2

(i)

5π (c) 6

5π 4

(j)

4π (d) 3

(e) 540°

5π (e) 3

7π (f) 20

(f) 140°

3 1 + 2 2 2 2 π π π π cos + cos sin RHS = sin 4 4 6 6 3 1 1 1 = ´ ´ + 2 2 2 2 3 1 = + 2 2 2 2 LHS = RHS π π π π π π So cos cos + sin sin = sin cos + 4 4 4 6 3 3 π π cos sin 4 6 =

π (g) 12

2π 3

3.

(a) 0.98

(b) 1.19

(c) 1.78

(d) 1.54

(e) 0.88

4.

(a) 0.32

(b) 0.61

(c) 1.78

(d) 1.54

(e) 0.88

5.

(a) 62° 27’ (b) 44° 0’ (c) 66° 28’ (d) 56° 43’ (e) 18° 20’ (f) 183° 21’ (g) 154° 42’ (h) 246° 57’ (i) 320° 51’ (j) 6° 18’

6.

(a) 0.34 (b) 0.07 (c) 0.06 (d) 0.83 (e) −1.14 (f) 0.33 (g) −1.50 (h) 0.06 (i) −0.73 (j) 0.16

6.

(a)

7.

(a)

8.

(a)

9.

(a)

Exercises 5.2 1. π 3

π 4

π 6

sin

3 2

1

1 2

cos

1 2

1

tan

2

1

1

3 2

cosec

3 2

2

3 2

2

10. (a)

3 2

sec

1

cot

2.

(a) (g)

3.

4.

(a) 1

(a)

1 2

(c)

2− 3

(h)

(b)

1 4

(b)

6+ 4

2

3 3 8

(b)

(d)

2+2 2

6− 2 4

3

1

3

1 3

2

2

(c)

4 3 3

(i) 3 2

3

(e) 0

3−2 2 2 (d) 1

(f) (j)

(e) 4

2 3+1 2

2+ 3 2 1 4

3π 4π π = − 4 4 4 π =π− 4 5π 6π π = − 6 6 6 π =π− 6 7 π 8π π = − 4 4 4 π = 2π − 4 4π 3π π = + 3 3 3 π =π+ 3 5 π 6π π = − 3 3 3 π = 2π − 3

11. (a) − 1

12. (a) (i)

(b) (i)

13. (a)

3−2 (e)

(b)

3 2

(c) −

(b) 2nd

(c)

(b) 4th

(c) − 1

(b) 3rd

(c) −

(b) 4th

(c) −

(c) − 3

(d) −

13π 12π π = + 6 6 6 π = 2π + 6 1 2

π 5π , 3 3

1

(b) 2nd

2

1 2

1 2

3 2

1

(ii) 1st

(iii) −

(ii)

3

(b)

5 π 7π , 4 4

(c)

1 2

(e)

2

π 5π , 4 4

1

(d)

3

3 2

(iii)

(iv)

1

3

(v) −

3 2

π 4π , 3 3

5π 7π , 6 6

14. (a) sin θ (b) − tan x (c) − cos α

(d) sin x

(e) cot θ

383

384

Maths In Focus Mathematics HSC Course

15. sin2 θ

10. 9.4 cm2

3 4 16. cos x = − ; sin x = 5 5

11. (a)

17. x =

π 3π 5π 7π , , , 4 4 4 4

(c) 22 +

(b) π m

25π cm 3

π 7π cm (e) mm 4 2

1.

(a) 4π cm

2.

(a) 0.65 m (b) 3.92 cm (c) 6.91 mm (d) 2.39 cm (e) 3.03 m

3.

1.8 m

8.

13

V=

4. 7.5 m 5.

(c)

2π 21

(d)

6. 25 mm 7. 1.83

9. 25.3 mm 10. SA =

15. (a)

175π cm2, 36

(b)

3π 2 m 2

(c)

125π cm2 3

(d)

3π cm2 4

49π (e) mm2 8

20π 4 (18 + 5π ) = cm 9 9

225π cm3 2

(a) 0.48 m2 (e) 3.18 m2

3.

16.6 m2

(b) 6.29 cm2

7.

6845 mm2 8. 75 cm2 9. 11.97 cm2 8π

4. θ = 4

15 (7π + 24) 105π cm2 + 180 = 2 2

1.

(a) 0.045 (b) 0.003

2.

1 4

4.

1 343 622 km 5. 7367 m

3.

(c) 0.999 (d) 0.065 (e) 0.005

1 3

4 9

(c) 24.88 mm2

5. 6 m

6. (a)

7π cm 6

(d) 7.05 cm2

(b)

1.

y

(a)

49π cm2 12

1

π , r = 3 cm 15

x

Exercises 5.5

π 2

π

3π 2



π 2

π

3π 2



-1

6π − 9 3 2 125π − 75 m (c) cm2 4 3 3 (π − 3) 49 (π − 2 2 ) (d) cm2 (e) mm2 4 8 (a) 8π cm2

(b)

(a) 0.01 m2 (b) 1.45 cm2 (c) 3.65 mm2 (d) 0.19 cm2 (e) 0.99 m2

3.

0.22 cm

4.

5.

134.4 cm

6. (a) 2.6 cm (b)

7.

(a) 10.5 mm (b) 4.3 mm2

8.

(a)

9.

(a) 77° 22l (b) 70.3 cm2 (c) 26.96 cm2 (d) 425.43 cm2

25π cm2 4

3π (a) cm 7

y

(b)

2.

2

(b)

(b) 3:7

Exercises 5.7

2.

1.

(b) 24π cm2

Exercises 5.6

125 35 π cm3 648

(a) 8π cm2

10. θ =

121π 396 − 121π cm2 = 18 18

11π 11 (18 + π ) = cm 9 9

13. (a) 10π cm 14. (a) 8 +

Exercises 5.4 1.

(b) 22 −

12. (a) 5 cm2 (b) 0.3% (c) 15.6 cm

Exercises 5.3

7 mm 9

11π cm 9

9π (b) cm2 14

(c) 0.07 cm

5π cm 6

(c) 0.29 cm2

2

2

(b) 0.5 cm2

x

-2

ANSWERS

(c)

y

y

(g)

2

4 3 2

1

1

x 3π 2

π

π 2



3π 2

y

(h)



x

π

π 2

-1

5

y

(d)

π 2

x

π

3

2

(i)



3π 2

y

1

π 2 (e)

π

x 3π 2



3

y

(j)

x

π

π 2

3π 2



y

3

x π 2

π



3π 2

1 π 2

-3

π



3π 2

y

(f)

2.

(a)

y

4

π 2 -4

π

x 3π 2



1

π 4

-1

π 2

3π 4

π

5π 4

3π 2

7π 4



x

x

385

386

Maths In Focus Mathematics HSC Course

(b)

(g)

y

π 4

π 2

π

3π 4

5π 4

7π 4

3π 2



y

π 3

x

(h) (c)

2π 3

π

4π 3

5π 3



x

y

y

3

1

π 3

2π 3

4π 3

π

5π 3

x



π 2

–1

π

3π 2

x



-3 y

(d)

(i)

y

3

2 π 4

π 2

3π 4

π

5π 4

3π 2

7π 4



x x π

π 2

–3



3π 2

−2 (e)

y

6

π 3

2π 3

π

4π 3

5π 3



x

4

-6 (f)

y

(j)

y π 2

π

x 2π

-4

π

3π 2



x

ANSWERS

3.

(a)

y

y

4.

8 1



π - 3π - 2 4

-

π 4

π 2

π 4

3π 4

π

x

-1

7



3π - π 2 4



x



-8

y

(b)



π

5.

π 4

π 4

π 2

3π 4

π

(a)

y

1

x

-7 (c)

π 2

y

3π 2

π

x



-1 (b) y



-

3π - π 2 4

-

π 2

π 4

π 4

3π 4

π

x

y

(d)

5



-

3π π 4 2

-

π

π 2

(c)

π 4

π 4

π 2

3π 4

x

π

3π 2



x

y

1

-5 (e)

y

x

π 2

π

3π 2



π 2

π

3π 2



-1 2 (d)



-

3π - π 2 4

-

π 4

π 4

π 2

3π 4

π

x

y

3

-2

-3

x

387

388

Maths In Focus Mathematics HSC Course

(e)

(j)

y

y

5

2

4

π 2

3π 2

π



3

x

2 1

-2 (f)

y

π 2

-1

π

3π 2



x

4 6.

π 4

π 2

3π 4

π

5π 4

3π 2

7π 4



(a)

y

x

1

-4

-2

(g)

-1

1

2

1

2

x

y

-1 1 (b)

π 4

π 2

3π 4

π

5π 4

3π 2

7π 4

x



3

-2

-1 (h)

y

x

-1 -3

y

7.

(a)

y

1 π 4

π 2

3π 4

π

5π 4

3π 2

7π 4



y = sin x

1

x

π 2

-1 (i)

π y = sin 2x

3π 2



x

y y

(b)

3 2 2 1

1

-1

π 2

π

3π 2



x

-1 -2

π 2

π

3π 2



x

ANSWERS

8.

(a)

10. (a)

y

y

5 4

2

3

y = cos x + sin x

2

1

y = 2 cos x

1 π 2

−1

π



3π 2

−2

x

y = 3 sin x

3π 2



3π 2



x

-1

−3 −4

-2

−5 (b)

π

π 2

y

y

(b)

5 4

2

3

y = sin 2x – sin x

2

1

1

π 2

−1

π

−2 −3 −4

3π 2



x

x

–1

y = 2 cos x + 3 sin x

−5

9.

π

π 2

–2

y

y

(c)

y = cos 2x - cos x

2

y = sin x + 2 cos 2x

2

1 1

-1

π 2

π

3π 2



x π 2

π



3π 2

–1

-2 –2 –3

(d)

y 3 2

y =3 cos x − cos 2x

1

−1 −2 −3 −4

π 2

π

3π 2



x

x

389

Maths In Focus Mathematics HSC Course

6.

y

(e)

x = 0.8, 4 y

3 2 1

1

π

π 2

-1

3π 2



y = sin x - sin

-2

y = cos x

x

x 2

π 2

-3

Exercises 5.8 1.

(a)

π

3π 2

y = sin x

7.

(a) Period 12 months, amplitude 1.5 (b) 5.30 p.m.

8.

(a) 1300 (b) (i) 1600 (ii) 1010 (c) Amplitude 300, period 10 years

9.

(a)

y y= x 2

x



-1

-4

18 16 14

1

12 10 8 6

1 π 2 2

3 π

4 3π 5 2

x



6

4 2 0 January February March

y = sin x

-1

April

May

June

July

August

(b) It may be periodic - hard to tell from this data. Period would be about 10 months. (c) Amplitude is 1.5

There are 2 points of intersection, so there are 2 solutions to the equation. y (b)

10. (a)

4 3.5

y=

x 2

3 2.5

1

2

y = sin x

1.5 1 0.5



(b) Period 24 hours, amplitude 1.25 (c) 2.5 m There are 3 points of intersection, so there are 3 solutions to the equation. 2.

x =0

3. x = 1.5

4. x = 0, 4.5 5. x = 0, 1

Exercises 5.9 1.

(a) 4 cos 4x (b) − 3 sin 3x (c) 5 sec2 5x (d) 3 sec2 (3x + 1) (e) sin (− x) (f) 3 cos x (g) − 20 sin (5x − 3) (h) − 6x2 sin (x3) (i) 14x sec2 (x2 + 5) (j) 3 cos 3x − 8 sin 8x

pm

m 5p

6.1

11 .48

m

am .55

11

pm

6.2

0a

m

.48 11

m

5p

6.1

am

5a

11 .5

pm

6.2 0

11 .48

am .55

11

-1

m

0 5p

π 2 2

am

1

6.1

π -π -3 -2 - -1 2

x

6.2 0

390

ANSWERS

(k) sec2 (π + x) + 2x

(l) x sec2 x + tan x

(m) 3 sin 2x sec2 3x + 2 tan 3x cos 2x (n) (o)

2x cos x − 2 sin x x cos x − sin x = 4x 2 2x 2

13. (a) 14.

3 sin 5x − 5 (3x + 4) cos 5x sin2 5x

(p) 9 (2 + 7 sec2 7x) (2x + tan 7x)8 (q) 2 sin x cos x

(r) − 45 sin 5x cos2 5x

1 cos (1 − loge x) x cos x (u) (ex + 1) cos (ex + x) (v) = cot x sin x (w) − 2e3x sin 2x + 3e3x cos 2x = e3x (3 cos 2x − 2 sin 2x) (s) ex + 2 sin 2x

(t) −

5.

− sin x = − tan x cos x

7.

sec2 xetanx

9.

y = 2 cos 5x dy = − 10 sin 5x dx 2 d y = − 50 cos 5x dx2 = − 25 (2 cos 5x) = − 25y

10.

y = 2 sin 3x − 5 cos 3x dy = 6 cos 3x + 15 sin 3x dx d2 y = −18 sin 3x + 45 cos 3x dx2 = − 9 (2 sin 3x − 5 cos 3x) = − 9y

1.

(a) sin x + C

(i) −

12 4. 6 3 x − 12y + 6 − π 3 = 0 2

=−

(b) − cos x + C

1 cos (2x − 3) + C 2

(j)

2.

(e)

1 π

3 3 (g) 4

1 2

(f)

1

3.

4 units2 4.

6.

0.51 units3 7.

8.

3−

10. (a)

1

3−

(a) 1 (b)

3 2

=

=

d [loge (tan x)] dx sec2 x = tan x tan2 x + 1 = tan x tan2 x 1 = + tan x tan x = tan x + cot x = RHS

`

d 7 loge (tan x) A = tan x + cot x dx

π 12. d , 3

d

3−

π n maximum, 3

5π 5π n minimum ,− 3 − 3 3

(f)

1 cos 7x + C 7

1 sin (2x − 1) + C 2

2 3 3 (h) −

(c)

2 2

=

2

5. 0.86 units2

π units3 4

π 3 3− π = units2 9. 2 2 units2 3 3

V =π

#

b

y 2 dx



#

π 2

cos x dx

0

= π ; sin x E

π 2 0

π = π (sin − sin 0) 2 = π (1 − 0) = π units 3 (b) 3.1 units3 11. y = − 2 sin 3x

(d) −

1 5

2 units2 6

a

11. LHS =

(c) tan x + C

(k) cos (π − x) + C (l) sin (x + π ) + C x 2 (m) tan 7x + C (n) − 8 cos +C 7 2 x (o) 9 tan + C (p) − cos (3 − x) + C 3

2 3 9

8. 8 2 x + 48y − 72 2 − π 2 = 0

f (x) = − 2 sin x f l(x) = − 2 cos x f m(x) = 2 sin x = − f (x)

π cos x° 900

(c)

− 45 1 (d) cos x° + C (e) − cos 3x + C π 3 1 (g) tan 5x + C (h) sin (x + 1) + C 5

3.

3 3

−π sin x° 60

Exercises 5.10

4 cos2 x sin3 x − sin5 x = sin3 x (4 cos2 x − sin2 x)

6. −

(b)

15. a = − 7, b = − 24

2e2x tan 7x − 7e2x sec2 7x (x) tan2 7x e2x (2 tan 7x − 7 sec2 7x) = tan2 7x 2.

π sec2 x° 180

1 3

391

392

Maths In Focus Mathematics HSC Course

Test yourself 5 5π cm 6

1.

(a)

2.

(a)

3.

(a) x =

4.

(a)

3

15. (a) 25π cm2 12

(b) 3 2

(b)

3 2

(c)

3 π 7π , 4 4

(c) 0.295 cm2

(b) x =

(d) −

1 2

π 5π , 6 6 (b) x = 0.6 16.

3− 2 units2 2

17. 2 units2 18. 4x + 8y − 8 − π = 0

19. y = − 3 cos 2x

20. (a)

(b)

(b) x = 0.9, 2.3, 3

Challenge exercise 5 5.

(a) − sin x (e)

6.

(b) 2 cos x

(c) sec2 x

x sec2 x − tan x x2

(f) − 3 sin 3x

1 cos 2x + C 2

(b) 3 sin x + C

(a) −

(d) x cos x + sin x (g) 5 sec2 5x (c)

1 tan 5x + C 5

3− 3 1 1 e1 − o= 6 2 3

1.

0.27 2.

4.

(a) Period = 2, amplitude = 3 (b)

(d) x − cos x + C 1

(a)

8.

3x + 2 y − 1 −

9.

x = cos 2t dx = − 2 sin 2t dt 2 d x = − 4 cos 2t dt2 = − 4x

10.

1 2

2

(b)

2 3 3

7.

3π =0 4

units2 11.

13. − 3 3

14. (a)

5.

(a) y = − sin 3x (b) LHS =

π 3

units3

8π cm2 7

12. (a) 5

(b) 0.12 cm2

(b) 2

d2 y

+ 9y dx2 = 9 sin 3x + 9 (− sin 3x) = 9 sin 3x − 9 sin 3x =0 = RHS

3. r = 64 units, θ =

π 512

ANSWERS

13π 9π π 5π 18. d , 0 n, d , 0 n, d , 0 n, d , 0n 8 8 8 8

6.

19.

20.

7.

π sec2 x° 180

8.

(a)

sec2 x tan x 1 cos2 x = sin x cos x cos x 1 = ´ cos2 x sin x 1 1 = ´ cos x sin x = sec x cosec x = LHS

RHS =

(b) loge 3 = 9.

(c) Amplitude = 1

4 (2π − 3 3 ) 3 π o cm2 = cm2 − 3 2 3

14. 0.204 units3 15.

16.

3π π 10. (a) d , 4 n and d , 2n 4 4

(b) Maximum = 4

180 cos x° + C π

13. −

2−1 units2 2

1 = 2

f (x) = 2 cos 3x f l(x) = − 6 sin 3x f m(x) = −18 cos 3x = − 9(2 cos 3x) = − 9f (x)

1.

(a) R = 20 − 8t

(b) R = 15t2 + 4t

(c) R = 16 − 4x

(d) R = 15t − 4t + 2 (e) R = e (f) R = −15 sin 5θ 100 x 400 (g) R = 2π − 3 (h) R = (i) R = 800 − 2 r r x2 − 4 2 (j) R = 4π r 4

2.

3

t

(a) h = 2t2 − 4t3 + C (b) A = 2x4 + x + C 4 (c) V = π r3 + C (d) d = − 7 cos t + C 3 (e) s = 4e2t − 3t + C

1 loge 3 2

(2x cos 2x + sin 2x) e

12. −



Exercises 6.1

sec x tan x

x sin 2x

11. 8 e

2

Chapter 6: Applications of calculus to the physical world

2

` sec x cosec x =

1

1 2

cos x − sin x sin x + cos x

9 (π − 2) 9 π d − 1n = cm2 4 2 2

3. 9.

20

4. 1

5. 6e12 6. 13

7. 900 8. 2e3 + 5

dM = 1 − 4t; R = − 19 dt [i.e. melting at the rate of 19 g per minute (g min-1)] y = x3 − x2 + x + 6

10. R =

11. − 11 079.25 cm per second (cms-1) 13. 165 cm2 per second (cm2s-1)

12. 21 000 L

14. − 0.25

15. 41 cm2 per minute (cm2 min-1)

16. 31 cm3

17. 108 731 people per year 18. (a) 27 g (b) − 2.7 (i.e. decaying at a rate of 2.7 g per year) 19.

y = e 4x dy = 4e 4 x dx = 4y

20.

S = 2e 2 t + 3 dS = 2 (2e2t ) dt = 2 (2e2t + 3 − 3) = 2 (S − 3)

17.

393

394

Maths In Focus Mathematics HSC Course

Exercises 6.2 1.

20. (a) Q = Aekt dQ = kAekt dt = kQ

(a) 80 (b) 146 (c) 92 days (d)

(b)

2.

(a) 99 061 (b) 7 hours

3.

(a) When t ` M When t ` 95 0.95 ln 0.95 0.01 So M

dQ

= kQ dt dt 1 So = dQ kQ 1 t =# dQ kQ 1 1 dQ = # k Q 1 = ln Q + C k kt = ln Q + C1 kt − C1 = ln Q ekt − C = Q ekt ´ e− C = Q Aekt = Q 1

1

= 0, M = 100 = 100e− kt = 5, M = 95 = 100e− 5k = e − 5k = − 5k =k = 100e− 0.01t

Exercises 6.3 1.

(a)

v

(b) 90.25 kg (c) 67.6 years

t

t

4.

(a) 35.6 L (b) 26.7 minutes

5.

(a) P0 = 5000 (d) 8.8 years

(b) k = 0.157

(c) 12 800 units (b)

6.

2.3 million m2 7. (a) P = 50 000e0.069t (c) 4871 people per year (d) 2040

8.

(a) 65.61° C

9.

(a) 92 kg (b) Reducing at the rate of 5.6 kg per hour (c) 18 hours

v

t

(b) 1 hour 44 minutes

11. (a) B = 15 000e0.073t

a

(b) 70 599

10. (a) M0 = 200; k = 0.00253 (b) 192.5 g (c) Reducing by 0.49 g per year (d) 273.8 years

(c)

v

t

a

(b) 36 008 (c) 79.6 hours

12. 11.4 years 13. (a) 19% (b) 3200 years 14. (a)

a

P (t) = P (t0) e− kt dP (t) = − kP (t0) e− kt dt = − kP (t)

(b) 23% (c) 2% decline per year (d) 8.5 years 15. 12.6 minutes 16. 12.8 years 17. (a) 76.8 mg/dL (b) 9 hours 18. 15.8 s 19. 8.5 years

t

t

ANSWERS

Exercises 6.4

(d)

1.

(a) 18 cms-1 (b) 12 cms-2 (c) When t = 0, x = 0; after 3 s (d) After 5 s

2.

(a) − 8 ms− 1 (b) a = 4; constant acceleration of 4 ms-2 (c) 13 m (d) after 2 s (e) − 5 m (f)

(e)

2.

3.

(a) t2, t4, t6 (b) 0 to t1, t3, t5 (c) 0 to t1 (d) t5

3.

(a) 4 m (b) 40 ms-1 (c) 39 m (d) 84 m (e)

(a)

(b)

4.

(a) O, t2, t4, t6 (b) t1, t3, t5 (c) t5 (d) (i) At rest, accelerating to the left. (ii) Moving to the left with zero acceleration.

5.

(a)

6.

(a) At the origin, with positive velocity and positive constant acceleration (moving to the right and speeding up). (b) To the right of the origin, at rest with negative constant acceleration. (c) To the left of the origin, with negative velocity and positive acceleration (moving to the left and slowing down). (d) To the right of the origin, with negative velocity and acceleration (moving to the left and speeding up). (e) To the left of the origin, at rest with positive acceleration.

π 3π 5π , , ,… 4 4 4

(b) 0,

4.

(a) 2 cm (b) After 1 s (e) -7 cms-1

5.

(a) 2 ms-1 (b) 4e2 ms− 2 (d)

6.

(a) v = − 2 sin 2t

(c) -4 cm

(d) 6 cm

(c) a = 4e2t = 2 (2e2t ) = 2v

π 3π , π, ,… 2 2

(b) a = − 4 cos 2t

π 3π (d) 0, , π , ,...s 2 2

(e) ±1 cm

(c) 1 cm

π 3 π 5π (f) , , ...s 4 4 4

(g) a = − 4 cos 2t = − 4x 7.

(a) v = 3t2 + 12t − 2; a = 6t + 12 (d) 42 ms-1

(b) 266 m (c) 133 ms-1

395

396

Maths In Focus Mathematics HSC Course

8.

9.



••

(a) x = 20 (4t − 3)4 , x = 320 (4t − 3)3 (b) x = 1, cm, ox = 20 cms− 1, px = 320 cms− 2 (c) The particle is on the RHS of the origin, travelling to the right and accelerating. (a) v = 5 − 10t

10. v =

(b) − 95 ms− 1

(b)

dx = 6t2 − 6t + 42 dt

At rest:

(c) a = − 10 = g

− 102 17 ,a= (3t + 1) 2 (3t + 1)3

1 1 cms− 2 cms− 1 (c) − 6 36 (d) The particle is moving to the right but decelerating (e) (e3 − 1) s

11. (a) At the origin

(b)

12. (a) 3 ms-1 (b) When t = 0 s, 1 s, 3 s

dx dt 6t2 − 6t + 42 t2 − t + 7 b2 − 4ac

=0 =0 =0 = (−1) 2 − 4 (1) (7) = − 27 0: v > 0 Also 2t > 0 and 3t + 9 > 0 when t > 0 So 2t < 3t + 9 2t
View more...

Comments

Copyright ©2017 KUPDF Inc.
SUPPORT KUPDF